You are on page 1of 122

‫مصدر االسالة جلوري تيم فقط ‪ ،‬هذا الملف الجراحة ‪ surgery‬شامل من سيبتمبر الى يوم واحد ديسمبر ‪ ،‬نسال

هللا التوفيق وان‬


‫ينفع به ‪.‬‬
‫سبحان هللا والحمد هلل وال اله اال هللا وهللا اكبر وال حول والقوة اال باهلل‬
‫بسم هللا الرحمن الرحيم‬

‫‪1Breast tumor‬‬
🌹Birad 3 =Follow up for short time. F = 3
🌹Birard 4 = core biopsy 🌹 core ‫اربع حروف‬

🌷- female with ULQ lump increasing with time, size 15x15 cm, us show solid mass
with multiple cystic spaces with posterior costic enhancement, management?
a. neoadjuvant chemotherapy b. simple mastectomy✅✅ (case of phylloid tumor)

🌹 Female in her 47 , single, positive family history of breast cancer.


Underwent routine mammography which showed bilateral increased density
and glandular pattern. Core needle biopsy showed atypical ductal hyperplasia.
What’s the appropriate management? A. wire surgical excision. B. Simple mastectomy

🌹 Intraductal hyperplasia in patients with breast cancer family history:


A. simple mastectomyB. wire excision C. radical mastectomy D. preventive chemo

🌷 ULQ lump increasing with time, size 15x15 cm, us show solid mass
with multiple cystic spaces with posterior costic enhancement== (PHYLLOID TUMOR) ==
simple mastectomy🌹
....
🌹patient after modified radical mastectomy complaining of loss of sensation in inner side
of right arm, injury to which nerve is suspected:
A.Long thoracic B. Inter-Costo-Bracial nerve ✅
Inter = inner Costo = mastect Bracial = arm
..........

🌷lactating came with mastitis with abcsess 5 cm tx?


A. I&D ( for abcsess)✅. B. Mastectomy. C. needle aspiration. D. Antibiotic
🌷mastitis with abcsess treat by = incistion and drainge🌷

🌹Acute mastitis organism =staph aureus🌹

🌷mastitis alone treat by = Antibiotic🌷

🌹Lactaional mastitis case = oxacillin with continu breast feeding 🌹

🌹Lady 26 Y with lift breast redness around areola, mass in the right outer quadrant
tender and red , there is lymph node enlargement,temp high, dx
A. inflammation cancer B. abscess C. fibroadenoma

🌹young woman has painful breast lump with redness and tenderness. vitals are given
and show T 38.9 what will u do? A. incision and drainage. B. excision C. FNA

………..
🌹40 years old lady underwent mamogram and US Mamo showed fibroglandula tissue US
shows multiple cysts, BIRAD 3 What to do?
- core bx. -exision bx. -annual screening -short duration follow up✅
🌹What is the meaning of Birads 3?
BIRADS 3 often indicates the need for a 6-month follow-up mammogra

🌹 BIRAD 4, next step in management :? core tissue biopsy. 4= CORE‫اربع حروف مع اربعة‬

🌹 40 y/o female patient presented with breast mass and tethering, mammogram and U/s
were done and showed BIRADS IV, your next step == Core biopsy
…….
🌹25 y/o female with Breast mass 2*2 describe as Oval shape painful:
A. Duct ectasia B. breast cyst C. breast cancer

🌹Young female complaining of breast tenderness associated with menses, US done left
breast showing 3 cysts right breast 2 oval masses, diagnosis? - Fibrocystic change

🌹Young patient with breast lump on exam: Oval with smooth surface 2*2cm dx
A-fibroadenoma✅ B-cyst C-breast cancer

🍓20 years old female , presenting with recent mass in right breast which is mobile =
fibroadenoma
‫ له ثالث عالمات مهمة أوال ترتبط بالهرمون‬fibroadenoma ‫ هو ان‬fibrocyst ‫ و‬fibroadenoma ‫الفرق بين‬
‫ لذلك غالبا يجي في وقت‬، ‫وخاصة باالستروجين يعني كل مازاد االستروجين يزيد حجمها وكلما قل يقل حجمه‬
‫ ثاني شيء انها‬،menstrual cycle ‫ او قبلها ب أسبوع وتختفي او يقل حجمها بعد‬menstrual cycle
9‫ اذا ضغطت عليها تحس بالم وماله أي عالقة بالهرمونات‬cyst ‫ مافي أي الم لما تضغط عليها عكس‬painless
‫ ال‬. ‫ ثابت من اسمها كيس = تابت ما تتحرك‬cyst ‫ الكتلة تتحرك لكن‬adenoma ‫ من اسمها‬mobile ‫ثالثا‬
firm or smooth ‫ يكون سطحها‬adenoma

🌹 50 y women no family Hx of breast ca , mammogram normal When to do next


mammogram ? A. after 1 year B. 2 y C. 3 y D. 5 y

🌹40 years Pt with hx of bloody discharge from breast ? P/E normal Initial invest
A. US B. Mammogram C. MRI

🌹32year old complain of left breast mass with bloody discharge .. on examination the left
breast and axilla is normal what will do next A. Bilateral US B. FNA C. Bilat mammography
‫الن فيه دم‬
🌹 37 y/o Female with family hx of breast cancer complain of breast mass , She was concern
of breast cancer by ex of there was a breast mass with skin tethering Which of the
following is the most important next step ?
A. Bilateral Mammo ✅✅ B. Breast MRI C. Biopsy D. US

🌹 35 year old pregnant came with bilateral breast tenderness and mass what
investigation you’ll do? A. US B. mammography C. reassess after delivery
us ‫ ومافي دم وعمرها اقل من أربعين فنختار‬mass ‫فقط‬
🌹Pregnant patient with breast mass. What the best investigations? Bilateral breast US

🌹breast mass behind nipple, on US there is hypoecoich lesion, what next :


A- FNA ✅✅ B- core biopsy C- exicional biopsy D- reassess after ...

Pregnant with increasing breast mass 3 to 4 cm what to do?


A-Bilateral ultrasound ✅
B-Bilateral memo
C-Delay after pregnancy
D-Reassurance
🌹increasing breast mass 3 to 4 cm =Bilateral ultrasound🌹
.....
🍒Breast screen how frequent = Every 2 yrs 🍒
🌹breast mass, periareolar, nipple pulled inside, unilateral what next step:
first breast imaging, then biopsy, then staging 🌹
.....
37 y/o Female with family hx of breast cancer complain of breast mass , She was concern of
breast cancer by ex of there was a breast mass with skin tethering Which of the following is
the most important next step ?
A. Bilateral Mammo
B.Breast MRI
C.Biopsy
D.US ✅✅ ‫اقل من اربعين‬
..........
🍓Female pt going for breast examination but she wants female doctor to examine her
what you will do:
A. refuse
B. respect ✅

..........
🍓breast pain pre menses by 3 day Px no rmal next = reassurance.
...........
🍓strong family hx of breast come with identified breast mass what next
mamogram
.......
🌹strongest risk factor of breast ca in female : age
-young woman has painful breast lump with redness and tenderness. Vitals are
given and show T 38.9. What will u do?
a. incision and drainage✅
b. excision c. FNA

🌹A 50 year old lady presents with bloody nipple discharge, and it was found to be invasive
intraductal papilloma. The most appropriate management is= Wide local incistion.
🌹Patient with bloody stained nipple discharge, you’re suspecting intraductal papilloma.
What’s the next step? Excision ✅
....
🌹Color of discharge in intraductal papilloma ? A. Red B. Green C. Black
🌹duct ectasia, color of discharge?
A. red
B. blue
C. green ✅
Duct ectasia :Green 💚
Ductal papilloma : red ❤
🌷 breast biopsy shows intraductal papilloma what will u do
a. breast preserving✅(Surgical excision of involved duct to ensure no atypia with
breast preserving) B_radical mastectomy‫خطا‬

🌹breast biopsy shows intraductal papilloma what will u do A. breast preserving B. radical
mastectomy

*Breast Mass:-
After history and clinical examination.
🌷Breast Mass If patient less than 30= *Ultrasound*🌷
🌷Breast mass patient 30 or older ? *Mammogram*🌷
‫ بعد كذا اذا لقيت‬FNA ‫ هنا نستعمل‬cyst ‫ في التراساوند وشكيت انه‬mass ‫يعني اذا انا بالعيادة فحصت ولقيت‬
‫ بينما لو طلع‬9‫ وشفاف ومافي دم خالص اطمن المريض واتابعها‬9‫ كان نظيف‬cyst ‫بنتيجة العينة انه السائل الي طلع من‬
‫ اذا‬core biopsy ‫ أيضا متى استخدم ال‬، core biopsy ‫ فيه دم هنا الزم اخذ عينه ثانية من االنسجة اسمها‬cyst
. ‫ فيه كتلة واضحة‬mammogram ‫طلع بنتيجة ال‬
🌷Clinically suspicious Breast Mass =Fine needle aspirantion .
🌷 breast mass on mammogram = Core needle biopsy🌷
*Cystic on Ultrasound or FNA results:-*
-if cyst aspirantion show Clear fluids *Reassure & follow*
-if cyst aspirantion show Residual mass or Bloody= *Core needle biopsy*
*Solid on ultrasound results:-*
Likely benign? *Treat as needed*. -suspicious? *Core needle biopsy*
...... .... ....

🌹Management of atypical ductal hyperplasia?


A. Wide local excision B. Simple mastectomy
‫ الي فيها سرطان واالنسجة الي حولها السليمة‬Duct ‫ فالغالج نشيل هذه‬Duct ‫قالك ورم في‬

32 yo I think female concerned about breast cancer, because her mother


have it and her sister has ovarian cancer. What to do?
A. Mammogram B. US C. BRACA

15*15 mass in the breast getting larger and the skin overlying is intake and thin
due to mass effect what to do
A. Chemo. B. Radio. C. Radical mast. D. Simple mastectomy

1 phylloid tumor
Phelloid Tx:
Ÿ If he say in the case Benign > WLE
Ÿ If he say in the Case milgnancy>Simple mastectomy
Ÿ If he say Suspescion > You should start Mx as it Benign By WLE

🍒Benign breast phyllodes tumor = Wide local excision🍒‫فيل رجله واسعه‬


. . .. . . . .
🌹4-5 cm malignant phyllodes mx? A.simple mastectomy B. wide local excision 🌹✅

🍒Malignancy phyllodes tumor = Simple mastectomy 🍒‫الن خبيث الزم استئصال‬


...... . . . .. .
🌹years lady with breast lump 2*2 i think for year suddenly the mass increased in size?
phylloids
….
🌹Case showed cystophylloides breast treatment=
a. wide local excision✅ b. mastectomy c. follow up after 6 months
.....
🌹Lady breast mass 2 years ago but 6 month ago started to get bigger, mass well defined
mobile not attached diagnosis = A) fibroadenoma B) phyllode✅
….
with painful breast mass since 6months. It get increased.it is in the outer upper surface
(around 9 clock) Histopathology confirmed a benign What is the best next step?
A. simple mastectomy B. Wide local excision
......
🌹A 24 year old lady presents with a hard, mobile, well-circumscribed painless left breast
mass that has been increasing in size from the past few months, and was NOT related to
her menstrual cycle. The most like Dx is A. fat cyst B. Fibroadenoma C.Phylloid
……
🌹Pt with malignant phylloid tumor what is next? A.Contrast ct of chest B.pet scan

🌹women with 11*12 breast mass, examination showed no palpable LN. Core
biopsy was taken and showed malignant phylloid tumor, what’s the next appropriate
step? A.WLE B.PET scan C.Chest CT witout contrast

1pancreatitis
🌹typical pancreatitis: epigastric pain, high amylase what is next step? A. US B. CT scan

🌹Pancreatitis case borderline to high amylase + k/c of gallstone what you will do next? A. US B. CT scan

🌹patient who presented w/ signs and symptoms of cholangitis of biliary pancreatitis, q was "What is the
most important initial mx"? A. U/S abdomen B. CT abdomen C. Urgent ERCP D. IV fluids

🌹Scenario of abd pain post meal with a hx of pancreatitis month ago Fluid collection found what is it?
A. Cyst. B. Pseudo. C. Hematoma

🌹Pancreatitis 5 weeks ago. Now she has epigastric tenderness and cannot tolerate food with
vomiting each time. By ultrasound you found large about 12X10 mass With thick wall and fluid
inside. Labs: 346 amylase, Wbc 15k. What is the diagnosis?*
A. Pseudocyst B. Abscess C. Walled off necrosis

🌹A Case of chronic pancreatitis what you will find?!@


A. Increase insulin resistance B. Decrease gluconeogenesis C. increase lipolysis ‫اصحهم‬
D. Hypoglycemia ( Answer is : hyperglycemia, hypocalcemia)

🌹40 years old woman with no pain but you noticed jaundice. She has high direct
bilirubin and high ALT what is your diagnosis? Carcinoma in head of pancreas ✅

🌹 pt with pulstile epigastric mass and ask about investigation :


Serum k, Serum amylase ✅. Serum cl
Abdominal aortic anyurysm AAA ‫طيب عشان نفهم السؤوال اوال تشخيص الحالة هو‬
Ischemic pancreatitis : ‫ هو‬AAA ‫ طيب من مضاعفات الي تعملها‬epigastric ‫ في منطقة‬mass pulstile ‫الن قال‬
. ‫ وهو انزيم يرتفع في حالة حدوث مشكلة في البنكرياس‬. amylase ‫وبالتالي نختار‬

🍓patient with epigastric pain physical exam showed diffuse guarding and sluggish bowl
sound amylase was hight what is the next step:
erect chest xray✅. abdominal xray CT abdomen

🌹 case of acute pancreatities : Iv fluids and analgesic Abx

🌹 Diagnosed case of acute pancreatitis. Received IVF + analgesic Next step?


- Urgent surgical consult'tion✅✅ - Antibiotics
......
🌹 30 y/o Cystic fibrosis with GI Symptoms? what complication he might have?
A- *Acute* pancreatitis ✅ ✔ B-Chronic Pancreatitis C- Pancreatic cancer D- Stricture
‫ )كيف يعني ؟ يعني اي مخاط بالجسم او انزيم سائل مع‬mucus( ‫ اوال هو مرض وراثي ياثر على‬، ‫ بشكل مبسط جدا‬Cystic fibrosis ‫بحاول اشرح ايش هو‬
‫ المخاط يكون لزج ويقدر يطلع بسهولة ليه ؟ بسبب فيه‬، ‫ للمخاط‬water ‫ الن فيه خلل جيني مايدخل‬، ‫ ليه ثقيل‬، ‫هذا المرض يكون هذا المخاط ثقيل ثقيل جدا‬
‫ فبالتللي يكون المخاك ثقيييل لدرجة يعمل‬، ‫ لكن مع هذا المرض مافي ماء يدخل ويخليه لزج يحصل فيه بلوك‬، ‫ يدخل للمخاط ويخليه يتحرك بسهولة‬h2o ‫نسبة‬
‫ طيب‬، ‫ يعمل انسدادات‬thick ‫و ياثر على مناطق كثيرة بالجسم فلما يسير‬thick and sticky mucu ‫ ويعمل انسداد بالرئة‬، ‫انسدادات مايخلي االنزيم تخرج‬
‫ ياثر كمان على‬، ‫ في الشعب الهوائية ف يحصل انسداد في الرئة بسبب السماكة الشديدة‬، ‫ بالطبيعي ؟ في اماكن كثيرة منها الرئة‬9‫ فين موجود‬mucus ‫هذا ال‬
‫ مايقدر يطلع‬bile ‫ بحيث‬liver ‫ كمان ياثر على‬, ) obstruction ‫ فاالنزيمات البنكرياس ما تقدر تطلع ويحصل في قنوات البنكرياس (انسداد‬، ‫البنكرياس‬
. ‫ )) وليس كرونك‬acute pancreatitis(( ‫فيعمل‬ pancrease and lung : ‫ لكن اكثر واشهر اشهر مكانين ياثر فيهم هم‬bile duct ‫فيحصل انسداد فيه‬
Cystic fibrosis The body produces thick and sticky mucus that can clog the lungs and obstruct the pancreas. Cystic
fibrosis (CF) can be life-threatening, and people with the condition tend to have a shorter-than-normal life span. Sixty
years ago, many children with CF died before reaching elementary school age

1pancreatic pseudo cyst


pancreatic pseudocyst
Pancreatic pseudocysts are fluid collections around the pancreas. They arise due to sudden
or long-standing inflammation of the pancreas. While some will disappear when the
inflammation of the pancreas settles down., Serum amylase and lipase levels (limited
utility) – Often elevated.
Management
Most pseudocysts resolve without interference and only require supportive care. For some,
drainage is indicated.

🌹 best method for pancreatic pseudocyst drain ?


A. Laparoscopic B. Endoscopic C. Open D. US guided
🌹 Treatment of *Infected* pancreatic pseudo cyst: Percutaneous drainage ✅ ✔
pseudocysts, with final success rates of >80%. Or can treat by Percutaneous drainage)
، drain ‫ ويعملوا لها‬cyst ‫ يدخلوها من برة البطن من الجلد اتجاه المعدة حتى يصلوا لل‬tube ‫ معناها يجيبوا تيوب‬Percutaneous drainage ‫لما يقول‬
‫ غير كذا لو ماهي انفكتد نستعمل طريقة ثانية انهم‬infected cyst ‫ وتستعمل فقط مع حاالت ال‬، ‫ مثل االبرة الطويلة‬، ‫ من الجلد‬percutneos ‫يعني من اسمها‬
drain ‫ ويعملوا لها‬cyst ‫ حتى يصلوا لل‬endoscopic ‫يدخلوا بالمنظار‬
🌹 Patient recovered from acute pancreatitis episode , presents 6weeks after with vomiting and epigastric
fullness CT done:showed cystic collection behind the pancreas what is the most likely dx:(no fever)
A-Pseudo cyst ✅ ✔ B-Pancreatic abscess C-Pancreatic necrosis
. . .. . . . .. .
🌹 pancreatic pseudocyst, one is 18 cm x 24 cm , how to manage ?
A- per cutaneous aspiration B- endoscopic drainage ✅ C- surgical drainage
🌹 -one is a different scenario, duration of cyst is 6 weeks after acute pancreatitis, how to
manage: A- per cutaneous aspiration B- endoscopic drainage C- surgical drainage
* Percutaneous catheter drainage for (( *infective* pseudocysts )) .
* Endoscopic drainage = ‫ ما عدا انفيكتف‬، ‫ مع كل المرضى‬9‫ هذا عادي نستعملها‬.
.......
Pancreatic pseudocyst ttt:
- less than 6 cm and 6 week > observation
- more than 6 cm and 6 week > drainage
🌷 Pancreatic pseudocyst 5 week and 18 cm ? A. Medical B. Surgical ✅

🌷-Pancreatic pseudocyst if there is sign of infection as fever and leukocytosis do


Percutaneous drainage 🌷 -if sterile do Endoscooic drainage🌷
.... .... ......

🌹 Alcoholic pt has severe abd pain radiate to back ?


A- stomach cancer B_ alcoholic pancreatitis
....
🌹 Gray turner sign= its sign of Necrotizin pancreatitis = causes of =abdominal hemorrhage
(bleeding) 🌹
Causes of Grey Turner's Sign
Grey Turner's sign may be come after Cullen's sign. Both signs may be indicative of
pancreatic necrosis with retroperitoneal or intra-abdominal bleeding.
The underlying cause of Grey Turner's sign is generally thought to be abdominal
hemorrhage (bleeding), the main source of which is commonly accepted as the pancreas
............. ........
🌷 Poor prognosis of pancreatitis:
A. Hematocrit. B. Lipase above something. C. Amylase above something
…………………….
🌹Epigastric pain radiate to the back x Ray normal, high amylase, ct showed pancreatic
swelling, what is this complication? pancreatic pseudocyst

🌹Evacuation of pancreatic pseudocyst by? percutaneous 🌹

🌹Old with painless jaundice . Us show dilated gall bladder,intra and extra hepatic bile
duct . Ask about dx :- A. Klutskin Tumor B.Pancreas Adenocarcinoma ✅. C. Mirrizi

🌹Pancreatitis? IVF 🌹

🌹Case of pancreatitis Admitted, given ivf and i think pain killer Next to add in
managment? A. Ppi B. Antibiotics C.Urgent surgical consultation✅

🍓13X15 pseudo cyst of the pancreas in the lesser sac Patient vitals showed high
temperature treatment: A. endoscopic. B. per cutaneous✅
🌹Pt. With pictures of pancreatitis ( epigastric pain + high amylase) hxof gall bladder stone
,Next important initial step ? A. Crystalloid fluid iv ✅. B. ERCP. C. US
🌹What is the best initial drainage of pancreatic pseudocyst?
A. Percutaneous B. Open surgery C. Endoscopic
🌹 Pancreatic pseudocyst classical presentation. No signs of infection, there was 15×16 cm
fluid collection in the lesser sac. What is the management?
A. Imaging guided aspiration B. Surgical drainage C. Cyst excision D. Endoscopic fluid
aspiration.
🌹 Old patient with painless enlarged gallbladder ? . pancreatic Ca

🌹 A 50 year old man presents with progressive jaundice, dark urine, and right upper
quadrant pain and distention. On physical examination he has a palpable gall bladder.
Imaging shows an enlarged gall bladder and dilated Intrahepatic duct. Amylase = 481.
diagnosis is A. klatskin tumorn B. Pancreatic cancer C. Cholecystitis D. Mirrizi’s syndrome

🌹Asking about complication of pancreatitis after conservative treatment was done: :


Pancreatic pseudocyst

🌹pt had pancreatitis and gall stone , had been managed with fluid and stable what to do :
A. Cholecystectomy at the same admission B. Cholecystectomy after 4-6 weeks
N.B: Acute biliary pancreatitis: First resuscitation specially IVF.
Then : ERCP (stone extraction) followed by lap chole. Abx not indicated unless severe
necrotizing pancreatitis. If it was mild, moderate: lap chole in same admission.
Severe: lap chole in 4-6 weeks.
....
🌹Patient has pancreatitis with necrotic tissue, what is the most important step in
management? A. Antibiotics B. IV fluid C. percutaneous drainage

🌷65 y old patient presented with painless jaundice and wt loss = carcinoma head of
pancreas .

🌹Carpal tunnel syndrome


🌹Regional complex pain syndrome after carpal tunnel surgery release, complain of wrist
and hand pain , been treated with ortho, and pain clinic,, What is the appropriate mx?
A. Opiate. B. Triptan. C. physiotherapy
......
🌹Carpal Pain directly at the incision is typically only present for days or
weeks after the surgery. Protecting the incision can help alleviate pain, and it's important
to avoid lifting or gripping for several weeks after carpal tunnel surgery. ... Treatments for
pillar pain may include rest, massage, and hand therapy
🌹Carpal tunnel syndrome still in pain after surgery?
A- Physiotherapy✅. B- Analgesia. C- Open again
‫ = هو انه االلم يستمر بعد العملية وممكن كمان اشد من االول‬pillar pain carbal tunnel ......
🌹Patient after carpal tunnel realese complaining of chronic pain what’s ur
management ? A.opioids B. physio ✅✅ N.B if the pain was immediate post op most
likely incision site so opioids But if pillar pain > physiotherapy

🌹Patient has nerve radial injury , where level of injury ? spiral groove humerus 🌹
...............‫ = راديال ريدي = هل انت ريدي مستعد للهجووم ؟‬radial = ‫ = هيوميرال = هجوووم‬humeral = ‫ربط‬

Patient had loss of sensation on the snuff box and dorsum of the medial hand, hehad wrist drop.
At which level is the radial nerve injured? A. Axilla. B. Humerus groove✅ C. Olecranon D. Carpal
tunnel. Snuff box> radial> spiral groove of humerus
🍏🍏🍏🍏. humeral = humen = ‫ربط = سباير = سمعتي = راديل = الراديوا عن اخبار البشر والناس‬

🍓Paraesthesia and numbness with wrist drop (shows radial nerve injury) at
which Part A. Groove of the humerus ✅. B. Carpal tunnel. C. Olecranon
Wrist = Radial.
🍓Typist c/o tingling and pis in left thumb, index and middle & problem in dorsiflexion and
fingers extending which nerve affected ?
A. Median. B. Radial ‫يسال عن العصب‬

Patient with pain when writing at keyboard, some test showed hypoperfusion
to superficial palmar arch, what’s the artery affected?‫يسال عن الشريان‬
A. ulnar. B. Radial. C. Ant interosseous. D. Post interosseous
🌹Thenar muscle atrophy ? Median nerve🌹

🌹Female complaining of tingling sensation in her ring finger increase when she raise
her hand , in the examination positive arm elevation test = A) carpel tunnel syndrome ✅

🍒 Pt do surgery then Loss of sensation in ear pinna and upper neck what nerve is
injured ? Great auricular nerve
):‫ عشان المجوهرات‬pinna ‫ جزء الكبير والمهم في االذن هو‬great ‫ و‬، ‫ معناها اذن‬aurical = ‫ربط‬

🍒 Numbness on thumb and index finger what is the nerve affected? Radial nerve

🌹Male with left little finger and ring numbness or pain , rais stress test increases the
symptoms, what’s dx: A. carpal tunnel sy B. thoracic plexus

🌹Athlete with pain during standing PE: tenderness in medline planter sarface
A. plantar fasciitis. ✅ B. Tarsal tunnel syndrome

🌹Obese patient presented with little finger tingling and +ve raise stress test. the Dx?
A. chronic shoulder subluxation B. Ulnar artery thrombophlebitis
C. Tunnel syndrome D. Thoracic outlet syndrome

1Gallbladder
● Most common cause of biliary colic
A. cholydocolithaiasisis B. Gallstone

An elderly with IHD day 2 post cholecystectomy presented with sudden chest
pain SOB and vitally hypotension and tachycardia whats best ?
A. . CXR. B. ECG. C. CT angio D. LL duplex US

Cause of biliary colic?


A. Choledocholithiasis B. Gallbladder stone C. Gallbladder sludge

● pt with typical cholecystitis Mx? A. immediate lap chole B. wait

🍓what’s the common cause of cholesterol gallbladder stone : A. obesity B. rapid weight loss✅✅

🍒 Colicy abd pain with dilate CBD and intra hepatic duct causes of pain?Construcion of Sphincter of Oddi

Abdominal pain and US show stones in gall bladder with normal wall what is the
management ? a) Lap chole b) Ursodeoxycholic ✅
🌷Ursodeoxycholic Acid, is a naturally occurring bile acid and is used to dissolve gallstones
that are rich in cholesterol. It is also used to improve the flow of bile in primary biliary
cirrhosis.
🌹Old man came with jaundice and dark urine , palpable gallbladder , lab shows direct
belirubin =A) klatskin tumor. B) Perihailar tumor. C) CBD stone

🌷30 y/o male patient presented with abdominal pain related to meals, radiated to the
back, labs resulted high amylase, U/S showed bile sludge with no gallbladder stones were
notified, no CBD dilatation, what’s your management?
A.Labroscopic cholycystectomy . ✅✅ B. Endoscopic U/S C. Endoscopic sphenoidotomy
D. I don’t remember but it wasn’t MRCP or conservative management, so the question
directed to the definitive management to prevent future pancreatitis

🌷Patient during complicated lap chole the surgeon accidentally transected the CBD
(common bile duct( above the level of cystic duct, what’s your management?
A. hepaticoduodenostomy. B. hepaticojejunostomy ✅ C. choledocho- duodenostomy.

🌷Lowest risk for cholesterol gallstones


○ Nulliparity ✅ ‫ ○ المرءة التي لم تلد‬Sudden weight loss ○ Obesity ○Hypertriglyceridemia

🌷pt. With RUQ pain, US showed fluid around gallbladder, management:


A- US guided aspiration ✅ B- lap choly

🌹Recurrent RUQ pain aggregation with morphine, temperature 36.5 ?


a) Biliary colic ✅ ‫ نستخدم بديل له ميبرادين‬، ‫دواء المورفين يزود جدا االلم وال يخففه‬
b) Cholecytities ‫ يعمل حرارة‬c) Pancreatitis ‫مكان االلم يكون ب قمة المعدة ويمتد للظهر‬
** all of them can have pain aggravated by morphine due to constriction of sphincter of
oddi analgesia of choice is meperidine
🌹Pt came to ER with ruq pain and jaundice, with fever i think, 2 h later patient improved, us
showed multiple gallstones within normal walled gallbladder, management: A. lap choly B.follow
up C.ursodeoxycholic
....
🌹Pt post PTC has fever, on PR exam ant. Boggy mx ? A. Percut. Drainage B. IV Abx (if small)
...
🌹 RUQ pain on US Dilated common bile duct with intrahepatic biliary dilatation ,
what is the cause ? A. cholelithiasis(gallstone) B. Cholangitis C. Cholecystitis
‫ الن قالك كلمة‬dialantion ‫ معناها فيه‬stone
🌹 Patient with jaundice, abd pain and on US u see stones and dilated CBD vitals show fever
only asking about diagnosis...
a. ascending cholangitis✅ b. choledocholithiasis c. cholecystitis
Cholangitis is an inflammation of the bile duct system ( dilated CBD so inflammantion on
bile duct).

🌹Pt with DM and HTN and SCA , type of gallbladder stones will form?
A. Mixed B. Cholesterol stone C. Pigmented >> IF SICKLER
➔ SCD > pigmented
➔ sickle cell trait = Mixed
‫ر = سيكيل = خاليا حمراء = حمراء = صبغة‬.
🌹Diabetic obese elderly female with sickle cell trait has Lithiasis what kind of stone she
has:? A. Mixed B. Fat C. pigmented

.....
Least risk factors for gallbladder stone A.obes B-Hypetrglesride C.nullparty

Patient with history of lap chole few weeks ago presenting with right upper abdominal pain ,
respiratory symptoms , US done and show pus collection at site of gallbladder 12*6 cm What is the
most appropriate in management?
A. antibiotic B. precautions guide drainage✅ C. laparotomy D. laparoscopy
.....
🌹Patient post bariatric surgery complains of on and off fever for one week On
examination chest, abdomen and wound were normal How are you going tomanage:
A. Reassure B. CT abdomen C. Chest x-ray
N.B : Spiking fever > deep collections

🍓female pt present with right upper Q pain , febrile Lab test show: increased WBCs increase ALP
increase Direct bilirubin US Show : fluid around gallbladder , multiple stone What the most
appropriate management ?
A. Lap chole B. laparotomy C. precautions guide dringe D. antibiotic✅

🌹 Elderly pt admitted to Icu with acute MI , and developed pneumonia on tazocin , he


also have RUQ pain and tenderness. Management ?
A. ercp drainage B. emergency cholecystectomy
C. convert from tazocin to meropenem D. us guided cholecystostomy drainage

🍓 post cholecystectomy came with perihepatic collection, what will you do?
A- Laproscopic driange B- Percutanous driange✅✅ Dr.abdullah’s answer:
Any Post-Op Abdominal collection should be drained Percutaneously.

🍓 patient with diverticultits did sigemoidectomy , 5 or 7 days after had fever on per rectal
exam there is bogginess anteriorly what to do : A-drainage ✅ b-reassure

🌹 Patient with RLQ pain ct showed collection 13*10 =


A. percutaneous drain B. open appendectomy C. lap appendectomy

🌹 Post colectomy paracolic collection:management?


-Ultrasound guided drainage ✅ -open drainage
.....
🌹Patient with vomiting, constipation and fever Known case of crohns CT showed large collection
and fistula *initial* management? A. Percutaneous drainage ✅. C.Lap drainage and fistulectomy
It’s not recommended to do both procedure at the same time, you should drain first and later
fistulotomy
🌹UC pt with y shape something with very enlarged transverse colon and no haustrea what is Rx
A.proctocolectomy with ileal pouch B. pan colectomy with ileostomy✅
Answer is:IV Steroids if failed : total proctocolectomy or subtotal colectomy. Dx: toxic megacolon
. If no steroid in choice then choose B
🌹Pt with jaundice ; us showed 1.1 cm common duct dailated and dilated intra and extra hepatic ;
and very distended gallbladder: A. panceriatic cancer B. Mrizzi syndrome C. Klitskin D. Cbd stone
Kelastin or perhailar cancer >shrunken GB Pncreatic cancer >distended

🌹pancreatitis case managed conservatively at the day 3 the pt improving clinically and
laboratory . Every thing normal except Amylase 250 . US showed>> Gallstone at the gall bladder
with dilated extra hepatic ducts ( nothing was mentioned about CBD). What is the appropriate
management:
A. lap chole before discharge B. elective lap chole C. ERCP
….

1ERCP
🌹 Post cholecystectomy ( collection was found behind gallbladder) what to do:
_Percutaneous drain ✅ _Open drainage Laparoscopic drain

🌹 -Obstrictive jaundice pic and cholangitis on IV abx ,US (dilated ducts, gallbladder
has stones) what else in the mx? -ERCP✅ -cholecystectomy

🌹Typical pic of acute cholangitis, next investigation? A. US B. MRCP C. ERCP


Cholangitis is an inflammation of the bile duct system it happened bez there is stone on bile duct
and any stone on bile duct best manag with ERCP
‫ مثل ما يقولكم‬dialated ‫ سواء كانت‬bile duct ‫ او جابوا سيرة‬bile duct ‫ في‬stone Q‫أي مرة نشوف‬
ERCP ‫ دائما‬Q‫ اختاروا‬common bile duct dialated
🌹Pt with fever, jaundice and RUQ pain,initial management= A. abd US B. ERCP C. MRCP
cholangitis ‫هنا تشخيص الحالة‬

🌹patient with pneumonia take tazocin develop RUQ pain, ultrasound show dilated
common bile duct accumulation of fluid thick wall of gallbladder how to manage?
ERCP
‫ راح‬، ‫ فحتى لو عملنا استئصال لها‬gallbladder ‫ وهذه الحصوة خرجت برة‬، ‫ لذلك تراكم السائل وحصل تمدد في القناة‬bile ‫معناها فيه حصوة سدت مجرى خروج‬
.ERCP ‫ يعني منظار يفحص ويعالج بنفس الوقت وهو‬، ‫ في مثل هذه الحاالت الزم ادخل بمنظار واشوف مكان الحصوة واشيلها بالمرة‬، ‫الحصوة تظل موجودة‬
🌹Pt with history of Cholelithiasis, today present with abdominal pain after fatty
meal, on US : multiple gallbladder stones, thick wall , CBD 12 mm , what’s next:
A. ERCP✅ B. MRCP C. lap cholecystectomy D. open cholecystectomy

🌹Pt with calculous cholecystitis come with acute attack, When to do operation to
Her = A. Lapcholy as soon as possible ✅ B. Wait 2 or 3months

🌹 jaundice with us show stone in gall bladder and in common bile duct , tx.
a. Laparoscopic cholecystectomy b. ERCP✅
. ERCP ‫ هذا الزم‬commin bile duc t ‫ في‬stone ‫اذا قالك‬
🌹 patient presented with signs and symptoms of cholecystitis, days after hospitalization
the patient recovered clinically and her labs returned back to normal except for AMYLASE,
what is your management: ‫ بسبب حصوة‬9‫معناها حصل التهب في البنكرياس‬
A. ERCP ✅. B. Interval cholecystectomy 6-8 weeks
cholecystectomy is recommended for patients with gallstone-induced pancreatitis. ERCP
with endoscopic sphincterotomy (ES) within 24 to 48 hours is also suggested for the
treatment of acute gallstone pancreatitis.
......
🍓Case of post cholecystectomy and you found another stone in CBD how to ttt: ERCP✅

🍓 patient present with recurrent right upper pain , jaundice Vital sign :36.7tempratue and
stableLab test :Normal WBC increased direct bilirubin increased ALP US Show:Thick wall
gallbladder , multiple stone , and CBD more than 12 cm What is the next step:. ERCP✅

🍓 Post lap chole presented after few days with abd distension and ascites and abd pain
what will u do? a. open b. lap c. tapping D . ERCP

🌹 After long ERCP, patient hypotensive with pain ..Most common site injured:
A. Esophagus B. Duodenal C. Gastric
. Duodenum ‫ اثناء العملية هو‬injury ‫ اكثر مكان يحصل له ضرر‬ERCP ‫بعد عملية المنظار‬
N.B: Injury > duodenal Complication >pancreatitis،

🍓 Most common organ damage in ERCP: A. Duodenum B. Esophagus

🍓 Patient post cholecystectomy day 8or 9 develops right mouth corner pain and
fever 38.5 what is your mx: Paracetamol. Antibiotic✅✅. CT

🍓 Pt do cholecystectomy 8 days ago now he have pain in angel of the mouth for one day?
A.ABx B. ct abd C. xray D.paracetamol

🌹 Patient came with RUQ pain , ultrasound findings : thickining gallbladder wall ,
pericystic fluid and stones . What's best intervention :
A) percutaneous drainage B) Laparoscopic cholecystectomy ✅

🌹 6 weeks Post MI patient, planned to do cholecystectomy:


A.Do Lap cholecystectomy now ✅🌹 B.Do Open cholecystectomy now
C.Do lap cholecystectomy after 6 months D.Do open cholecystectomy after 6 months
.........
🌹Gallbladder polyp *less than 10cm* :-
*if elderly or with stone* > cholecystectomy
*if less than 50y >* follow up with us every 6m

🌹 Pt did abdominal US as htn screen and the results show gallstone polyp what to do?
A.Follow up in 6 month ✅ B.Surgical consult
🌹 Patient with biliary polyp, 0.6cm. Management:
A. reassure. B. follow up C. cholecystectomy
N.B: Gallbladder polyp less than 10mm :-
if elderly or with stone > cholecystectomy
if less than 50y > follow up with us every 6m

....
🌹Cholecystectomy 6 years back, presented with vomiting and increased bowel sound
movement (exaggerated) A. Adhesion ✅✅

🌹Kc of small gallstones presented with jaundice, Labs show high alk phos, high cong
bilirubin, Next? A. Ercp B. Abd us ✅✅
N.BYou have to start by US first If revealed dilated CBD go for ERCP

🍓Pregnant 10 weeks, complaining of attacks of biliary colic in the past 5 weeks TTT:
A. laparoscopic cholecystectomy now B. laparoscopic cholecystectomy after delivery
C. laparoscopic cholecystectomy in second trimester✅
D. laparoscopic cholecystectomy in third trimester

🍓Acute cholecystitis presents with right upper quadrant pain, fever, and leukocytosis.
Patients with acute cholecystitis should be treated with
laparoscopic cholecystectomy within 72 hours

🍓30y female patient, came to ER with abdominal pain... She has hx of elective
cholecystectomy due to Gallstones. Now she has high amylase, 9mm CBD and
jaundiced diagnosis= missed stone in CBD

🌹Lady post cholecystectomy , due to cholothiasis. week later presented with jaundice ,
pain , U/S: showed dilated CBD. Dx: = Retained stones

🍓70 years old patient with a history of Myocardial infarct that was 6 months ago, on this
presentation he has cholecystitis and requires cholecystectomy. when will
you operate? A. now✅✅ B. After 6 months

🍓cholecystectomy the surgeon accidentally cut off the Common bile duct at a level that is
just above the cystic duct, how to repair?
A. hepaticojejunostomy✅ B. Choledocojeujunostomy

🍓 laparoscopic cholecystectomy come with discharge from middle management ‫يعني‬


‫الججرح ينزل منه صديد‬
A.daily dressing B- wound inspect. C- exploration

🍓Middle discharge after cholecystectomy= A. Abs B. Exploration✅


No dressing or inspiction

🌹Patient on 3rd day post cholecystectomy develops fever, has no abdominal pain.What is
the likely cause of fever: A.UTI B.Wound infection C.Chest infection
3 day = UTI = 9‫ثالث حروف‬
...
🌹Patient post cholecystectomy, presented early (don't mention which day) with fever and
lung consolidation (atelectasis), and US showed 10*12 collection of fluid in lesser sac ,Mx?
A. Physiotherapy B. abx C. Precut drainage
......
🌹Pt have fever and jaundice and RUP pain dx? Cholingitis
.....
🌹Charcot triad = fever , jaundice, RUP pain = Cholingitis ) not cholecystitis)
...
🌹Post cholecystectomy recently (not mentioned the day) with fever spiking,
decrease air entry in right lower lung, dullness in percussion and tenderness in
right hypochondrium US showed collection 10x15cm in gall bladder fossa, ttt:
A. physiotherapy B. iv abx C. drainage
.....
🌹klatskin tumor= is cholangiocarcinoma located at bifurcation of common hepatic duct =
CA 19-9.

🌹Patient after prolonged difficult ercp, developed neck, chest and abdomen surgical
emphysema. What was injured? A. Esophagus B. trachea C. Duodenum D. bile duct

🌹RUQ pain and dilated common bile duct, what is the diagnosis? Choledocholithiasis ✅
Common bile duct stone, also known as choledocholithiasis, is the presence of gallstones in
the common bile duct (CBD) (thus choledocho- + lithiasis)
....... . . .. ..
🌹Choliduocholithisis managment?- ERCP ✅

1GIT
🌹Pt alcoholic ,smoker came with sever pain generalize in the abdomen In
examination was tenderness and guarding= . Duodenal perforation
🌹Post percutaneous transhepatic cholangiography developed upper gi bleeding, invx?
A. Endoscopy B. CTangio C. US

🌹Elderly smoker with dysphagia , showed high grade dysplasia , management?


A. Add ranitidine B. Ask him to stop smoking C. CBT D_ endoscopic mucosal resection

🌹best diagnostic test to detect ischemia :


A. stress Echo ( Approved)✅ B. CK

🌹What is sign in radiology for duodenal ulcer ==clover leaf


‫ = ربط‬doudenum = ‫ = ودود‬love = clove
🌹 Old man suddenly had vomited two episodes of bloody vomit, Physical Exam shows no
Tenderness over his abdomen or sternal area, he is stable, what is your diagnosis:Mallory
weiss tear

🌹 24 hours vomiting after food poisoning develop hematemesis on endoscopy blood


streak on stomach Dx? Mallory weiss

🌹 adult patient ( i forgot the age may be in 30 ) have repetitive vomiting come
with mild bleeding ( mallory weiss syndrome )what is the appropriate
management?
A. conservative management B. laparotomy exploratory C. laparotomy with resection

🍓C/I to liver transplantation= Cirrhosis with active alcohol✅

🍓patient with hepatitis B cirrhosis with ascites and have 6 cm lesion found in the with
high vascularity = treatment? A. anti viral. B. resection. C. chemo embolization✅✅
Transcatheter arterial chemoembolization is a minimally invasive procedure performed in
interventional radiology to restrict a tumor's blood supply. 

🌹 pt with Chronic hepatitis b then , then discover 70% of the liver was multiple lesions,
next: Colonoscopy.
...

🍓Male Pt, smoker, have a history of appendectomy, his brother have crohn’s ,
what’s the risk factor to develop crohn’s : A. male B. smoking C. family history✅ D. history
of appendectomy

🌹Case of GERD with High grade dysplasia what you will do


A.Endoscopy 6 m B. Laparoscopic esophagectomy ✅✅

🍒 pt underwent pneumodilation for achalasia cardi , present after while with signs of
esophageal perforation . ask about mx ?
A.stent B.esophagectomy Answer is : stable > stent , unstable > surgery
🌹Healthy female with dysphagia diagnosed as achalasia, best treatment
A- pneumatic dilatation B- botulinum injection C- fundoplication

🌹Valvulas location: A.Cecum B.rectum C. sigmoid ✅

🍓Y shaped colon, Sigmoid volvulus what to do? Colonoscopy or sigmoidoscopy🍓

🍓Old patient history of vomiting with palpable mass imaging finding Y shape next app
action? A. Fleet enema B. Ct C. Sigmoidoscopy D. Colonoscopy

🍓Old male with generalized abdominal pain, ascites, constipation X-ray >> massively
enlarge loop up to right part ? A. Sigmoid volvulus ✅ B. Acute diverticulitis

🍓52-Patient came with abdominal diatention x ray showed y shaped colon what’s your
management? A. Fleet enema B. Sigmoidectomy with end colostomy C.
colonoscopy✅✅(obstruction)

🌹 Pt with UC the most associated risk of Cancer is?


A. primary sclerosing cholangitis✅ B. More than 3 years UC illness duration
C. UC confined to part of colon. D. Mild inflammation

🍒 Case of Sclerosing cholangitis, what you do for the patient? A- CT B- Colonoscopy ✅


‫ لما‬، ‫ عشان كذا‬ulceraitive colitis ‫ يكون عندهم‬Sclerosing cholangitis ‫ ؟ الن كثير من الحاالت الي مصابين ب‬Colonoscopy ‫تعرفوا ليه اخترنا‬
‫ الن هذا المرض في الغالب‬، ‫ عن طريق عمل منظار القولون‬ulceraitive colitis ‫ الزم نستبعد وجود‬Sclerosing cholangitis ‫نشوف مريض مصاب ب‬
.‫ هللا يشفيهم‬، ‫ الزم يكون معاه مصيبة ثانية‬، ‫مايجي لحاله‬
🍒 Abdominal distended, vomiting, picture of obstruction. Dx? volvulus

🍒 What is a volvulus? A volvulus causes bowel obstructions, which may cut off the blood
supply to areas of the bowels. A volvulus occurs when part of the colon or intestine twists.
The twisting causes bowel obstructions that may cut off the blood supply to areas of the
bowels

🍒 Management of unstable patient with sigmoid volvulus?


A-Sigmoidectomy B-the other opsions were all conservative management.

🌹Most common site of sigmoid volvulus?


➔ Elderly > sigmoid ➔ Children > Ceacum

🌹Patient with basal ganglia disorder and absent gag reflex. How to provide nutrition?
A-Gastrostomy B-Jejunostomy✅ . C-NGT!@

🌹Pudendal nerve block. Which organs are still sensitive?


A-Vulva. B-Perineal body. C-Urogenital diaphragm D-Rectum✅✅
🌹Colorectal surgeon performing a low anterior resection for CRC and the pelvis won’t
stop bleeding, so he consults a vascular surgeon and he does:
A-Heavy packing of pelvis B-Arteriography intra operatively C-Infraceliac clamp !@

🍓bleeding aortic in OR and you did packing but he is still bleeding. Next?
A. clamping infrarenal B. clamping supraceliac C. cable CTA!@

🌹Patient had RTA with seat belt sign x ray of the spine showed chance fracture what most
probably you will find in = Duodenal perforation
‫ او تظهر على بطن المريض الي عمل حادثة العالمة هذه معناها مثل (ربطة حزام االمان) بالضبط‬x.ray ‫ يعني عالمة تظهر في‬seat belt ‫اول شيء عالمة‬
‫ لكن مافي مصدر يقولك اذا شوفت عالمة‬bowel ‫ وخاصة‬organic damage ‫ هذه العالمة اذا شوفتوها في المريض اعرفوا انه حصل له‬، ‫وشوفوا الصورة‬
‫ لكن المصادر تتكلم بشكل عام ان هذه العالمة اذا شوفتوها اعرفوا انه فيه‬، ‫ تحديدا‬dudenal perforantion ‫ معناها فيه‬seat belt ‫حزام االمان الي هو‬
.abdumen ‫ االعضاء الي موجودة في‬pancrias , dudenum ‫ سواء‬organic damage
The seatbelt sign is both a clinical and radiological sign. It is simply the presence of bruising/abrasions in the distribution
of a seatbelt (i.e. horizontal and/or diagonal) across the abdomen, chest and sometimes neck.  A positive seatbelt sign, in
combination with abdominal pain or tenderness, results in a higher likelihood of intra-abdominal injuries, in
particular bowel/mesenteric injury
Seat belt signs remain an important physical finding following motor vehicle crashes. Should be aware of seat belt sign and
there should a higher index of suspicion to rule out underlying organ injuries.

🌹Rta patient his speed was 130 and he was on seat belt came with stable status no hypotension or change in
consciousness what you will do = A. CT B. us of abdomen. C. laparotomy

🌹Pt elderly known to have IHD come with sever central abd.pain Amylase: slightly
elevated = Mesentric emboli of occlusion

🌹Patient known to have ischemic heart disease complain of abdominal pain (the only complain)
with lab showing amylase of 600 and x-ray showing signs of obstruction ?
A.Acute appendicitis B. intestinal obstruction C. mesentric ischemia ✅✅

55 years old male presented with sudden abdominal pain radiated to the back the patient has
history of cardiomyopathy, the patient mentioned that he passed small amount of loose stool,
during the examination there is exaggerated bowel sound Labs: Amylase = mildly elevated above
normal What is the Diagnosis?
AAcute pancreatitis B. Mesenteric vascular ischemia✅ C.Ruptured aneurysm

🌹seat belt sign = bowel/mesenteric injury like Duodenal perforation.


….

🌹pt complain of malena and epigastric pain diagnosed as peptic ulcer and received ppi but
not improved endoscopy done showed multiple ulcers in the antrum what is the most
accurate management a.antrectomy✅✅. b.partial gastrectomy. c.total gastrectomy
##multiple ulcers in the antrum = antrectomy ##
....
● Multiple antrum ulcers , by Bx >H pylori what is the mx ?
A. start Abx B. Chemo C. Radiation

🌹 GIST , 5 cm in posterior wall, mx: A-Total gastrectomy. B. partial gastrectomy. C. WLE

 Gastrointestinal stromal tumors  are soft-tissue sarcomas that can be located in any part
of the digestive system. Their most common sites are the stomach and small intestine.

🌹gastrointestinal stromal tumors with metastasis = treat by = tyrosine kinase inhibitors


(imatinib)

🌹 GIST (Giant gastrointestinal stromal tumor ) in body stomach 3x4 cm no lymph node
metastasis , ttt? A- gastrectomy B- wedge wide local exsion ✅✅
🌹Giant gastrointestinal stromal tumor = wedge wide local exsion 🍬== Dont choice (gastrectomy) 🌹

🌹pt with GIST (Gastrointestinal stromal tumor less


than 2 cm ) , what is the most accurate
management:- a.total gastrectomy b.partial gastrectomy c.observation ✅. d.ppI
if 2 cm or less then observation
....
🌹Gastrointestinal stromal tumor 3x4 cm. No infiltr'tion, no lymph nodes. Wh'tʼs the
m'n'gement? - G'strectomy- _Wedge resection with clear margins✅ - Chemo

🌹stomach tumor with positive secretin stimulation test


a.VIPoma. b.glucagonoma c.gastrinoma✅✅
. ‫ الي يخرج من المعدة‬gastrin ‫من اسمها ورم في المعدة مع هرمون‬

🌹pt with LLQ pain, and hx of constipation, on exam, bulky mass in LLQ, with wild
discomfort, no worrying sings or old age, dx: A- constipation ✅ B- others on related

🌹50 yrs man diabetic, well controlled had colon cancer surgery ( coloectomy)
they kept him on insulin and dextrose, after surgery by 2 days he became irritable, in
shock, his electrolyte ( Na 129 ) ( K 3.2 ) urine and serum osmolality Normal, what's the Dx:
Fluid overload ✅ ✔

(( Hypervolemia is a condition in which there is too much fluid in the blood. It is also known
as fluid overload. , Hyponatremia is decrease in serum sodium concentration <
136/L caused by an *excess of water relative to solute* .))

🍒Long case IBD and do surgery not mention it and now have to take fat soluble vit which
part of intestine had been rescued = terminal iluem  Vitamins A, D, E, and K, fats, and
cholesterol are absorbed in the lower third the ileum. ) 🍒

🥦What vitamins are absorbed in the ileum?🥦


Protein is absorbed approximately midway through the ileum. Vitamins A, D, E, and K, fats,
and cholesterol are absorbed in the lower third the ileum. Vitamin B12 is absorbed just
before the small intestine joins the large intestine. Bile salts are reabsorbed in the
distal ileum and the ascending colo

🍉If the terminal ileum is missing a deficiency of vitamins A, D, E and B12 may occur. 🍉
Vitamins A, D, E, and K = Fat soulebal vit.
🌹Patient while doing laparoscopy for esophageal perforation with Bp 80/50,,RR25 pulse
120 which type of shock?
A.Cardiogenic B.Hypovolemic ‫ نزيف بسبب الثقب‬C.Septic D.Anaphylactic
🌹patient with GERD underwent endoscopy show multiple antral ulcer Ttt?
A.Total gastrectomy B. Partial distal gastrectomy ✅ C. Pylorectomy ❌
🌹 Succession splash -‫ نسمعه في البطن عن الفحص‬9‫ هذا اسم صوت بالسيتيسكوب‬symptoms of
obstruction patient have? A.met acidosis B.Met alkalosis ✅ C. Compensated met
alkalosis D. Compensated met acidosis =
........... alk = ‫ = صوت = قلق‬succ =‫ربط‬
….
🌹Symptoms of obstruction and X-ray done showed Dilated loops towards RUQ =
A.loop obstruction B. Sigmoid megacolon ✅🌹
..
🌹Bowel obstruction symptoms. Imaging showed stricture. How to treat a stricture
A.Bowel resection ( 2 options of different parts of bowel) B. Stricturoplasty ✅🌹
….
🌹 A case of crohn's and abdominal obs due to stricture It was single and 1cm away from
ileocecal valve, Mx? A. Stricturictomy B. Observe C. Resection D- laparoscopy with surgical
resection!@

🌹 K/c of chrons with perianal mass , painful , there discharge when touch it , Mx?
A. Increase dose of infliximab and reduce B. fistulotomy. C-MRI pelvic
….
🌹Crohn's patients on steroids and infliximab came with perianal pain and discharge and
low grade fever since 3 weeks what will you do?
Antibiotic Pelves MRI ✅ c. Increase the dose of infliximab!@
Examination under anesthesia (EUA) is considered the gold standard to diagnose and
classify perianal fistulas in CD patients. An accurate diagnosis is also possible using imaging
modalities such as pelvic MRI and/or endoanal ultrasound (EUS), and, in some cases,
transcutaneous perineal ultrasound (TPUS). Any of these methods should be combined with
the endoscopic examination to assess the presence or absence of active inflammation in
the rectosigmoid colon, the presence of in
(Angiodysplasia is a small vascular malformation of the gut. It causes sever bleeding
therfore must be treat )

*Treatment for angiodysplasia*


- If the anemia is severe, blood transfusion is required before any other intervention
is considered.
- Endoscopic treatment is an initial possibility, where cautery or argon plasma
coagulation (APC) treatment is applied through the endoscope.
- Failing this, angiography and embolization
- Resection of the affected part of the bowel may be needed if the other modalities fail.

🌹Angiodysplasia in 60 year patient, how to manage? argon plasma coagulation (APC) and
bipolar electrocoagulation (BEC) = Both are safe and effective ✅ ✔

🌹Patient with rectal bleeding done technetium scan and Dx with angiodysplasia in left
colon TTT: -
A. Conservative B. laser ablation C. angioembolization > IF SEVER D. left hemicolectomy

🌹management of angiodysplasia= A) left hemicolectomy B (cautery embolization (if failed


choice A)
….
🌹Old p'tient >50 ye'rs old. Alternating diarrhea and constipation. Physic'l is normal.
Underwent sigmoidoscopy, revealed multiple lesions in the dist'l sigmoid. Biopsy =
'adenocarcinoma'. Next step in management? Sigmoidectomy Colonoscopy CT bdomen!@
........
🌹Case of perianal swelling , perrectal bleeding , investigated : it is 1 cm from anal
verge , biopsy: ((adenocarcinoma )) (rectal) Mx
- abdomen perineal resection ✅. -low anterior resection. -chemo/radio
….
🌹 20-year-old weight lifter developed abdominal hernia that is irreducible, tender. Imaging
shows air-fluid levels in small bowel and no free air in large bowel. Type?
○Irreducible. ○ Incarcerated. ○Strangulated
Incarcerated hernia and strangulated hernia both can cause SBO.
Incarcerated hernia presents an irreducible hernia with tenderness and abdominal
distension.
Strangulated hernia: ischemic segment will cause symptoms such as erythema,
leukocytosis and fever. (Toxic patient)
-- ----- ----- -----
🌹4th day post OP (appendicitis) presented with diffuse abd pain, distension , vomiting
and sluggish bowel sounds Abd xray: Multiple air/fluid level
-ileus ✅ ‫ يكون معاها صوت االمعاء كسول او خاملة‬-adhesions and SBO ‫يزيد صوت االمعاء‬
(( sluggish‫ كسول او خامل‬bowel sound == ileus))
((Incresse bowel sound ==adhesions and SBO ))
...... . .. . . .. . . .. .
🍓Most common cause of Small bowel obstruction > adhesions✅ Large bowel > neoplasm✅

...... . .. . . .. . . .. .
-Pt 6 yrs post abd operation has bloating and signs of obstruction cause?
a. adhesions✅ b. perforation

🌹case scenario of intussusception what is the gold standard investigation= barium enema
best intinal = us Gold stander =enema
‫ منطقي الن الصبغة توضح كانك تشوفها بعينك انه حاصل تداخل باالمعاء‬.
...... ....... ..... .
Patient with symptoms of intussusception what is true about it:
A/reduction using enamas only in case of peritonitis
B/Treated by imme`diate surgical intervention
C/Recurrence is high after surgical treatment✅✅
….
🌹-What type of cancer that should be screened for with out symptoms :
Colon ✅ Pancreas Skin
........

🌹 28 male healthy, hx 2 month of abd pain with 2 bloody stool, proctoscope done with
numerous polyps covered the linings and multiple biopsy taken , no details in hs of family
hx or sexual.. Dx:
A. familial polypoid✅ B. UC C. diverticulosis coli D. human papillomavirus polyp

🍓Multiple polyps in colon and duodenal And bloody diarrhea?


A, Familial Colon Cancer Syndromes✅. B. UC. C. Crhrons

🌹Long case with Thumbprint sign on abdominal x-ray ? Pancritits or Ischemic colitis 🌹

🌷Thumbprint sign in abdomen = Pseudomembranous colitis AND ischemic colitis🌷

🌹40 yo male K/C of hyperlipidemia present to ER with epigastric pain, and normal Amylase
and lipase, slightly elevated AST, ALT, abdominal xray show air in LT hypochondrium " and
give the Name of sign" Dx?A. Acute pancreatitis. B. Ischemic colitis

🌹Anterior duodenal perforation tratment = : Graham omental patch✅


Graham ‫) وبالتالي يحصل نزيف عالجها ب شيء يسمى ب‬perforation( ‫ وما تتعالج يتطور الى ثقب‬duodenal ‫لما يحصل قرحة في‬
duodenal: ‫ فلما نشوف مريض عنده‬، ‫ يغلق هذا الثقب‬omentum‫ يعني نخيط الثقب باالومينتم ب غرزة معينة بحيث ال‬omental patch
.perforation = treat by = Graham omental patch
Graham omental patch ‫ الي هو نخيط الثقب بال‬oment .
🌹Man with high grade dysplasia of the esophagus== Refer him for surgery

🌹A young healthy man medically free came complaining of indigestion problem


otherwise he is normal no vomiting no nausea no bloody diarrhea you gonna do ? A.
Routine abdominal x ray B. Urgent surgery C. Routine referral to gastro D.functional dyspepsia

1Truma
🌹Patient post RTA with warm peripherals Which type of shock?
A. Cardiogenic B. Septic(cold priphral) C. Neurogenic bez (warm peripheral)

🌹15year, fell and revived a supracondylar fracture above the elbow. Management?
If Brachial pulse present >> Reduction If Not present >> Surgical Exploration

🍓Patient in ER due to RTA , patient denial any history of loss of consciousness ، GCS 15 ,
suddenly patient deteriorated and loss of his conscious with dilated pupils hats is the most
likely diagnosis : A. AV malformation B. subdural hematoma C. Epidural hematom✅

🍓What kind of shoulder dislocation.. difficulty in addiction and internal rotation?


A. anterior B. posterior dislocation ✅✅

🌹abdominal trauma (by wooden stick) with painful RLQ pain and grey discharge, pain
increased by extending the leg. Next step? A.Culture B. Give antibiotics ✅ C.CT
abdomen

🌹Female had femoral fracture then after I think a week developed respiratory
symptomes ? Fat embolism syndrome ✅
‫ اي مرة تشوفوا كسر في‬Femoral ‫ في بالك هو‬9‫ شي يخطر‬9‫ بالذات وحصل بعظها ضيق بالتنفس اووول‬fat
embolism ‫ الن كسر عظم‬، ‫ اياكم تنسوها‬femoral ‫ هو اكثر شيس عنده خطورة في حصول‬fat
Fat =Femoral
.......
🌹5 days post orthopedic surgery had sudden dyspnea and confusion on
examination shows rash on neck and on cxr bilateral lower lobe infiltrates cause?
a. Fat embolism✅ b. PE c. pneumonia

🌹Colles fracture in pedia what is the most important mangement ? closed reduction
Colles = Close ‫من اسمها كوليس = كلوز‬
🌹 Picture of green stick and ask for treatment?
A. close reduction with cast

🌹 20 y MVA with normal vitals except pulse increased. Which type of shock
A.cardiogenic B.anaphylactic C. hypovolemic ✅ D.neurogenic
.‫للمعلومیه السوال كذا جا حرفیا وال في شي زیاده في الهستور مافي سوائل كثير فالنبض زاد حتى يحاول يعوض الجسم عن النقص الي سار‬
...........
🍒Patient 72 years old have DM,,,For 3 month cant be stand from the chair What can
happen to him ? A-Fall ✅. B-Rta. C-gunshot. D-Fire

🍒Most common site of aortic injury ? Proximal to left brachiocephalic

🌹Blunt truma in chest or thoracic what to injured?


a-Left brachiocephailc b-Aortic arch c-Brachiocephalic something D- left subclavian artery
….

🌹 Most common site of thoracic aortic blunt trauma :


A. Aortic arch B. aorto-ligament C. proximal to subclavian... artery
D. distal to left subclavian artery
=subclavian = sub = ‫ = صعب حادث باالورطى شريان مهم = صعب‬distal ‫ربط = حادث في االورطى = بعيد الشر عليكم = بعيد‬
. left = ‫هللا يبعدكم عنه = يبعدكم‬
.....
🌹 Most common injury in blunt chest trauma :
A. aortic arch B. proximal left proximal subclavian artery
C. Left distal medial subclavian artery

🌹after RTA pt hospital is away 40km what would you do??
A-call surgical oncall✅. B-chest tube

🌹A traumatic pt lost 25% of his blood..which of the following is the most suspected tobe
effected first A-Urine output. B-Pulse pressure ✅. C-Respiratory Rate. D-Glasco-coma scale
‫ يبدء‬، ‫ اول مايفقد االنسان كمية دم‬، ‫ لكن اول شيء يتاثر هو النبض‬، ‫ ظمكن واحد فقد دم كثييير و الوعي وكل شيء ممتاز‬، ‫يقولك اول شيء يتاثر‬
‫القلب يزداد سرعته حتى يحاول يعوض عن النقص‬

🌹male involved in RTA in ER he is fully conscious and no abnormality in vital signs admitted
for observation and a FAST scan done ,.what do you expect to find
a. pneumothorax b.major vessel bleeding in the thorax c. peritoneum free fluid✅✅

🌹Traum' patient.Intra abdominl hemorrhage and seen injury. Underwent laparotomy and
splenectomy. Which of the following will be low? - Vasopressin. - Insulin✅. - Gluc'gon.
Paintnt‫ من مضاعفاتها يستأصلوا حزء من البنكرياس فيحصل نقصان في االنسلين‬spleen ‫ فلما يستأصلوا‬head of pancreas ‫ الصق فيها‬spleen‫الن ال‬
MVA,hypotensive tachycardia tachypnia can shrug the shoulder but cant

🌹best method to clear cervical trauma in ICU patient after motor vehicle accident
A-clinical judgement B-ap lateral xray. C. CT✅

🌹after RTA pt hospital is away 40km what would you do? ABC .

🌹Case of pt with neck injury after RTA ?? O2 mask. ETT. Coricothyrdoctomy


Remember ABCDE of primary survey.. O2 mask > intubation > chest tube

🌹45 years old man had MVA presented with isolated head injury and coma for 5 days in
ICU , the best way of feeding?
A- Nasogastric tube feedings ✅. B- Gastrostomy feeding C- Central line feeding
D- Peripheral line feeding
Isolated head 🌹‫ اسابيع في غيبوبة او مايقدر ياكل‬٤ ‫ ؟؟ اذا كان اكثر من‬Gastrostomy feeding ‫متى نستعمل‬
injury and coma for 5 days = Nasogastric tube feeding

🌹Gastric cancer metastasizing to the liver= chemotherapy .🌹

● Pt e gastric cancer & gross ascites , next step in management :
A. Abdominal paracentesis B. Endoscopy .

🌹 A soldier walks for a long time with pain in his foot And have flat foot which the
tendon affected ? spring ligament

pt fallen down from 3 meter height he felt severe pain and swelling at the lower Rt leg xray
showed commonuted fracture of the lower tibia what is the most accurate management
a. closed reduction, cast and elevation
b.open reduction, internal fixation and elevation ✅✅✅ c.external fixation
.....
Mid shaft Femoral fracture 30 d angulation child 4 years old what management ==
A)_Traction. B)_ ORIF (( open reduction and internal fixation)) C- ORIF with IM nail.
......
🌹 6 years patient with mid shaft femoral fracture, 30% anterior angulation, what is the
management:
A.Reduction with bed rest B.Closed reduction with hip spica cast C. ORIF D.OR and
intra medullary nail !@

🌹Most important thing to do before reduction of fracture))) ==== Check vascular


status(pulse) ✅
‫ ال اهم شيء هو نتاكد من النبض الن لو مافي نبض الزم فورا عملية جراحية معناها فيه نزيف وكذا ا خطر نعمل رجوع للكسر‬، ‫اختيار ثاني يقول نعطي مسكن‬
....... ....... ....... ........
🌹Head trauma post MVC at speed 130 km/hr. Tight seat belt. Patient is stable and alert.
Most appropriate next step in management?
○Abdominal US ○ CTabdomen ✅. Diagnostic laparotomy ○ Exploratorylaparotomy

🌹A scenario of patient with vascular problems in lungs causing him pulmonary HTN, which
class/group of pulmonary HTN ? A. 1. B. 2. C. 3. D. 4. E. 5

🌹Elderly with MVA on exam "Battle sign" what's the diagnosis?


A. fracture of the mandible B. Le froke. C. Basal skull fracture‫من اسمها‬

🌹Case of head trauma, presented with ear bleeding, ruptured eardrum, what would be
the cause? A-Basal Skull fracture ✅ ✔. B-Subarachnoid hemorrhage

🌹 Ear secretion after trauma external ear was intact ? Basal skull fracture
. ‫ لذلك لما نفحص االذن من برة نالقيها سليمة‬، ‫ هذه ليس معناها انه الضربة جائت باالذن‬، ‫ يخرج من االذن دم وافرازات ثانية‬basal ‫لما يحصل كسر في منطقة‬

🌹RTA patient injures jugular foramen, what will happen?


A. paralysis of muscles of mastication B. something happening to the vocal cord ✅✅
. ‫ = فالزم نلبس جاكيت‬vocal cord ‫ = جايكيت = نلبسه بالبرد = في البرد يسير لنا التهاب بال‬jugular ‫ربط = جاكيوالر‬
......
🍓Battle' sign indicate which fracture: = posterior cranial fossa.
.... ......😋 . battle = 🍟🍟9‫ = عصير راني 🥤 مع بطاطس‬cranial = ranial = ‫ربط‬

🌹Pt had trauma cause base skull fracture .. jugular foramen:


A. ipsilateral vocal cords injury B. something with mastication muscles

🌹The most probable viral cause of repture of ear drum with abcess or pus ?
A.Rhinovirus B.Adenovirus C. RSV ✅

🍒Case of alcoholic patients with epigastric pain radiating to the back with x ray showing
air under diaphragm= A-acute pancreatitis. B- chronic pancreatitis.
C- duodenal perforation✅

🌹Pt have RTA can shrug shoulder but can’t move elbow and lower limbs ?
High spinal cord injury

🌹 Man who was in MVA, hypotensive with slow heart rate and can't move legs or
hands, what caused his shock? high spinal cord injury
🌹What is the cause of this hypotension? : A-Upper spinal ingury ✅✅ B-Abdominal
bleeding
upper spinal injury :
Patients with upper cervical spinal cord injuries the more dysfunction can occur.:
*Inability to breathe if injury on (C1-C4) ...
*Paralysis in arms, hands.
*Numbness, tingling, or loss of feeling below the level of the injury.
............
🌹Young male, post MVA Opens eyes spontaneously Responds to verbal
commandsShrugs shoulders Shallow breathing Left chest wall
contusion Cannot flex elbows or move lower limbs Respiratory rate: ?
BP: hypotensive Most likely diagnosis:
A. Cardiac tamponade B. Left tension pneumothorax C. High spinal injury
….
🌹Football player received a trauma to lateral side of his left knee, the patient now is
complaining of severe pain and swelling of the medial side of his left knee, positive valgus
and (-) anterior drawer and lachman, most likely diagnosis:
A.Medial meniscus tear B.Lateral meniscus tear C.Medial collateral ligament sprain
D.Lateral collateral ligament sprain !@

🌹Player professional football with knee Injury in lateral side , medial knee
swelling , lachman and MCmurray test ( negative) ?
A. Medial meniscus tear B. Medial collateral ligament sprain C. Medial collateral ligament

Pt came with instable knee. Ex femur come in front of tibia, which ligament
injured ? A. Pcl B. anterior cruciate ligament (ACL) 
1Thyroid
🍓After total thyroidectomy hypocalcemia what to do?!
A. Give PTH* B. give potassium C. measure magnesium D. give levothyroxine

🍓Patient admitted for thyroid surgery because of progress compression symptoms


Hemithyrodectomy done and biopsy showed 8mm papillary carcinoma next? radio
ablation B. no need for further intervention C. complete thyroidectomy. D-follow up
🍓After right hemithyroi dectomy On biopsy found follicular lesion 8cm from the
original. Mx? A. Radio iodine B. Followup C. Complete thyroidectomy!@
‫ او راديوا حتى يقتل لو فيه خاليا متبقية خبيثة ماشلناه بالجراحة‬9‫اي ورم شلناه وكان احتمال يكون خبيث الزم ياخذ عالج كيماوي‬

🍓After thyroidectomy history follicular ca small with lesion 8mm from the lesion
what will do :A. Total thyroidectomy B. Iodine scan C. Radio ablation * D. Flow up

🌹Pt for thyroidectomy and with mitral valve prolapse?


A. Prophylactic for endocarditis B. Cefazolin for surgical site infection
C. No need D. Prophylactic for endocarditis and surgical infection

🌹neck lump with hot nodule = hemithyroidectomy

🌹Young male with midline neck mass that ascends and descends with swallowing,
diagnosis:Thyroglossal cys✅ A. Cystic hygroma B. Midline dermoid cyst

🌹pt after thyroidectomy , develop hypocalcemia despite calcium replacement more than
one time , next : check serum magnesium level

🍓A patient post thyroidectomy can not make high pitch sounds. Damage to which
nerve is responsible? A. Inferior laryngeal nerve. B. Recurrent laryngeal nerve. C.
Glossopharyngeal nerve. D. Superior laryngeal nerve ‫ تقع فوق‬.‫الغدة‬

........
Pt came after thyroid surgery with dysphagia and horseness of voice. Which
nerve injured? A. Recurrent. B. Internal. C. External

🌹Patient with hyperthyroidism for 10 m and treated with anti hyperthyrdism


But his condition did not improve ultrasound done and show multi nodule and
diagnosed as Graves’ disease what is the best next step ? A.total thyroidectomy
B.subtotal thyroidectomy C. iodin therapy ✅ D.increase the dose of drug
Treatment Graves’ disease
First start with propylthiouracil (PTU) or methimazole (MMI)
ƒ PTU also inhibits peripheral deiodination of T4 to T3
• symptomatic treatment with `-blockers
• thyroid ablation with radioactive 131I if PTU or MMI trial does not produce disease
remission
• subtotal or total thyroidectomy (indicated rarely for large goitres, suspicious nodule for
CA, if patient is intolerant to thionamides and refusing RAI ablation)
🌹Neck mass biopsied showing (thyroid folicullar cells)? Ectopic thyroid ✅
 ‫ اطلب ساندويتش فول من برة‬follical = ‫فول‬
🌹thyroid nodule hard ultrasound report Next step? A.FNA ✅. B.lobectomy

🌹Question about pt with symptoms of hyperthyroidism and he is on medical


treatment for 10 months with no improvement what next!@
A-Total thyroidectomy B-Partial C-Radioactive iodine✅✅ ‫ على الحوامل‬9‫لو الحالة وحدة حامل ال نختاره الن ممنوع‬

🌹Thyroid enlargement (hot nodule) FNA normal What is next step?


A- thyroid scan B- repeat FNA C.start antithyroid therapy
....
🍓Thyroid nodule, cold, investigation? A. fna
……
Patient with thyroid nodule asymptomatic And thyroid function test all within normal
US done What’s next:
A. FNA.(bez function thyroid normal) B. Start antithyroid medication. D. thyroid scan
Answer is: A
🌹 Pt with thyroid nodule and hyperthyroidism lab , radioactive iodine shows the nodule
hot, but all the remaining thyroid is cold. Initial Management:
A. hemithyroidectomy B. tota ectomy C. antihyperthyroid drugs D. radioactive ablation
....

🌹 Hashimoto’s increases risk of: A. thyroid lymphoma B. papillary thyroid


...
Pt with hx of hashimoto’s thyroiditis for years. Presents with of thyroid mass
around 2x3, what is it? A. Papillary cancer B. Thyroid lymphoma C. Subacute thyroiditis

Q- Familial Mediterranean Fever which drug to avoid=
C. Macrolids +verapamil ✅
A. aspirin
B. NSAID
C_Amox

🌹A patient post total thyroidectomy developed neck swelling 5 hours after the surgery.
most appropriate management?
A. tracheostomy B. bedside evacuation C. percutaneous aspiration D. observation
....
🌹 27 yrs female c/o neck pain and tender thyroid, hx URTI weeks ago, sx of
hyperthyroidism, mx? A. Methimazole B. PTU C. Thyroid scan
.N.B: In another recall there was 4th choice propranolol if was there choose it
.....hot or cold ‫ هل هي‬nodule ‫ هذه‬9‫ الزم بعدها نعمل ثايرويد سكان عشان نشوف‬9‫ في الثايرويد‬nodule 9‫اذا شوفنا‬
🌹Medullary thyroid cancer mange? Total thyroidectomy

🌹2 cases about asymptomatic neck node lymph node, no thyroid signs , normal labs, FNA :
normal follicular thyroid cells the dx: A. ectopic thyroid B. lymphoma C.Metastatic CA
.‫ لكن هي ليست الغدة الحقيقة لكن سرطان يشبه خاليا الثايرويد فالزم نستاصله‬thyroid ‫ تشبه بالضبط‬C‫ اعرفوا انه فيه ورم خالياه‬normal cell folical ‫لما يقولك‬

🌹 normal follicular thyroid cells , how to manage :


A. repeat FNA B. reassure C. follow up clinic D. hemithyroidectomy
....
🍓female , with lateral mass (lymph node) , thyroid was normal , biopsy showed normal
follicular cell, what is the diagnosis? follicular metastasis
..
🌹Normal thyroid, cervical lymph node bx showed normal follicular thyroid cell dx?
A.Ectopic thyroid B.follicular carcinoma C. lymphoma D. apparent ✅✅
..
pt with left Lateral neck mass ((third triangle just below angle of mandible)) ... U/S thyroid
normal and post. Cervical lymph node enlargement, on specimen it’s shows follicular
thyroid cells = A- thyroid carcinoma/ or lateral aberrant. B- apparent thyroid gland.✅

🍓Case of nodule in the cervical LN show follicular cell :


A. aberrant✅ B. ectopic

🌹 Pt with normal thyroid and swelling in the neck this swelling is cervical LN and FNA
showed normal follicular thyroid tissue :
_Remove this LN. _Refer to surgery ✅ ✔. _Radio. _Chemo

🌹 Thyroid nodule measuring 4 cm hemithyroidectomy


....
🌹Thyroid nodule measuring less tnsn 1 cm =follow up

🍓All things is normal but have pain in his nick they chick his left neck and found small mass
5*7 mm what is the most important thing to do :
A. FNA. B. complete his thyroid investigation✅
......
🍓medullary thyroid carcinoma management? total thyroidectomy

🍓hematoma after thyroidectomy. Next? A. OR. B. bedside evacuation✅


........
🌹Patient post thyroidectomy, during the ward he has expanding neck swelling with
difficulty of breathing what to do?
A. Immediate bed side opening of wound B. neck US C. take to OR

🌹High TSH , low T4 ,T3 , high ESR diagnosis? Subacute thyroiditis 🌹


….
🌹Complication after thyroid surgery > nerve injury ( Hoarseness of voice = recurrent LN )
....
🌹Insufficient thyroid FNA, what would you do next?
• thyroid scan. • repeat FNA✅
When to repeat thyroid fine needle aspiration cytology?
The aim of of the study was to investigate the possible changes of primary fine needle aspiration (FNA) diagnoses after
subsequent check ups. We investigated 948 thyroid nodules and the main indications for repeat FNAs were
inadequate/indeterminate FNA findings and growing nodules at ultrasound check up. FNA findings were subdivided
into inadequate, benign, low-risk lesion
.......
🌹Strongest indication of thyroid surgery
A. -pediatric B. -presence of eye symptoms. C. -presence of anti tsh . D. -failed antithyroid meds.

......
🌹pt with hyperthyroidism feature and irregular irregular pulse first test?
Thyroid function tests
….
🌹Thyroid Bethesda IV management? A.Total thyroidectomy B.Hemithyroidectomy ✅
‫ = ربط‬beth = ‫ = بيت‬hemi = home .
🍒25 years female with thyroid nodule TSH and T4 normal , FNA cytology done and
according to Bethesda classification Stage IV what is the most accurate management ==
LOBECTOMY
🍒25 years female with thyroid nodule TSH and T4 normal , FNA cytology done and
according to Bethesda classification Stage III what is the most accurate management =
repeat FNA and observation
...... ....
🍒 same question and same choices but besthesda Stage VI = Near total thyroidectomy
....
=‫ربط‬
🍒 Stage lV =LOBECTOMY
V ‫ بعدين يجي‬I ‫ الي هو اول حرف مع أول‬L ‫حرف‬
🍒 Stage III=repeat FNA and observation
‫ كلمة‬repeat ‫ حرف‬I ‫ ثالث مرات‬9‫متكرر‬
🍒 Stage VI= Near total thyroidectomy= ‫ نشيل الغدة كلها‬، ‫ المرحلة االخيرة‬.

🌹Patient with large left thyroid mass, FNAC shows follicular cells(bethsada 4) what is the
management? A. Repeat FNA B. left lobectomy C. total thyroidectomy

🍒 unilateral neck swelling in the RT side by investigations : hot thyroid nodule TSH is high,
T3, T4 low No LN enlargement Treatment?
A- antithyroid drug✅. B- RT thyroidectomy. C- Hemithyroidectomy. D- radioactive iodine
...
🍒 Patient came for 3 months neck pain, thyroid function test all normal, ultrasound:
single solid thyroid mass in right lobe. Best next management ?
A.Thyroid scan. B.FNA✅. C.Imaging Follow up. D.Right lobectomy.
.... .... ....
🍒 neck mass, pt had hashimotos thyroditis, found malignant cells, dx:
A- follicular. B- medullary. C- anaplastic. D- lymphoma ✅✅
Hashimoto's thyroiditis may increase the risk of developing a rare type of cancer called
thyroid non-Hodgkin's lymphoma
. . . .. . . .
🍒 Euthyroid thyroid nodule, what you will do? Fine Needle Aspiration ✅ ✔
. . .. . .. ..Fine needle ‫ بما انه طبيعي نحتاج الى ابرة رفيييعة جدا‬، ‫ يعني وظائف الغدة طبيعية‬Euthyroid : ‫ربط‬

🍒 pt with enlarged lymph node and normal thyroid FNA of LN showed norm'l follicul'r
thyroid tissue : Abx. B- Remove the LN C- Refer to surgery ✅

🍒 toxic nodule and rest of the gland is suppressed


A-Radio ablation B-Thyroidectomy
If small go for radio If large or cuusing compression, go for hemithyroidectomy
.. . . . . . .
🌹Patient with congested throat symptoms and 2 cm palpable cervical lymph nodes. What
is the most appropriate investigation?
Chest x-ray. Neck CT. Lateral neck x-ray. FNA of the lymph nodes✅ N.B: FNA if there’s
worrisome features:- 6 weeks without resolution + lymph node more than 2 cm in siz

🌹Patient in his fifties K/c of hashimotoʼs thyroiditid for years Presents with neck
mass, it is engulfing carotids , invading capsule bx showed malignat thyroid what is the
type: Medullary Follicular Lymphoma ✅ Anaplastic

🌹Pt with diffuse thyroid enlargement on exam there is 1 nodule in each lobe, labs showed
hyperthyroidism what will you do? A. Thyroid scan. B. FNA from both nodule
….
🌹 48 yo lady with diffuse goiter, high T4 low TSH, US show bilateral thyroid nodules , right
3x4 in size , left is 1x2 size what to do? A. FNA both B. FNA the larger one C. total
thyroidectomy!@

🌹Patient with spiral fracture, parathyroid hormone high, ca high.. Most likely diagnosis?
a) Parathyroid adenoma ✅🌹 b) Parathyroid carcinoma ❌
….
🌹A guy came with swollen legs with after hitting something. Examination and imaging
showed spiral thigh fracture with reabsorbed periosteal something? Labs
showed high PTH and Ca. Asking about the disease == hyperparathyroidism
...........
🌹parathyroidectomy pt have increase in ca ? Missed adenoma
….
🌹 Indication of elective parathyroid gland removal?
A. Age > 50 B. Evidence of osteoporosis.
.....
After near total thyroidectomy the patient have persistent calcium decreased despite
giving him calcium multiple times. What to give him ? Mg

1Post ope
🌹 Pt Postoperative blood transfusion develope fever and pain at site of infusion
A. febrile non hemolytica reaction B. hemolytic reaction C. bacterial contact

🌹 Pt post op triple A repair become unstable even with fluid replacement and
have negligible urine in cath decreased vascular resistance and increased
cardiac out put.Type of shock ?
A. cardiogenic B. Hemorrhagic C. anaphylactic. D. septic

🌹 female had a hip fracture now before 2 weeks I think she had a surgery.with a
complication of DVT as I remember now she is on heparin but her platelet is low 58 so
what will you do?
A. Shift to SC enoxaparin B. Stop heparin and start other anticoagulant✅

....
🌹 Patient morbid obesity for treatment what will u do before?
A. endoscope B. ultrasound C. CT D. barium swallow

🌹Pt with progressive abdominal pain post sleeve, what to do:


A. Laparoscopy B. CT C. Exploration
....
🌹 POST bariatric 3 months with worsening abd pain : A. us B. Endoscopy C. Laparotomy

🌹 pt. with heart failure and admitted for surgery for some disease and connected
to IV fluid, post-op 2 days later complained of SOB and bilateral basal.crepitation, how
could this be prevented:
A. IV Furosemide immediate post-op B. monitoring IV fluids daily✅
‫ سوائل كثير الن فشل االعضاء هذه صعب جدا تطلع الماء برة‬9‫ او ياخذوا‬9‫مرض مثل فشل القلب والكلى انتبهوا يشربوا‬
‫فراقبوا كمية السوائل‬،‫الجسم فيتراكم بالجسمم‬
...

🍓Scar post surgery increasing in size?Keloid

🍓during laparoscopy doctor just start the procedure pt become hypotensive 84/50 what’s the
cause:
A. cold gas B. increase preload C. rapid inflation of the abdomen✅
Peritoneal stretching > vagal stimulation

🌹 Most common complication post operative?-


wounds infection. - pneumoni'. -Atelectasis ✅

- 24 hrs postop patient has dyspnea and hypoxia cause?


a. lower lobe atelectasis ✅b. PE

.....
🌹patient with abdominal mass after lifting heavy objects mass not change with cough. Dx?
a) rectus sheath hematoma b) hernia

🌹Pt 56 c/ o sudden RLQ pain after lifting heavy object ; and mass in RLQ ; Cough
negative ; abd muscle tense even with clenching . He is on anticoagulation due to
A fib What the most appropriate mgx ? (Rectus sheath hematoma)
A. angioembolization and .. B. rest and analgesic and stop anticoagulants

. . . . . .. . .
🌹Morbid obese pt with GERD wich procedure is good for him ?
A. Sleeve gastrectomy B. Biliary C. Roux y

🌹13yo obese boy found to have large hiatal hernia and grade 3 reflux asking
about bariatric surgery choice: A. Roux-en-Y Gastric Bypass B. Sleeve C. Balloon

🌹Obese male tried lifestyle not effective He is diabetic hypertensive what to do , Weight
125 ,Hight 173 , What next ? ( You should calculate BMI )
A. Medication lower weight B. bariatric surgery C. Exercise D. diet

🌹Women obese HTN diabetes controlled BMI 28 Tried excersise and lifestyle
modification but didn’t work What to do?
A. Bariatric surgery B. Orlistat (druge for obsity)

🌹Case of pt with constipation for 1 week then developed watery diarrhea,


he has tender abdomen and small distention = A. Constipation B. Acute colitis
🌹29 Years old male did sleeve gastrectomy 6 days ago, came to the ER complaining of
persistent vomiting since operation, on exam nothing significant,ABG normal( there were
numbers),Us normal. Best management?reassure
‫ ف هنا االعراض طبيعي ف نطمن المريض بينما لو حصل له الم شديد جدا في بطنه او‬.‫الن طبيعي ا مع عمليات تكميم المعدة يحصل هذه االعراض وهو استمرار القيء وتعب وهذيان‬
‫ استفرغ دم هذا ماينفع نطمنه الزم يروح للمستشفى‬.
-- ----- ----- -----
🌹Diabetic and HTN .., BMI > 40 (you calculate .. height and weight were given) .. he tried
to reduce weigh but still obese .. he is not on medications .., NEXT:
A- Medications reduce weigh B- Sleeve surgery ✅✅. C- Diet D- Exercise
‫ بينما لو‬، ‫ ويشكتي من اعراض السمنة ومضاعفاتها مثل سكر وضغط وكليسترول‬35 ‫ او اذا شخص كتلة جسمه فوق‬، ٤٠ ‫شوفوا متى نعمل عملية السمنة ؟؟ اذا شخص كتلة جسمه فوق‬
‫ نختار‬٣٥ ‫ اقل من‬A
-- ----- ----- -----
🌹Patient done gastric sleeve after 3 days come irritable and have some blotting what
to do? A-upper endoscopy B-laparotomy C-stool analysis D-reassurance ✅

🍒-Patient morbid obesity for treatment what will u do before?


a. endoscope b. ultrasound c. CT d. barium swallow

🌹70 year old pt fall on the ground surgeon will do hip replacement surgery before surgery
procedure what is the best prophylaxis thrombolytic drug for him ? LMWH , enoxaparin .
🌹After surgery by 4 days patient developed MI .. what to give ? angioplasty
. thrombolytic ‫ لو مر عليها ايام نعطيهم مذيب للجلطة‬، Angioplasty ‫ او يسموها‬PCI ‫ دقيقة نعمل لهم قسطرة الي هي‬90 ‫ لها‬MI ‫لو مدة ال‬

🌹Pt with pelvic fx and bleeding per rectum >> urethrogram > retroperitoneal
urethra injury Mx?
A-Folly's cath B-Supra pupic cystostomy C-Labroscopic repair

🌹Membranes urethra injury, management?


A. Fully Catheter B. Retropubic catheter C. Retropubic repair D. Penial repair
🌻 Membranous urethra injury is posterior injury
/….
Old patient had percutaneous cholecystostomy, presented after 24hrs with
upper GI bleeding what is the most important investigation?
A. angiography. B. Endoscopy. C. Ultrasound D. CT (they didn't mention w contrast)!@
sPseudoaneurysm of hepatic artery is a rare but known complication of laparoscopic cholecystectomy (LC). Such
pseudoaneurysms may bleed in biliary tree, upper gastrointestinal (GI) tract or peritoneal cavity leading to life-
threatening internal haemorrhage. It is very rare for them to present as lower GI bleeding.
……..
Female with thyroid mass asymptomatic us shows hard nodule what are the next steps?
A. Thyroid scan with iron B. Fna C. Ct D. Mri

🌹 What is the fovarble place for AV fistula?


A-brachial artery basalic vein B-brachial artery cephalic vein
C-radial artery basalic vein D-radial artery cephalic vein ✅✅✅
🌹 Q about renal impairment with arteriovenous anastomoses or shunt
A- arteriovenous B- brachiocephalic C- radiocephalic ✅✅
🌹55 man hypertensive and has renal insufficiency. He is evaluated for AV fistul
his non dominant hand , what type of fistula should be offered for him first=A-B
Patient diagnosed with CKD now he is coming in outpatient clinic and you decided to
method of access to start hemodialysis?
A. Arteriovenous graft‫ جرافت مشاكلها كثيير‬B. Arteriovenous fistula
C. Tunneled venous catheter D. Non tunneled venous catheter

🌹2 Pics of child’s hand with distal radial fracture with and angulation and Deformity
reduction with below elbow cast
.....
🌹 Patient had thenar muscle atrophy however his sensation was intact over that
Radial B.Median ✅✅ C. Axillary D. Ulnar
.........
🌹 Hypothenar atrophy but sensation intact? Ulnar injury

🌹weak of dorsoflection of hand , which nerve ? Radial 🌹

1Hydatid cyst

🌹Echinococcus syptomatic calcified Rx? A.Surgery(cystopericystectomy)


✅🌹 B.Metronidazole C.Aspiration

🌹 Case of liver cyst (echinoccosis) treatment ?. Albendazole ✅ ✔


(‫ اذا )هاي مع قلب‬Hydatid .= ‫= من اسم‬Hy + ‫ قلب‬، ‫ = ألب‬Albendazole = ‫نربطها‬
🌹 hydatid cyst 10*15 cm with multiple daughter cyst inside the main one.
What is the treatment? A. Albendazole B. PAIR. C. deroofing surgery
....
🌹Picture of Ct hydatid liver cyst: One large, manage: A. open cyst deroofing ✅B.
albendazole
.‫ المتجمع داخلها‬pus ‫ يعني افتح الكيس بالمنظار واشيل‬deroof ‫ لكن لو كبير الزم اشيله كلمة‬، ‫ وهو يختفي باذن هللا‬alben ‫ صغيرة نعطيه دواء‬cyst ‫لو كان‬

🌹Pic of Hydatid cyst on U/S it was big but daughter cysts was mentioned. What should
you do: Surgical Deroofing
.daughter = ‫ديرفينج = ظفييرة‬
....
amebic liver disease, what drug to give: Metronidazole
‫ نطفي المتروا ب الميبوا‬9‫ و متروا حصل فيه حريق‬، 9‫ ميبوا كريم حروق‬، ‫🔥🔥 ربط‬
..... .....
🌹 Classic amebic abscess scenario with clinical and imaging findings. Management?
○ Metronidazole ✅. ○ Drainage. ○ Aspiration
-- ----- ----- -----
🌹 Long scenario with liver abscess culture reveals anti amoeba histolytica treatment?
A-liver transplant B-metronidazole ✅✅. C-drainage ‫اول شيء مضاد حيوي وبعدين دراين‬
M=M Firstly metronidazole then drainage
.......
🌹Large cyst >10cm or complicated:- Surgery 🌹

🌹Patient with hydatid cyst, complex with septations and 10x15 in size, initial step in
treatment?
A. Cephalosporin B. Metronidazole C. Percutaneous aspiration D. Percutaneous drainage

1Operative
🍒Indication of surgery for Aortic stenosis?A. pt symptoms EF less than 50

🌹after surgery what is best dvt prophylaxis? enoxaparin and mechanical

🌹 lady was admitted and discharged just a day before she experienced severe right leg
pain and edema. (Basically she was discharged and the next day she got this presentation)
diagnosis:A. DVT B. Acute thrombotic disease C. Acute embolic disease

🌹 Patient has resection rectal surgery .. next days has left leg pain what do you give ?
Enoxaparin

🌹 Patient was stabbed in the right lower side of his chest, FAST revealed free abdominal
fluid = A-Thoracotomy B-Laparotomy ✅✅. C-Tube thoracostomy
Laparotomy ‫ عمليات الن معناها فيه نزيف داخلي ف نختار‬OR ‫ فورا على‬FAST‫ = ظهر في ا‬free abdominal fluid ‫اول ما نسمع كلمة‬

🌹 pt complain of malena and epigastric pain diagnosed as peptic ulcer and received ppi but
not improved endoscpy done showed multiple ulcers in the antrum what is the most
accurate management : a.antrectomy✅. b.partial gastrectomy c.total gastrectomy
antrum ‫ في‬ulcer ‫ فالسؤوال هنا يقول انه حصل‬antrum ‫ ؟ المعدة تقسم الى ثالث اجزاء اخر جزء من المعدة اسمه‬antrum ‫تعرفون ايش هو ال‬
.antrectomy. ‫ عالجها‬antrum ‫فالعالج القرحة هذه ه باستئصال الحزء الي حصل فيه قرحة ومن اسمها‬
Patient known case of peptic ulcer disease in medications but failed to respond, uppe endoscopy done and revealed multiple
ulcers in antrum. What is the best treatment?
A. Pyloroplasty and vagotomy✅✅✅ B. Total gastrectomy C. partial gastrectomy

Vagotomy was a way to reduce the acidity of the stomach, by denervating the parietal cells that produce acid. This was done
with the hope that it would treat or prevent peptic ulcers. It also had the effect of reducing or eliminating symptoms of
gastroesophageal reflux in those who suffered from it.🍬
🌹 anterior abdominal stab wound , the omentum is bulging out through wound ?
Exploratory laparotomy✅
..... .. . .. .. . . . .
🍒 Patient with right lower chest stab wound. Fast showed free abdominal fluid. What is
your next step = exploratory laparotomy ( Not chest tube) 🍒🍒
‫ على طول عمليات الن دليل على وجود نزيف داخل‬free fluid by FAST ‫ كلمة‬9‫اول مانشوف‬
.......
🍒🍒Pt diabetes pregnant and baby has diaphragmatic hernia and baby driver and do for
him resuscitations and stabilization what next step = surgery🍒🍒
….
Pt is with non-acth dependent cushing for rr adrenalectomy ..what is the
postoperative management ?!@
A. postoperative fludrocortisone (my answer not sure )
B. postoperative metatone ?
C. preoperative hydrocortisone
D. pre operative (drug for pheochromocytoma forget its name )

1Peripheral artery dis


🌹Acute limb ischemia with absent distal pulse, diminished sensory and
altered motor ex, mx? A. heparin. B. Above knee amputation. C. Cath and thrombolysis

🌹 pain ,paresthesia ,pallor ,,Hx of MI before weeks ,Dx?
A. Acute arterial thrombosis B. Acute arterial embolism C. DVT!@

🌹An elderly man came with lateral malleolus ulcer, he is hypertensive but
not diabetic What is the first thing to do A. venous duplex B. check pulse C. ABI

🌹Sudden right lower limb pain. Diminished sensation + altered motor. Pt on


anticoagulant for afib.Best next step:
A.CTA B.Us C.emblectomy

🌹What is the best to confirm arterial disease ?


A. Catheterization B. Ct angio C. Conventional ct

🌹Patient with HTN and diabetes came with ingrown toe : what is the most important thing
to do before the surgery :A- check pulse B- check the other toe

🌹 Old male with htn and dm presented with claudication examination reveals
right femoral intact pulse and diminished popliteal and distal pulse and left
diminished pulse intervention?:
A. CT angio B. conventional angio C. mra D. venous US

🌹Old male with htn and dm presented with claudication examination reveals
right femoral intact pulse and diminished popliteal and distal pulse and left diminished
pulse whats the most appropriate intervention:
A. ct angio B. conventional angio C. mra D. venous US

🌹 50 years old patient, know case of DM, HTN presented with leg pain associated
with hair loss , ABI more than 0.9, what is your diagnosis:
A. acute thrombus ischemia B. Chronic limb ischemia

🌹Diabetic patient with pseudo hyper epithelialization in situ= ulcer Debridement

🍒lateral malus ulcer with abaent pulse most IMP investigation?A. CTA✅ B. venous
doppler

🌹Dm pt with unilateral leg erythema Increases in dependant position, Cold, Femoral pulse
is present, distal pulse can be palpated No tenderness, no swelling, no fever Otherwise
unremarkable Dx?
A.Cellulitis B. Arterial insufficiency ✅✅ C.Superficial thrombophlebitis

🌹Patient with diabetes and hypertension smoking 40 c per day came to er complaining of
leg pain on examination on lower limp the pulse was intact on femoral and pupletial
artery’s and diminished on distal pulse first question was what is the best appropriate next
step ? A. morphin B. heparin ✅ C.ct angio D.vascular us

🌹 Pt have deep thigh injury what to do =A.torniquete B. put pressure above site of
injury ✅ C.put pressure on injury

🌹Pulse abdomen mass in middle abdomen what next step to confirm management ?
A. angiography B. CT C. US ✅
N.B: dx is abdominal aortic aneurysm intial US, confirmation and management plan
according to CT angio

🌹Pulsatile abdominal mass Abd X ray showed no air level What investigation to
order:A. abd UC B. CT angio

🌹60 years old patient come with epigastric pain radiating to back . He’s smoker and long
standing for DM and HTN . Abdominal examination show pulstile
subra umblical mass . What’s Dx ?
A. Abdominal aortic aneurysm B. secondary liver metastasis C. renal cell carcinoma

🍒Surgery was done for diabetic septic foot ,no wound healing for 2month what is the
cause? A.Neuropathy B. Poor blood supply ✅✅

🌹PT admitted due to MI after 2 days of discharge developed sever pain in his leg Dx?
1/ acute arterial thrombosis 2/ Acute Arterial Embolus ✅ 3/ DVT

🌹 claudication with distance peripheral artery disease. ABI = ankel brachio index🌹
🌹 what is the first symptom to be find in compartment syndrome =
a.pain ✅. b.parasthesia between toes. c. swelling

🌹 Patient with thigh hematoma, which of the following will indicate surgery?
Pain✅. Caludication. Cosmetic Congestive HF
‫ وهذا ممكن يسبب‬thigh hematoma ‫يقولك عنده‬
. Compartment syndromes = Due to bleeding will compresstion to Tissue and blood vessle
‫ طيب من عالمات‬، ‫ قبل ما تتكور الحالة ويحصل بتر‬، ‫ الزم فورا يروح للعمليات‬Compartment syndromes ‫ف لو ظهر اعراض و عالمات‬
‫ فالجواب هنا متى اطلعه للعمليات لما‬، ‫ وكمان تخديى وتغير اللون واشياء ثانية‬، ‫ هو الشعور بااللم ويكون الم شديييد‬Compartment syndromes
‫ ل بداية حصول كومبارمنت سيندوم‬9‫يحس بااللم الن االلم من العالمات الهامة‬

🌹 Crush injury to his feet , after stebalization of the patient , o/e cold and
decreased pulse in the affected foot . What is the best next step u will do ?
A. ct angio B. angio C. duplex US D. compartment compression

🌹Patient came after limb trauma, with severe pain and paresthesia between his toes and
it was pale. X ray shows fractures, intracompartmental pressure was 35mmhg what to do?
A. Internal fixation B. closed reduction C. external fixation
D. external fixation with 4 fasciotomy
....
🌹pt involved in RTA stable, but there is left leg swelling and paresthesia between toes xray
showed fracture tibia pressure in posterior leg compartment is 35 mmHg :
a.internal fixation b.external fixation with multiple fasciotomies ✅✅
c.internal fixation with multiple fasciotomies
….
9/ pt involved in RTA stable, but there is left leg swelling and parasthesia between toes xray
showed fracture tibia pressure in posterior leg compartment is 35 mmHg :
a.internal fixation. b.external fixatin with multiple fasciotomies
c.internal fixatin with multiple fasciotomies✅
….
🌹 30 years old had trauma which resulted in fracture of his right tibia and fibula After a while he
started to complain of numbness and severe pain within that area What would you do?
A.Heparin B.Embolization C.Fasciotomy ✅✅

🌹 wi th Reversible Eye manifestation for 20 min? TIA ✅ ✔ transient ischemic attack


‫ يعني عابر شيء مؤقت ويرجع للطبيعي‬transient ‫من اسمها‬
🌹 Female patient after cryotherapy for varicose 2years ago complainig of leg and
foot pain what nerve is damaged A- siatic. B- saphenus ✅. C- femoral D- obbirator

🌹Female 30s asking for cosmetic treatment of varicosity in her thigh,


asymptomatic, no other varicosity, how would u investigate?
A. Duplex B. CT venography C. No need for further investigation
/…
🌹lady who have Spider nevi she she wants to treat it cosmetically:( they mean before
treatment what should you do ) A) No need for investigation B) US doppler

🌹 ▪most common location for arterial ulcer >> Lateral. No pulse>> arterial
🌹 ▪Most common location for venous ulcer >> medial. pulse >>> Venous
*But the pulse will be the clue .*
🌷arterial ulcer= Lateral + No pulse🌷 🌷venous ulcer = medial + pulse 🌷
medial ‫ عادي في االمان‬venus ‫ يعني جدا حساس على الحافة يادوبك لو سار له اصابة راح يهدد حياة الشخص بينما‬latral= ‫ربط= الشريان على الحافة‬

🌹 female present with bilateral lower limb swelling non putting not known to have any
medical illness : A-CT angio B-duplex ultrasound✅

🌹 Patient with unilateral left leg non-pitting edema, skin thickening and hyperpigmentation
(dermal fibrosis) for 2 months. Next best appropriate step?
○ CT angiography. ○ MRV. ○Duplex US ✅ (initial and the next)
NOTES: Non-Pitting edema is due to Lymphedema, Ultrasound is useful to exclude other
etiologies like DVT, venous insufficiency and can also help in identifying tissue changes and
masses that might be the cause of lymphatic compression.

🌹 Patient with unilateral left leg non-pitting edema, skin thickening and
hyperpigmentation (dermal fibrosis) for 2 months. Management?
Ÿ Lymphatic bypass
Ÿ Compression bandages (combination of physical therapies; two-stage approach) ✅
NOTES: Nonspecific treatment of lymphedema includes exercise, elevation, compressive
garments, manual lymphatic drainage, intermittent pneumatic compression, and surgery
(excisional procedures, microsurgery).
. . . .. . . .. . .
🌹 same question but swelling unilateral ask about management
- A- lymph by pass. - B- anticoagulant. - C-lymphatic massage ✅
‫ الليمف اديما هو في االنتفاخ الشديد بسبب تجمع السوائل و ياثر في مظهر‬9‫ اوال مشكلة‬، ‫ االن نيجي للعالج‬، Lymphedema ‫طيب تكلمنا كيف نشخص‬
‫ ف اول ك عالج دوائي لهذا المرض الاايوجد مافي عالج نهائي له ولكن فيه عالجات مثل رياضة معينة‬، ‫الرجل تكون جدا منتفخة وثانيا تسبب الم‬
‫ ماراح تخليه يختفي لكن يقلل من االنتفاخ بحيث يوزع‬، ‫ او نلبسهم شراب ضاغط كل هدف هذه العالجات تقلل االنتفاخ‬masaage ‫ نسوي لهم‬، ‫للقدم‬
.... ‫السوائل هذه المتجمعة ويقلل االلم‬
There's no cure for lymphedema. Treatment focuses on reducing the swelling and controlling the pain. Lymphedema
treatments include:
1) _ Exercises. Light exercises in which you move your affected limb may encourage lymph fluid drainage 
2)_ Compression garments. Long sleeves or stockings made to compress your arm or leg encourage the flow of the
lymph fluid out of your affected limb.
‫ الي هو (شراب ضاغط ) مثل شراب طويل يكون مرة ضيق على الرجل‬.
3)_ Massage. A special massage technique called manual lymph drainage may encourage the flow of lymph fluid out
of your arm or leg.
. . .. . . . . . .. .
🌹Female on her fifties develop unilateral leg non-pitting edema with skin
discoloration (dark) and thickening of skin ,mx? -lymphathic bypass. -anticoagulant
-lymphatic massage and pressure dressing ✅✅ ‫مليون صح كان الدكتور االوعية الدموية يعطيهم شراب ضاغط‬

🌹presentation lymphedema, what is the important thing in hx to ask about?


A. Medication B. Family hx C.Trauma D. Smoking
: ‫ مثل‬truma events ‫ للمرة االولى بعد ما يحصل اي‬lymphedema ‫ممكن يحصل‬
such as bruises, cuts, sunburn, and sports
injuries
‫ ؟‬Q‫ نسالهم هل حصل سابقا اي تروما‬Q‫فمهم في الهيستوري‬
.....
🌹Bilateral great saphenous vein mange ?
A. sclerotherapy B. something endoscopic C. ablation

🌹Lateral malleolus ulcer. Diabetic. Intact pulse. Best initial test?


A- Douplex affected leg✅ B- Douplex both legs C- CT angio

🌹progressive Non pitting edema, no DM or HTN. Best Investigation?


A- venous douplex B- CT C- MRA

🌹Male, diabetic, HTN, came with ulcer in lateral left malleolus for 3 months, pulse is
intact? A. MRA. B. Ct angio. C. Conventional angio. D. Venous duplex for left lower limb

🌹Pregnant lady in 15 WK gest comes with marked upper & lower limp oedema.
BP 150/95 What is your best management? A.complete evaluation ✅ B.atenolol
Note :Edema happens in normal pregnancy but upper & lower could be significant.
...... ....... .....
Lateral malleolus ulcer. Diabetic. Reduced pulse. Investigation?
A- Douplex affected leg B- Douplex both legs C- CT angio ✅
...... .. . . . . .. . . .. . .
🌹Diabetic patient present with leg redness increase with dependent position, on
examination, diminished pulse and cold skin What is the most likely diagnosis? Peripheral
arterial disease ✅🌹

🌹 Patient present with painful of cord like swelling of left leg and she has hx of vircous
vein 20 years back Best initial treatment? a) NSAID ✅. b) Celxan. c) Heparien!@

🌹Varicose veins Tx for therapeutic reason: - Endovascular laser ablation 🌹

🌹 Varicose veins Tx for cosmetic: = -Sclerotherapy 🌹

🌹Pt w/t varicose veins c/o swelling and heaviness, no pain, pulse intact .. everything
normal .., NEXT:A- Duplex US✅. B- MRV. C- CT. D- NO need for investigation
.. . . . .. .
🌷spider veinous case , what is the management? -Endoscopic laser ablation
‫ فعالج هذا‬، ‫ و دوالي المريء هذه تعمل نزيف‬esophageial varicosis ‫ تعمل‬cirrosis ‫ ومن مضاعفات‬liver cirroais ‫ عالمة على‬spider vein ‫الن‬
. . . .laser ablation ‫الدوالي يكون ب‬
….
132-Female presented with vascular malformation in the leg (hemangioma
maybe) increasing. What is the indication of removal? A-Pain✅✅. B-Ischemia C-Cosmetic

🌹 What is hard sign of vascular surgery:-_Weak pulse _Skin color change. _bruit ✅
the presence of hard signs mandates immediate action. The presence of hard signs of
vascular injury mandates immediate operative intervention.
....... ........ ......
🌹 Hard sign to detect vascular injury
A. multiple fracture B. change color C. decrease pulse D_pulsetile bleeding

● Smoker elderly with preserved femoral and popliteal pulse put diminished
dorsalis pedis, Mx ? A. Localized thrombolysis B. Heparinization and observer

1Testicular
🌹Infant with Bilateral non tender scrotal edema and redness extending to groin. What's the diagnosis
A. Testicular torsion B. Epididymo-orchitis C. Edema of testicular appendages
D. Idiopathic testicular edema

🌹Patient came with testicular enlargement after an exercise what will you
do for him? US

🌹decrease in left testicular size post hernia repair ?


A. Testicular artery occlusion. B pampiniform plexus occlusion

🌹Pain with absent cremasteric reflex: A- Surgical consult ✅ B- US C- Culture


‫ وهذه حالة طارئة الزم‬testicolar torstion ‫ ايش اول شيء يجي في بالنا؟؟‬absent absent cremasteric : ‫مثل ماقلنا سابقا نشوف بس كلمتين‬
. ‫فورا جراحة‬
🌹5 years old child found to have one testes in the scrotum and the other in
the ingunal area what to do? A-orchiopexy ✅ ✔. B-orchiectomy. C-wait tell puberty.

🌹Pt recent abdominal surgery presented by fever&otheres. what is the most cause
A-sepsis B- subphrenic abscess✅
(Other names. Subdiaphragmatic Abscess. Specialty. Infectious disease,
gastroenterology. Subphrenic abscess is a disease characterized by an accumulation of
infected fluid between the diaphragm, liver, and spleen.)
.....
🌹female patient complains of urinary dribbling, dyspareunia, dysuria. What is the most
likely diagnosis?
A. Overflow incontinence B. Urethral diverticulum✅ C. Stress incontinence
A urethral diverticulum occurs when an unwanted pocket or sac forms along the urethra.
......
🌹yearsold boy brought by his parents, he has nausea, severe
vomiting for 20 minutes and now semi comatose. The parents
mentioned that he has same episode two weeksago for 5 minutes
without deterioration in consciousness. On examinationthere is right
testicular mass that does not transilluminate with light. What is the best
actionto do ?- Surgical exploration. ✅

🌹Testicular pain absent cremaster reflex (for 4hours) , clinical pic of (torsion) what
to do? Surgical exploration ✅

1Appendsitis\
🌹Female (obese) with typical appendicitis: Right iliac pain and tenderness
Nausea and vomiting. Loss of appetite. Leukocytosis What is the proper management:
A. Abd ct B. Abd us C. Expiatory laparo D. Open appendectomy

● Post appendectomy female came with LR abdomen mild tenderness Ex


Normal By CT there is 2*2 collection in Retrocecal :
A. Exploring laparotomy B. percutaneous drainage C. laparoscopic D. conservative

● Pathophysiology of appendicitis in 58 male ?


A. change in blood distribution B. peripheral vasoconstriction C. decrease heart index

🌹Patient obese came complaining of rt iliac fossa and tendrenss :


A- CT abdominal B- us abdominal Answer is: A, HE IS OBESE

🌹Young unilateral testicular swelling since 1 day, on exploration viable but cord
edematous :A-Torsion B- Incarcerated inguinal hernia C- Appendicular torsion

🌹Post appendectomy day-4 presented with abdominal pain and feculent discharge most
appreciate step? A. IV antibiotics✅ B. Ex lap
‫ يقولك عنده‬pus ‫ اهم اهم خطوة والزم هو نعطيه مضاد حيوي‬9‫يعني تجمع بكتيريا‬.

🌹Case scenario ....appendicitis but not ruptured best investigation


a. CT abd✅ b. US abd c. Exploratory
If child or female > US. If adult male > CT

🌹Peds 8 yrs old with RLQ pain and rebound tenderness what's confirmatory
test? A. US abdomen B. MRI abdomen C. CT abdomen

🍓Post appendectomy came with mild pain and collection 2x2 Asking about management:
A. Per cutaneous drainage B. Conservative with Abx✅
..

🍓Post-appendectomy, 0.5 cm carcinoid found on the tip of the appendix, mx?


Nothing = If carcinoid tumor less than 2cm > appendicectomy only
More than 2cm or at base of appendix> right hemicolectomy

🍓After appendectomy dr found a carcinoid mass on the tale of appendix Next? C.Tscan
abdomen and chest for staging

🌹Most surgical emergencies in pediatrics ? Appendicitis 🌹

🌹Old age came with symptoms if appendicular mass and treat it What you will do
A. Colposcopy 6 weeks ✅ B.12 weeks

🌹after initial conservative treatment of appendicular mass :


A.Open appendectomy after 12 weeks B.Laparoscopic appendectomy after 12 weeks ✅✅

🌹Appendicular abscess 15cm*17cm in a 33 y old man. How to manage


A. antibiotics ✅ B.Surgical (No drainage in the options)

🌹 During app. Surgery appendix not find !? . How to find it = Follow tenia colli

🌹pt febrile , abdominal pain , rebound tenderness at mcburney's point , wbc high , x ray
show right fossa ? A- appendectomy✅✅ B- gall bladder US C- urology consultation

🌹1 week post appendectomy pt came with right iliac tenderness not associated with nausea or
vomiting, infrequent loose stools . CT 2*2 collection behind cecum= A. percutaneous aspiration B. open.
C. laparoscopy D. consevative Mx✅ ‫كوليشكن بسيط مايحتاج ال عمليه وال اسبيراشن وخاصة حالة المريض مستقرة مافيه حرارة‬
‫وال التهاب بس الم‬

🌹After open appendix, seroma collection and leak from gap of sutures , no
inflammation, what to do? a-Dressing b-Evacuation c-Ab

🌹 Post appendectomy case 1 week , not complain ,but upon exam there was
seroma with in gaping wound .what u will do ?A. repeat and care of the dressing
B. drainage C. ct abdomen

🌹appendicular mass management


A- lap appendectomy after 12 weeks✅ B_open appendectomy after 12 weeks
The management of appendiceal mass is surrounded with controversy. Traditional management has been
conservative, with interval appendicectomy performed weeks after the mass had resolved. 
..... ..... ....
🌹Perforated appendicitis post laparotomy, presented with fever, abdominal pain CT done
showed foreign material , reexploration done, gauze was found intraabdominal
،intraabdominal ، What to do?
A-Issue complaint against assistant B-Call your lawyer and report the incident
C-apologize to the patient and tell her what happened ✅ ✔
…..
🌹Young male with right lower abd pain and tenderness, low grade fever On abd xray:
radioopaque material (small at iliac crest site) What to do?
-urgent uro consultation-non contrast CT -appendectomy ✅
🌹Case of acute appendicitis, fecolith and abscess seen on CT management?
- open Appendectomy and drainage -lap appendectomy and drainage
-percutaneous drainage ✅
‫ تكونت وهي تحصل نتيحة التهاب الزائدة‬stool ‫ فيه قطعة‬Q‫يقولك كان‬
fecalith marked by the arrow which has resulted in acute appendicitis. Specialty. General surgery. A fecalith is a
stone made of feces.
((Drainage of Periappendiceal Abscess and Removal of Free Fecalith—‫))هذا عنوان البحث‬
.‫ وبعدها نعمل العملية إزالة الزائدة‬stool ‫يفضل اول شيء نشيل قطع‬
🌹After laproscopic appendectomy . Pt come with retrocecal collections. :
A. surgical drainege ❌❌ B. percatenous drainage ✅✅

🌹Appendicitis case with abscess how would u manage it :


A.open B.laparoscopic C.antibiotics D. percuatanous drainage ✅
..... ..... . ......
8 days Post appendectomy with abdominal pain, distention, inability to
pass stools, In exam: feculent material discharge with signs of peritonitis
A- CT B- IV antibiotic C- Exploratory laparotomy

1prostat
A 73-year-old man presents pain in his right thigh. This has been getting progressively worse for the past 9
months despite being otherwise well. An x- ray is reported as follows: X-ray right femur Radiolucency of
subarticular region suggestive of osteolysis. Some areas of patchy sclerosis Bloods tests show: Calcium 2.38
mmol/l Phosphate 0.85 mmol/l Alkaline phosphatase 544 u/L Prostate specific antigen 4.4 ng/ml. What is
the most appropriate action?
A. Vitamin D supplementation B. Check serum testosterone C. Referral to an orthopaedic surgeon
D. Referral to a urologist E. IV bisphosphonates (could be prostatic Ca mets to the bones.)
…..
🌹75 y male c/o back pain, difficulty passing urine, psa: 84(high) , ALP: 410
(high) A. prostatitis B. prostatic cancer C. urinary bladder ca D. BPH
.N.B: High ALP indicates bone metastasis.

🌹 80 y old man with bilateral hydronephrosis on Us images


A. Prostate enlargement / Ca I'm not sure.
B. Bladder ca > come more aggressive pain, hematuria C. urethral stricture
IF THERE IS BPH, GO FOR IT. IF NO BPH, CHOOSE PROSTATIC CANCER.

-Part of urethra affected by surgical trauma in males


a. membranous b. penile D.Bulbar✅

🌹Part of urethra affected by surgical trauma in males ? bulbar if not in choices penile.

🌹Part of urethra affected by surgical trauma in males


A. membranous B. penile C. prostatic N.B: Membranous for pelvic fracture after MVA

🌹Old patient with loain pain . Us shows biliteral hydronephrosis =enlarged prostate🌹
🌹By Cyctoscopy: Redness in the dome of bladder What is the most likely diagnosis:
A-ischemic colitis. B-diverticular disease. C-Transitional bladder cancer✅
…..
🌹Old M c/o urinary sx, prostate median lobe hypertrophy, what is best for this pt?
Annual Prostate-specific antigen ✅
. ‫ في البروستات من اسمها‬Q‫ هذا انتيجين يرتفع لما يكون فيه ورم‬high Prostate-specific: ‫🍬للعلم‬
..…
🌹 Old pt came with difficulty urination and low back pain with high Prostate-specific
antigen dx?! A- Prostatic cancer✅✅. B- Benign prostatic hyperplasia. C- Prostatic
inflammation
Benign prostatic ‫خذوا هذا السر ( اي احد كبير في العمر وجاء يقولك عنده الم في اسفل الظهر وكان عنده تاريخ سابق ب‬
( ‫ وسار له‬prostata cance ‫ نشك على طول انه عنده‬difficult urination ‫ او قالك من االول وانا عندي‬hyperplasia
‫ فيجيك ب الم‬spinal ‫ واكثر اكثر مكان يروح له ل مرض سرطان البروستات هو‬، ‫ يعني بدء السرطان ينتشر‬spinal ‫ ) لل‬metastasis
:‫ لو فيه مرة جاك مريض كبير بالعمر واشتكى لك من‬، ‫ اعيد لكم‬، ‫في الظهر‬
🌹 difficulty urination and lower back pain plus high Prostate-specific + lower back pain = think about
prostatic cancer metastasis .
...... .....
🌹 Urethral trauma with bleeding post MVC. Next step? Suprapubic catheter✅✅
‫ ماهي سليمة‬9‫ماينفع نختار فولي كاث الن فيها اصابة ماينفع ندخل القسطرة واليوريثرا‬

retrograde urethrogram is essential for diagnosis of urethral injury, or urethral stricture
-- ----- ----- -----
🌹 75yeard old male with ((( back pain ))) and urinary symptoms (obstructive) ,ALkaline
phosphatase hight and (((PSA 80))) what is the most likely Dx
-BPH. -prostatic cancer ✅✅
** PSA = Prostatic specific antigen = if above 4 = ‫معناها سرطان الن هذا انتيجين مايرتفع اال في االورام البروستاتا‬
.......... ....... ...... ......
🍓65 yrs came with mild decrease in urination , us showed median lob hypertrophy of
prostate , PSA and digital rectal was normal , urine analysis and renal function normal
what to do? A. annual renal function tese ✅ B. Periodic investigation of PSA C. Beta
blockernD. Cystoscopy

1Hemorrhoids
🌹 sclerosing patient with history of rectal bleeding, anoscopy show swelling at 3,7 o'clock,
sclerosing therapy is planned what is the most appropriate to do sclerosing therapy!@
A. external hemorrhoids B. internal hemorrhoids C. prolapsed hemorrhoids D. thrombosed hemorrhoids

🌹 post hemorrhoidectomy pt has urinary retention. Cause: a. Trauma b. Anesthesia affect✅

🌹Patient after hemorrhoids operation develop Suprapupic pain with inability to pass urine what is
cause? Inadequate analgesic
...
🌹Patient 1 day post hemorrhoidectomy complaining of abdominal pain and distention
and he cant urinate, and there is anterior boggy aspect of anal digital examination . What
to do?A. Its Analgesia complication B. drainage C. CT
🌹 20s years old presented with pain during defecation menimum amount of blood, on
examination the doctor seen a posterior midline fissure. He couldn’t do PR exam as the
patient was in severe pain Vitally stable What’s your next step?
A. EUA (Examine under anasthesia EUA) ✅. B. LIS. C. LES D. injection of silicone
..
🌹 Old patient who has constipation on and off with streakin of blood in the stool with no fulness in the
rectum ( no mention of pain )? A. Sigmoid cancer B. rectal cancer !@ C. chronic hemorrhoid.

🌹 Old pt-fatigue sign of anaemia hb8 , stools with streak of blood -has hemorrhoids stage 2
? A. Sigmoid ca. B.rectal ca✅✅ c.chronic hemorrhoid
، Rectal cancer ‫ وجود‬9‫ الزم الزم تستبعد‬hemorrhoid ‫ اي مريض يكون كبير بالعمر وعنده‬: ‫سمعوا هذه المعلومة‬
‫ كويس الن‬rectum area ‫ الاازم بالمرة تفحص‬hemorrhoid ‫لو انت جراح وجاك مريض ((كبير بالعمر )) وعنده‬
cancer ‫كثير بنالقي صدفة عندهم‬
🌹 Bleeding from the rectum is the most common sign or symptom that both rectal
cancer and hemorrhoids share.
symptoms include blood mixed with stool, a change in bowel habit , unexplained weight
loss in the absence of dieting, bowel obstruction, anemia and fatigue. = These symptoms
are not usually seen with hemorrhoids =it gose with Rectal cancer.
🌹 Hemorrhoids produce the symptom of pruritus (itching) in the rectal and/or anal area
while rectal cancers usually do not.

🌹 Patient came with painless blood after defecation, whats the diagnosis?
A. Abscess B. Fistula C. Hemorrhoids

🌹 Sclerotherapy use in management of ? A. Internal hemorrhoids ✅. B.External hemorrhoids

🌹Hemorrhoids type 4, what is the management: Hemorrhoidectomy


….
🌹case of hemorrhoid. what is the Indication of sclerotherapy? A- Internal✅. B- Externa

🌹Patient with perianal painful swelling -vitals normal, wbc normal Dx:
A-pile. B-perianal abscess C-perianal hematoma. D-anal fissure

🌹 Anal fissure with sentinel pile not responsive to medical therapy. Next step?
A)_Lateral external sphincterotomy. B)_ Lateral internal sphincterotomy(LIS) ✅
‫ ( = ربط‬Anal = internal ) ‫ حرف‬n ‫ مع‬n .
Lateral internal sphincterotomy is an operation performed on the internal anal sphincter
muscle for the treatment of chronic anal fissure. The internal anal sphincter is one of two
muscles that comprise the anal sphincter which controls the passage of fece

🌹 streaks of blood after defecating and pain? Anal fissures ( lateral Sphincterotomy)✅
*s*treaks of blood = fi*ss*ures
...... ..... ...

🌹 pt with intermittent perianal pain and discharge, on P/E theres low-lying fistula
with an opening in posterior wall above anal verge. What to do?
A. MRI B. Fistulogram C. Fistulotomy D. Lateral sphincterotomy
....‫ وعالجها نشيل الفيستوال‬fistula ‫واضح انه عنده‬
🌹Perianal pain on examination no lesion, finger exam showed internal swelling
with purulent discharge on the anal canal , Dx: A. perianal hematoma B. internal abscess

🌹 Male adult with perianal swelling and discharge, then ruptured =


A. Anal fissure B. hemorrhoid C. anal abscess D. fistula
.....
🌹Pt with anal abscess then you found fistula on examination draining pus , the
opening is in the posterior rectum wall: management:
A. MRI B. fistulography C. fistulotomy/ectomy (one of them only)
....
🌹31 year old male with constipation and during perianal exam there was sever pain and
linear laceration at 6 and 12 oclock whats the dx:
A_ Anal fissure B. anal fistula. C. hemorrhoids
.....
🌹pt with bleeding after defecation ( painless ) A. Anal fissure B. Hemorrhoid
….
🍓pt presented with 3 months bleeding after stool, a perianal mass, and weight
loss. Histology confirmed adenocarcin oma. CT finding shows no lymph node
involvement and mass limited to 3cm from anal verge. What's treatment modality?
A. Radiotherapy B. Abdominoperineal resection✅ C. Chemotherapy D. Low resection only
Less than 6cm from anal verge > Abdominoperineal resection
More than 6cm > Lower anterior resection.!@
......
🌹 Case of perianal swelling , per rectal bleeding , investigated : it is 1 cm from anal verge
biopsy: adenocarcinoma (rectal) Mx:
A-Abdomen perineal resection ✅ ✔ B-low anterior resection C-chemo/radio
.....
🍓Adenocarcinoma of colon, 1cm from anal vege?
A. low anterior resection of rectum (LAR) B. Abdominoperineal resection!@
......
🍓patient with anal swelling and discharged, swelling is rapture, No fever:
A- fistula abscess. B- thrombosed pile

🍓Anal itching and pain post defecation and ask ddx?


A. internal hemorrhoid B. Anal fisure ✅!@

🍓Anal fissure failed medical Treatment? lateral internal anal sphincterotomy

🌹Anal fissures with skin tag not respond to drug, want procedure?
A- Internal sphernctomy✅✅ B- External sphrenctomy C- Anal curettage with remove tag
🌹Post partum woman with painful defecation and bleeding and pain after defecation;
what is the cause A. anal fissure ✅✅ B. Haemorrhoids C. thrombosed pile

🌹pt with Anal dischsge and he gave history of ruptured aanal abcess :
Anal fistual ✅. Anal' fissure Anal piles

🌹Patient have pain with and after defication and some blood with it diagnosis?
A-hemorrhoids. B-anal fistula. C-anal fissure‫✅✅ التمزق يعمل الم‬. D-abscess

1wound
🍓Neck trauma stable anterior to the angle of the mandible A. CTA✅ B. exploration

🌹 aLots of questions on wound care/ exploration of wound/ abx — VERY


IMPROTANT TO KNOW WHEN TO DO EACH
🌹 First evaluate surgical wound by *inspection *✅
🌹 If there’s signs of infection* (pulurent discharge, redness, tenderness) then
wound exploration
if superficial > clean, dressing and you may give abx
🌹 If deep collection suspected >* order CT
🌹 If small collection less than 4cm >* abx
🌹 If large 4cm and more* > percuteanous drainage + abx
🌹 If signs of peritonitis > Laparotomy

🌹10 cm laceration in anterior thigh what to do?


a-Tourniquet b-Compress ✅. c-Vascular consult d-Band or tourniquet above femoral
artery
‫ انتوا تخيلوا الموقف لو اصابكم اي جرح كبير ونزل دن كثير ايش اول شيء تسوونه ؟؟ اول شيء بتاخذون قماش وال منديل ( تضغطون) على الجرح عشان يوقف نزيف‬.
... .... ....
🌹 Elderly pt w/t sacral ulcer .., management:
A- Depriment and skin graft(if necrosis) B- Primary closure C- Daily dressing
….
🌹 Elderly pt underwent for a major surgery ..he need blood transfusion 15 PRBCs .. after
that he start bleeding from wounds, nose, from NGT (stomach) .. what is the cause:
A- vWBD. B- thrombocytopenia✅. C- hemophilia. D- no DIC in choices
... ....
🌹 25 years old with perforated appendix did laparotomy, after 5 days wound is tender and
pus discharge what to do. a-wound dressing b-exploratory laparomtmy. c-iv abx✅
..
🌹 Girl with stab wound in the right anterior axillary line below the costal margin, stable
A. Observation. B. immediate surgical exploration. Perform FAST✅

🌹 Neck stap wound = if ask about Next step = wound explorontion (clinical exsmnation) =
if ask plan or managmant choice = C.T angio to neck ( stable pt) = if not stable Surgical
explorantion.
….
🌹trauma to neck asking about investigation to different zone:
🧁 Zone I and Zone III > C.Tangio
🧁Zone II :- if asymptomatic > C.Tangio
🧁if symptomatic > Surgical exploration

🌹 Pt with neck injury in zone I , vitaly stable , what’s next : A. neck exploration B. CT face and neck✅

🍓Neck injury->pt had Subcutaneous emphysema_> next step?


A. Ct neck✅. B. explore neck If zone I and III CTA Zone Il> surgical exploration

🌹18 yrs with stabbing wound 10cm of thigh what next step management:
A. compress on wound ✅B.tourniquet upper thigh
……
🌹 Neck trauma in area retro auricular above the angle of the mandible (area 1 was
not mentioned), patient stable and oriented. Most appropriate step?
A. CT angio. B. Neck exploration

🍓Patient has stab wound affect the duodenum and vital signs are stable what is your
management ? A. Ct scan✅ B. Laparotomy C. Conservative
.......
🍓pt with forearm fracture and open wound 1cm , what’s TTT :
A. close reduction. B. wire. C. cast D. debridement , irrigation , fixation✅
…..
🍓Pt post operative and leak 20ml fluid from the wound: !@
A. Dressing B. Wound exploration✅ C. Lap
......
🍓Post open appendectomy case with pain in wound site on examination u see
pus oozing from site of surgery what will u do next?
a. percutaneous b. US abd c. iv antibiotics d. open surgery
Antibiotic ‫ نختار‬pus or ooze ‫اذا قالكم‬
open ‫ نختار‬Fever ‫ والمريض عنده‬absess ‫اذا قال فيه‬
🍓Pt has infected wound has pus come out from it what next step?
A-Inspection B-exploration of wound
....
🌹After herniotomy surgery for 5yrs boy came with fever and pus discharge and
part of mesh seen ? A.give iv antibiotics B.draining of pus
C. draining of pus and remove mesh ✅ D. observation

🍓X Ray showing both distal ulna and radius fracture with volar displacement On
examination a 1 cm wound was seen at the volar aspect of the wrist Asks about
initial management:
A. Closed reduction with above elbow cast B. Closed reduction with below elbow cast
C. Irrigation of the wound and Closed reduction with below elbow cast
🌹Young male with MVA came by ambulance to ER conscious, with thigh
wound covered with sucking gause : call surgeon in duty
......
🌹Trauma patient with a wound on his thigh subcutaneous fat is lost vasculature
underneath is exposed what provides the best management
A. Debridement with primary closure B. Primary repair
C. Debridemnt with secondary closure D. Debridement with vacuum assisted closure
....

....
🌹 25 year old male Pt 8th day post surgery with wound site redness & tenderness with
purulent discharge.. most appropriate?
A. IV antibiotics B. CT abdominal C.open drainage D.exploratory laparoscopy
.....
🌹Patient post surgery, there surgical wound redness, tenderness with no discharge,
abdomen is soft and lax .what to do? A. Antibiotic B. wound drainage C. CT abdomen

🌹Patient post surgery, there pus coming from surgical wound with tenderness and
leukocytosis, abdominal examination was done it was soft lax with no tenderness all over,
what to do? A. Antibiotics B. wound opening C. CT abdomen
...open ‫ نختار‬Fever ‫ والمريض عنده‬absess ‫ اذا قال فيه‬Antibiotic ‫ نختار‬pus or ooze ‫اذا قالكم‬

🌹Patient was hit with wood 5 days ago, now he comes with severe RLQ pain. On
examination there is a small opening with pus discharge, put when tried to extend his
thigh there was severe pain and you couldn't move it. What to do?
A. CT B. wound drainage C. antibiotics
...
🌹Patient post resection and colostomy presents with spiking fever for 1
week. Chest is clear. Abdomen is clear. Wound is clear. Fever 38.2 Next step?
A- CT abdomen B- Reassure
....
🌹person with car accident on left thigh show neurovessle fat and tissue out mange ?
debridment and skin grafting

🌹k/c HTN with lateral mules ulcer, next appropriate management?


A- wound examination ✅. B- duplex. C- Ankle brachial indexes

🌹Pt was stabbed by knife into his neck (ant to the right ear way through his mandibule )
Hé is stable with 1 or 2 cm wound ozing blood What is the next step in managment :
A-CXR. B-C.T angio. C-wound exploration ✅
‫ مو‬forgen body ‫ نشوف هل فيه اي‬، ‫ نسوف عمقه‬، ‫ انه نفحص الجرح‬stap wound ‫ اول شيء نعمل ل‬, next step ‫طيب هو السؤوال يقول‬
.…‫ كلمة‬، . ‫ ف انا بختار سي‬، ‫على طول بدون ما افحصه اوديه للسي تي‬

.... .... ....


🌷Patient with perforated appendicitis after surgery had pus from wound, pain
localized to the surgical site. No guarding no fever what best initial treatment is:
A. Antibiotics✅✅ B. Open drainage. C. Imaging guided drainage D. Wound exploration
‫ مازال مستمر وماراح بالمضاد‬pus‫ لو ال‬culture ‫نركز هو قال اول شيء نسويه والمصادر تقول نعطيه مضاد حيوي ك اول خطوة مع اخذ مزرعة‬
. . ‫ وخااصة لو المريض قالك عنده حرارة‬، ‫الحيوي الزم افتح الجرح واشيل الغرز‬

You may be started on antibiotics to treat the surgical wound infection. The length of time
you will need to take the antibiotics varies, but will typically be for at least 1 week. You may
be started on IV antibiotics and then changed to pills later. Take all of your antibiotics, even
if you feel better.

🌷- Patient post surgery presents with disch'rge from middle of the wound. Next step?
- Daily dressing. - Wound inspection✅✅ - Wound explor'tion
‫ اول خطوة في اي فحص الاازم اول شيء‬inspection
....... ....
Patient with right lower chest stab wound. Fast showed free abdominal fluid. What is your
next step? A. exploratory laparotomy ✅ B.thoracentesis C.chest tube
...
🌹Pt e deep wound 10 cm in front of thigh. What's your best next step =
A.tourniquet at level of femoral B.torniquet above wound level
C.apply good pressure at wound site ✅ D.call vascular surgery doctor
....... ...... .....
Patient with stab wound what to do next ? Local wound exploration✅

1radiology
🌹An elderly with IHD day 2 post cholecystectomy presented with sudden chest pain SOB and vitally
hypotension and tachycardia whats best ? A. CXR B. ECG C. CT angio D. LL duplex US (most likaly has PE)

🌹Elderly with weight loss (10kg) and anorexia in endoscopy there is large ulcer in stomach , biopsy was
taken and the result is invasive adenocarcinoma . What is the next step:
A. endoscopic US B. CT abdomen and chest and pelvis C. abdominal US

🌹Best diagnostic method for pulmonology embolism? A. D- dimer B. Spiral CT✅


... pulmonary angiography (CTPA): best definitive diagnostic test...

🌹Chronic thromboe-embolism confirme = A. QV ratio B. spiral CT C. D dimer

🌹 supraclavicular LN met what you do for primary site : Gastroduodenoscopy

🌹How to diagnose Adhesion post operative? CT, but Initially > abdominal X-ray (Air-fluid
level, Bowel dilatation)

🌹Old patient with worsening of his constipation, labs shows positive occult blood in stool.
Colon cancer suspected what to do?
A. Colonoscopy B. sigmoidoscopy C. CT abdomen
🌹 Pt melena and fresh blood per rectum, upper and lower endoscopy negative,
next investigation: Capsule endoscopic
‫ نستعمله‬، ‫ على كل الجهاز الهضمي ظن بدايته لنهايته‬9‫الكابسوال هذه فيها كاميرا صغيرة ندخلها للجهاز الهضمي ويمشي‬
‫ لما نكون عملنا‬endscope ‫ للكبسولة هذه‬9‫ فنلتجا‬. ‫ ومالقينا سبب النزيف‬.

🌹Old Man with falling trauma CT brain is normal but he is complaining of


bilateral foot pain: Check pulse > analgesia > X-ray

🌹best diagnostic for Coarctation of the aorta for neonate ? echocardiogram is the most
commonly used test to confirm the diagnosis ( Not ultrasound or C.t )

🌹 A man who is a known case of diabetes presented with hemiparesis 15 hours after
some procedure A. tpA B. Warfarin C. Ct angio
‫ فنعمل سي تي انجيوا‬small vessl brain ‫ الن قال عمل بروسيجر فممكن سار له ارتفاع شديد بالسكر واثر على‬strok ‫احتمال يكون جاه‬
blood vessel ‫ عشان نشوف‬angio ‫ اخترنا‬،
🌹Pt with bronchogenic carcinoma, presented with progressive SOB,
there’s elevated jvp, clear lung and quiet heart sounds. What will confirm your dx:
A. CXR✅ B. ECHO C. ECG
‫ يعني حصل له متالزمة اسمها‬elevated jvp ‫ وعنده‬bronchogenic carcinoma ‫اوال المريض عنده‬
‫ طيب اهم‬، SVC ‫ لل‬obstruction ‫ الن الكانسر الي بالرئة عمل‬Superior vena cava syndrome
....SVC ‫بنشوف الورم وهو ضاغط على‬. CHEST X.ray ‫خطوة لتشخيص المتالزمة هذه هي‬
🌹Pt presented with stabbed wound after wound exploration you found anterior
abdominal fascia penetration, (his vitals were stable) what's your next step?
A. CT abdomen B. MRI abdomen C. Exploratory laparotomy D. Diagnostic laparoscopy

🌹 Best way to clear show cervical spine:


A. Clinical judgement or assessment. B. Lateral cervical x-ray

🍓child episode of PR bleeding, fresh ; black stool (have both melena and hematochezia) ;
NGT showed greenish fluids ; colonoscopy was negative what is next to detect source of
bleeding ? A. upper endoscopy. B. Tc 99m✅. C. Barium. D. US
.... ......

🌹Chlid overweight with X-ray? slipped epiphyseal plate 🌹

🌹Old with back pain radiated to back , CXR air under diaghram = perforated peptic ulcer

🌹 Pt in ICU with coffee ground vomit? Stress gastritis


🌹 K/c of ulcerative on med presents with abd pain Invistigation show enlarged transver
colon 15 cm or mm? Management?
A.Total colectomy with ileostomy B.Proctocolsctomy with ileal pouch C- steroid

🌹 30 something morbidly obese male , how to decide best reduction surgery he will
have ? A.Barium enema B. ct abdomen C. ultrasound abdomen ✅ D. GI endoscopy

🌹 farmer, wood brick in his abdomen 4days ago, 3 days ago he developed severe pain in
the same area, on exam there was green discharge coming out, what will you do?
A. take culture from the discharge. B. CT abdomen. C. Drainage

🌹Stap in abdomen right anterior axially line, below costal margin, patient stable
conscious what to do? a-Ct abdomen b-Us FAST

🌹cervical trauma in ICU patient after motor vehicle accident = C.T🌹

🌹 3 weeks post gastrectomy presented with progressive abdominal pain increasing


in intensity. What is the next appropriate investigation?
1_US 2_ X-ray ✅ 3_ CT 4_Laparotomy
‫ وباالكس‬، suture ‫ من‬leak ‫ من المضاعفات الي تسير بعدها هو حصول‬، ‫ليه اكس راي ؟؟ الن الي يعل عملية بالمعدة سواء قص معدة او اي عملية‬
leak ‫ لذلك افضل طريقة للناس الي بتعمل جراحة بالمعدة كل فترة نعمل لهم اكس راي الن لو حصل‬، stable ‫ بين‬leak ‫راي راح يوضح جدا انه فيه‬
. ‫في البداية ممكن ما تعمل اعراض‬
🌹 Patient complained of abdominal pain 3 weeks after sleeve gastrectomy, what is the
diagnostic test? 1-US 2-CT with contrast✅ 3- Plain abdominal X Ray( if ask next step )

🌹 Old male patient admitted as a case of large intestinal obstruction, underwent rigid
sigmoidoscopy which showed a mass in sigmoid region, biopsy was taken and resulted as
adenocarcinoma, what’s your next step?
A. Colonoscopy ✅ B. CT abdomen C. MRI pelvis D. Sigmoidectomy
............... ....

🌹 CT in truma important for = retroperitoneal injury✅

1spleen
Splenic laceration and thoracic aortic injury. You want to transfer the
patient. What is important?*
A. Transfer him to laparotomy B. Transfer him to thoracotomy C. Angiogram

🌹 Post splenectomy came complaining of left side pain, Reduce air entry in the left side,
Dx? A. Subphrenic abscess ✅✅ B.Post splenectomy overwhelming syndrome
🌹RTA patient, presents 2 days later with ascending aorta injury and splenich laceration,
the best immediate management is:
A.Take the patient for thoracotomy B. Take the patient for laprotomy ✅ C.Call vascular
surgeon

🌹 Patient after pacreatitis episode develops upper GI bleeding picture , scope was done,
gastric fundus bleeding was found ,sclerotherapy done Duplex ultrasound showed: splenic
vein thrombosis with patent portal vein whats is your management:
A-Splenctomy ✅ ✔. B-Distal splen renal shunt. c. Portocaval shun

🌹A 12 year old received a nonspecific blunt trauma on his abdomen and later presented
with generalized abdominal pain. Imaging of the spleen showed a 7mm hematoma and 4
cm tear(grade 3). Your management:
A. splenectomy B. Spleen preserving surgery C. Conservative
....
🌹Spleen injury 1= consertive treatment
.....
🌹Spleen injury 3=partial resect. ( preserving surgery )
....
🌹4 cm tear with hematoma 7cm grade 3
A. splenectomy B. splenic conservative surgery
...... ‫ جمل‬3 ‫ مع‬3 . ‫ حسب جايد الين‬3 ‫ الن جراد‬spleen‫يعني نعمل لهم جراحة بدون استئصال ال‬
🍒Laceration spleen grade 4 ? Splenectomy 🍒

🍒You are in (( small hospital or something )) and you have a patient with aortic injury and
spleen laceration grade 3 or 4 and you want to transfer him , how do you call ?
A.ICU B-Vascular surgeon C. General surgeon ✅✅

🌹aortic injury and spleen laceration grade 3 or 4 and you want to transfer him , how do
you call = General surgeon ( not vascular )
...........
🍓30 years old male after RTA had a splenectomy, what will be low after hours from the
surgery?A. insulin✅ B. glucose C. vasopressin ‫ الن فيه جزء من البنكرياس الصق في السبليين‬.
.......
🍓 Patient child with fever , abdominal pain and splenomegaly and the spleen is tender
what culture is important
A/Single blood culture B/Multiple blood cultures✅ C/bone marrow aspirate culture
most likely typhoid: Bone morrow as most Sensitive, but invasive so multiple blood
culture better .
…..

🌷If splenectomy > pneumococcal, HIB, meningococcal > given 2 weeks before the
surgery of splenectomy✅✅🌷
‫ و االنفلونزا احتمال كبير جدا بعد عملية ازالة‬meningitis‫ للنيمونيا و ال‬vaccin ‫الن ازالة الطحال يقلل المناعة جدا جدا لو ما اعطيناهم هذا ال‬
‫ سبحان هللا‬spleen ‫ عشان تعرفوا نعمة‬، ‫الطحال يصابوا بهذه العدوى بسهوووولة‬
🌷 Patient 3 days post splenectomy develop fever 38.5 what is the cause of bacteremia?
A- UTI ✅ ✔ B- Peritoneal transfer N.B: fever day 3-5 post op is UTI.

-Pt postoperative day 3 has gram negative bacteremia how it rech to blood ?
A-translocation B-UTI C- gut

Alcoholic present with hematemesis. Imaging shows splenic vein thrombosis.


Management? ○Splenectomy✅

‫ هذا العالج الوحيد‬. Splenectomy ‫ على طول اكتب العالج‬splenic vien thrombosi ‫📣اول ما نشوف كلمة‬
. ‫لهذه الحالة‬

1hernia
🌹 Woman in her fifties asymptomatic has a small femoral hernia, what is the best management:?
A. Observation B. Open with mesh C. Simple open D. Laparoscopic repair
*Femoral hernia*= We prefer to repair a femoral hernia *laparoscopically* because of its ease of
access. Old people > femoral we do open
….
🌹 Solder with bilateral inguinal hernia ?
A. Lap w mesh B. Open w mesh C. Open w/o mesh
…..
🌹30 year-old has a large indirect inguinal hernia sac that is reducible with mild effort. Bowel sounds are
heard in the hernia sac. This hernia is the result of which of the following?
A. A defect in the abdominal wall with protrusion of abdominal contents through
the internal inguinal ring, canal and external ring
B. A traumatic defect in the abdominal wall
C. A congenital defect in the abdominal wall allowing protrusion of abdominal
Contents through the inguinal canal
D. A defect in the abdominal wall caused by constipation

🌹Pt 56 c/ o sudden RLQ pain after lifting heavy object ; and mass in RLQ ; Cough negative ; abd muscle
tense even with clenching . He is on anticogulation due to A fib . What the most apporpieate mgx ? ^
here I am thinking of Rectus sheath hematomea ==A. angioempolizion and .. forget . B. rest and anaglesic

🌹Man developed rt abd swelling after lifting heavy object, painful irreducible , no
cough impulse ..dx? A. Ventral hernia B. hematoma

🌹hernia, how to manage:


A- laparoscopic repair with mesh ✅✅. B- open repair with mesh + same options without
mesh!

🌹pt did herinal repair with mesh 12y ago and came by obstruction sign and no sign hernia
repair ?A. Adhesion B. Late onset crhons
‫اي عملية تكون في البطن ومر عليها ( سنوات) من اهم المضاعفات الي كصير تحصل لهم هو وجود التصاقات دتخل‬
‫البطن هذه االلتصاقات تسبب انسداد في االمعاء‬
🌹What type of mesh used in ventral wall hernia? A. Subly B. Onlay C. Urblay
🌹 40 years old man underwent open hernia relain and 2 weeks later presented
with tenderness at the site of hernia repair and severe parasthesia/numbness/tingling
around his thigh that went down his leg. Your management?
A. remove mesh staples. B. Neurectomy and mesh removal. D.NSAID
....
🌹 soldier needs to undergo bilateral hernia repair. So what’s your management:
A. open with mesh B. Lap with mesh

🌹 Pt with Reducable hernia comes with intestinal obstruction manifestation and


on exam was redness and on Xray : multiple air fluid level =
A. incarcerated (very sever pain + nusia+ vomting + herniated tissue becomes trapped and
cannot easily be moved back into place
B.strangulated(come with shock)
C. obstructed(Bez say Reducable )✅
...
.....
🌹60 yo pt Post ventral hernia surgery with mesh 6yrs ago pt came with abdominal
distention and on radiology cut off sign and target sign and soft tissue mass
A. late onset crohns B. adhesions with mesh C. small bowel cancer (not large intestin).
...
🌹Elderly with signs of heart failure (lung crpeteaion, lower limb edema , raised jvp) has a
small ventral hernia booked for elective surgery :
A. do open hernial repair as booked B. do lap hernial repair as booked
C. wait until strangulation or obstruction D- delay
....
🌹 t for elective ventral hernia repair, HTN and DM, on examination high JVP, LL edema,
basal crepitation, your management:
A. open surgery B. lap with mesh C. delay operation until addressing current symptoms

🌹 65 year old patient came for elective surgery for ventral hernia , in examination he has
bilateral crickels and lower limb edema , what to do to him?
1- do laproscobic surgery as reschedule 2- do open surgery as reschedule
3- obtimate his symbtoms and surgery late✅✅
4- he is old and asymptomatic don't need surgery

🌹 Patient with femoral small hernia asymptomatic =Observe without surgery
....
🌹8 month infant with right inflamed red hemi scrotum, on examination there was red firm
irreducible painful scrotal swelling which is extended to to left inguinal region. Left testis
couldn't be palpated. What is the diagnosis? A. Torsion appendix testis. B. testicular
torsion c- epididymo orchitis. D. incarcerated inguinal hernia!@
🌹Inguinal intractable pain with hyperaesthesia with pain radiating to thigh 3 weeks post
inguinal hernia with mesh, upon physical exam, no recurrence or surgical site infection,
what's the mx:
A- Nerve block B- Systemic Anti-Inflammatory C- Neurectomy with mesh removal
D- mesh removal
....
🌹 after lap hernia repair with mesh, wound draining pus, your management:
A- explore wound + remove mesh B- put surgical drain C- something + daily dressing ✅

🌷patient has persistent pain after hernia repair, what to do?NSAIDS then nerve block ✅🌹

• 45-year-old female developed hernia located inferior and lateral to pubic tubercle. Type?
○ Femoral✅ ○Obturator. ○ Direct inguinal. Indirect inguinal
-- ----- ----- -----
🌷Patient came after 5days of hernial repair complain of inguinal mass , there is no
erythema , no tender and not associated with cough , what is most likely the cause :!@
A) Serosa✅ ‫ لكن ايش معناها ماعرفت‬، 9‫ الن هي اصحهم‬9‫اخترناها‬
B) Hematoma ‫ مافي عالمات لها‬C) wound infection 9‫مافي حرارة او الم او احمرار‬
D) recurrent hernia ‫مايطلع مع الكحة‬
...... ....

-🌷Surgery for hernia w/t mesh, complains of neuropain radiates to thigh, he’s already on
analgesia and once he stop it pain come back, not improve next?
A- NSAIDs. B- Nerve block. C- Remove mesh. D- Remove stapes of mesh

Persistence pain after hernia repair & he’s already on analgesia and once he stop it pain
come back, not improve so we go to nerve block.
.... ......
🍒 Surgery for hernia w/t mesh .., after 3 weeks came to ER c/o pain and discharge x-ray
shows 3x3 cm collection of fluid .. high WBC .. NEXT?
A- Open and remove mesh B- I don’t think there was drainage ?!
if there’s collection we don’t remove the mesh. 1st if there’s collection we aspirate or if
open wound we do dressing, along with IV abx Removal of the mesh is the last step
either in abdominal hernia or inguinal
........
🍒 Bilateral inguinal hernia, how to manage:
A. lap repair with mesh✅ B. open repair with mesh C. lap repair without mesh
….
underwent hernia repair for right inguinal hernia, presented now complaining of that
ipsilateral testicular size is decreased after hernia repair. Dx?
A. Testicular artery occlusion. B. tight mesh C. pampiniform plexus compression✅✅
pampiniform plexus compression= lump in one of your testicles+ swelling in scrotum.
…..
varicocele is an enlargement of the veins that transport oxygen-depleted blood away from
the testicle.is believed to develop as a result of dilatation and tortuosity of veins of
the pampiniform plexus 
..... .......
🌹Lower limb paresthesia after hernia repair with mesh ?
A. nerve block ✅. B. remove mesh C. neurectomy ........
.....
🌹Elderly pt e hx ( .. months) of open hernial repair operation with mesh , complaining of
groin swelling reach scrotum +ve cough impulse how to manage ? A. Lap with mesh ✅.
........
🌹 Pedia no family with inguinal hernia, pedia surgeon advise treatment
A.Refer to police B.Refer to social worker C. Ask ethical comity ✅🌹D.Refuse
........
🌹60 yrs old female with femoral hernia, asymptomatic, accidental finding mx?
A.observation B.open C. open with mesh . D.laparoscopic !@
.. .
🌹Old man came to you with Inguinal reducible hernia, and he was advised to do hernia
repair, and PE exam was normal he is asymptomatic what will you do:no surgical treatment
...
🌹Case of hiatus herni' and ask about immediate management: Lifestyle modific'tion ✅✅
….
🌹Making a few lifestyle changes may help control the symptoms and signs caused by
a hiatal hernia. Try to: Eat several smaller meals throughout the day rather than a few large
meals. Avoid foods that trigger heartburn, such as fatty or fried foods, tomato sauce,
alcohol, chocolate, mint, garlic, onion, and caffeine

🌹Pt do hernia repair "not sure about the type" then he present with mass in the inguinal
hernia firm, regular, transpulsation "there was no Erythema or tender"and no fever :
A. saphenous B. psudoanyresm✅✅ C. Abscess
…….
#Pseudoaneurysm = A complication of laparoscopic inguinal hernia repair
…..
🌹year-old weight lifter developed abdominal hernia that is irreducible, tender. Imaging
shows air-fluid levels in small bowel and no free air in large bowel. Type?
A- Irreducible. B- Incarcerated✅✅. C- Strangulated
🌹incarcerated hernia is a part of the intestine becomes trapped in the sac of a hernia—
the bulge of soft tissue that pushes through a weak spot in the abdominal wall. If part of
the intestine is trapped, stool may not be able to pass through the intestine.

🌹strangulated hernia occurs when the blood supply to the herniated tissue has been cut
off. This strangulated tissue can release toxins and infection into the bloodstream, which
could lead to sepsis or death. Strangulated hernias are medical emergencies.
An incarcerated hernia occurs when herniated tissue becomes trapped and cannot easily be
moved back into place. An incarcerated hernia can lead to a bowel obstruction or
strangulation. In other words, a strangulated hernia cannot cause an incarcerated hernia.
Incarcerated hernias are not life-threatening
‫ والي يعمل انسداد هو‬obstruction ‫ معناها حصل انسداد في االمعاء‬air-fluid levels in small bowel ‫قالك‬
‫ بينما لو قالك‬، ‫ يتحرك فيحصل انسدداد‬9‫ ينحبس ومايقدر‬stool‫ الن يقولك االمعاء تكون محصورة فال‬Incarcerated
‫ هذه الكلمة معناها كانه خنقت‬، strangulation ‫ وعنده حرارة عالية هنا نفكر في‬sepsis shock ‫المريض دخل في‬
. ‫ وغالبا مايجيك باالعراض الي في السيناريوا‬، ‫ منعت الدم تماما منه‬، ‫المكان وماخليت يوصله دم ابدا‬
‫ ان‬. ‫ الحل الصحيح الي كتبته االن‬9‫ فخذوا‬، ‫ السؤوال هذا ممكن اكون وضعته سابقا وكان الحل يختلف عن هذا‬: ‫تنبيه‬
‫ باالجابة‬9‫شاء هللا تكونوا اقتنعتوا‬
. . . .. .
4 y/o came to clinic w/t umbilical hernia without any Sx .., next:
A- Reassure ✅✅✅. B- Surgery C- Band

18 year old healthy male was playing baseball and suddenly he felt abdominal pain. On
examination he has para-umbilical mass. His vital signs Bp 100/76 RR 30 HR 100 O2 sat.
95% oxygen mask. What is your next step in management?
A. Abdominal US. B. CT scan. C. Erect CXR. D. Reassurance and send home
** Seems umbilical hernia incarceration due to sudden pain and hypotension

1tumer
🌹Sclerotic lesion in distal femur: A. osteosarcoma B. chondrosarcoma C. ewing’s sacroma

🌹old with weight loss epigastric pain with supraclav lymph node : A. gastric cancer
🌹Patient w incidental finding of macroadenoma of pituitary during evaluation of
her headaches, physical and medical hx was unremarkable, what’s the best
next step? A. - ant pituitary hormone scan B. - Referral to neurosurgery

🌹Mid-thigh lump with normal overlying skin and +ve fluctuating test. diagnosis?
A. Lipoma B. Sarcoma C. Aneurysm D. Sebaceous cyst
...
🌹Patient came the weight loss and abdominal distention, Ct found: Soft tissue mass retro-
peritonal , and multiply hypo-dense or hyper (forget ) liver lesions :
A- liposarcome B- germ cell tumor C- lymphosarcoma

🌹what makes lymphadenopathy malignant more than infectious:


A. less than 5 y B. duration less than 4 weeks C. cervical lymphadenopathy
D. supraclavicular lymphadenopathy

🌹Old man has progressive dysphagia e mass in mid esophagus ,chest x-ray normal . What
is the most likely cause A. adenocarcinoman B. SSC C. Lymphoma
SCC > middle and upper Adeno > lower

🌹Man with high grade dysplasia of the esophagus. How will you manage?
Refer him for surgery

🌹Smoker with chronic dyspepsia, endoscopy biopsy is squamous cell with


low grade dysplasia, what is your management?
A. PPI B. Refer for distal Esophageal resection C. Observe

🌹 Most common/important risk factor for esophageal cancer is:


B. Heavy smoking D. Barrett's esophagus

🌹Low grade dysplasia barrett's esophagus, what is the management:


A. Pantoprazole B. ranitidine
....

🌹what is the most common small bowel cancer ? Adenocarcinoma

🌹Epigastric pain diarrhea and peptic ulcer with + secretin ? A. Gastrinoma B. Carcinoid

🌹klatskin tumor= is cholangiocarcinoma located at bifurcation of common hepatic duct =


CA 19-9.

🍓Pt known case of lung cancer and had history of epilepsy and on medication,
medication is contraindicantion to this pt ?
A _ nicotine replacement medication B_ Bupropion = epilepsy

🍓colon cancer surgery after that he had 7ml/h urine output for 8 hours , blood pressure
and heart rate was normal what is the best management :
A. 500 NS challenge✅✅. B. diuretic. C. Inotropes

🍓23 ylo women Typical IBS scenario her uncle in 60s has colon cancer, all blood
labs normal except mild low Hb 11 (12-15)!!!! Ask about Dx?
A. UC. B. Colon cancer✅. C. IBD

🌹Adult patient came to a clinic with a small mass on her forearm since childhood. Asking
about the management: A.Laser B.Excision C.follow up ✅ D.radiotherapy

🌹 Pt hypertensive have low abdominal pain they do adrenal hormone workup (normal)
and ct abdomen show hypoechoic and lipo.. A.Nothing to do B. Biopsy ✅.
C.Adrenalectomy
Indications for adrenalectomy may seem straightforward at first. However, with the
increasing rates of diagnosis of asymptomatic and likely benign adrenal masses,
disagreements regarding optimal management remain. [11]
Masses larger than 6 cm have a rate of adrenocortical carcinoma of 25% and should be
managed surgically. [9]  Masses smaller than 4 cm may be safely observed because their
rate of adrenocortical carcinoma is 2%. [9]
🌹ct abdomen show hypoechoic on adrenal gland = biopsy ‫حتى نتاكد هل هو خبيث او ال‬

🌹if mass more than 6 cm do Adrenalectomy if less than 4 cm , usully observantion . 🌹

🍒adrenal tumor with high cortisol symptoms next management :


A. Surgical resectio ✅🍒 B. Chemotherapy C. Radiation

🌹 Old man came with vague abdominal pain with 20cm×20cm mass , finding in u/s
multiple hypoechoic masses ( or nodule ? ) What is the diagnosis :
A) libosarcoma ✅ B) lymphosarcoma C&D) can't remember them
🌹abdominal pain with 20cm×20cm mass , finding in u/s multiple hypoechoic masses
== libosarcoma

🌹Sarcoma metastasis to ? lung

🌹Sarcoma Biopsy most Diagnostic test: A. Incisional biopsy B. Excisional C. Needle core
‫ = ساركوما = سكين = تقطيع‬incistion
🌹Sarcoma "no incisional in choices "? Core needle biopsy

● Most important about retroperitoneal sarcoma :


A. compress over the organs . B. Met to the near organs

🌹 Sarcom' of the thigh. Whats the best imaging for staging work up?
- CT ✅ sarcoma = c.t = s=c

🌹pt with mass 2 cm away from anal verge on proctoscopy it appears cauliflower:
A. Anal cancer B.Chondalymia acuminta ✅

🌹70 year old male Weight loss fatigue proctoscopy shows mass 2 cm from anal verge
cauliflower like friable mass ? A. anal ca B. rectal ca C. colon ca D. condylomata acu

🌹Cancer in body of stomach with no metastasis and no lymph node =


A.gastrectomy B. wide local local excision with clear margins ✅

1Chest tube
🌹by x.ray there is pneumothorax 2 cm : A observanton B- chest tube

🌹Pt with trauma has mild respiratory symptoms, no deviation of trachea, ...etc (all mild
symptoms) , What is the dx? A- tension pneumothorax B- simple pneumothorax
🌹 25 years male involved in RTA in ER he is fully continues and no abnormality in vital signs
admitted for observation and a FAST scan done ,.what do you expect to find
A. Pneumothorax B. major vessel bleeding in the thorax C. peritoneum free fluid

🌹Pt came to ER after MVA with multiple trauma and hypotension, tachycardia, x ray shows
cardiomegaly and sternal fracture , Dx ? hemopericardium ✅

🍓Thoracocentesis , where to insert ( which rib) ? 8-9


Thoracocentesis: between 9 & 10 ribs midaxillary line. Needle thoracotomy: 2nd rib space
....

🌹Elderly smoker with progressive SOB, on CXR massive Rt pleural effusion what's your next
step? A. Thoracocentesis✅ B. Intercostal tube under water seal
. Thoracocentesis ‫ نطلع السوائل ب‬plural effustion ‫عنده‬

🌹 lateral pleural effusion ask about site of needle insertion ? 8-10🌹Latral = 10

🌹Case about pleural effusion on chest tube after a while there was blood in the water
under seal (Hemorrhagic Pleural Effusions and Hemothorax), how to manage?
A-Thoracentesis B-Tube thoracostomy (chest tube). ✅ ✔ C-thoracotomy

Hemorrhagic pleural effusion ttt usually by chest tube, if the drain produced > 1.5 L on
insertion *or* > 1 L within the first hour then we shift to thoracotomy..
....... ....... ....
🌹Case of fall truama Increase jvp Decrease air entry (Low bp) (tackycardic) No more infos!
> low bp mostly bleeding so hemothorax Tx? A. Thoracostomy ✅ B.Thoracotomy
C.Unrelated
……
Patient had fall 50 meter, absent lung sound on the right side, CXR was
provided but not clear: A. Intubate B. Thoracostomy
🌹pt fall from hight and Vitaly insatable and Pic of Cxr with symptoms and signs of
hemothorx and ask 'bout ttt : Chest tube ✅✅. Thor'cotomy. Abx Anelgesics
chest tube ‫ على طول عالجها‬hemothorx ‫معروف اول مانشوف‬
....... ....... ....
🌹A man presented with a gun shot wound in his chest that he received an hour ago, and
you’ve done needle thoracentesis. 15 mins later 100 ml of blood is drained, what is the next
most appropriate action? A. needle thoracentesis B. Thoracotomy
…..
🌹 Patient who had had multiple traumas in MVA, they mention presence of free
fluid in the abdomen and spleen laceration + thoracic aortic rupture. Next
A. Thoracotomy B. Laparotom

🍓RTA , presented with slow breath and decrease breath sound left lower lobe lung xray
picture showed infiltrate in lower left lobe and he has lower limb fracture His blood
pressure and HR was normal ,What is your management
A. thoracostomy tube✅ B. thoracotomy C. angiography with stent

🌹young adult was playing football with his friend, the ball strongly hit his chest, after
sometimes he had SOB, what is the diagnosis?
A-Pneumothorax B-Pulmonary Contusion ✅ ✔ C-Tension Pneumothorax
. .. . . .. . . .
🍓RTA pt in hospital with limited facilities Found to have tension pneumothorax and
fracture femur You insert chest tube And immobilize the fracture Pt stable Decided
to transfer him To tertiary hospital , On the way to ambulance He suddenly collapsed, What
is appropriate next step? A. Intubate. B. Check chest tube insertion and functioning!@

🌹Patient involved in RTA was transmitted to near by limited facilities hospital and the
physician decided to refer him to advanced facilities hospital Patient is unconscious:What
is the most important thing he should be maintain:-intubation ✅

🌹left hemithorax : A. Chest tube of left hemithorax ✅ B. Needle decompression

🌹A known Copd patient presented with sob but was not in distress vitals stable
examination was normal.. no tracheal deviation and equal air entry on both sides..cxr
shows a 2cm pneumothorax.. wat will u do
A. Oxygen and observation (bez 2cm) B. Chest tube C. Pleurodesis D. Needle
decompression

🌹Young patient comes with dyspnea and chest pain after a long flight. Upon examination:
patient is tall, thin. CXR finding: pneumothorax.management:
A. Thoracocentesis (with effustion) B. Thoracostomy tube C. Conservative management❌
...
🌹Patient with pneumothorax tube thoracostomy inserted after 15 min water seal bottle
is filled with blood PB: 90/60 HR: 100 Rr:22 =
A- thoracotomy. B- thoracostomy tube ( it's already in the q I don't know why it's in the choices ) C- ct
abdomen

*Patinet who was hit in the chest while playing football, after it SOB, tracheal shift ,
hypotension and raised JVP , diagnosis : -Tension pneumothorax ✅

1intupation
Child with CXR finding hyperlucency of upper left zone with compression of adjacent
lobe with shift to the right, what to do?
A. thoracotomy B. Thoracostomy C. Intubation

Pt with small stab wound in anterior aspect of neck. On exam he is Alert


and conscious but in labs oxygen sat 82%. What to do
A. Oxygen mask B. Cricothyroidotomy C. Endotracheal intubation

🌹Proper ET position how to chick :


A. co2 monitor (my choice) B. chest movement. C. easy insertion.

🌹RTA , facial fractures , unconscious, what is opportunity in mx?


A. Cervical spine. B. airway

🌹Multi trauma pt, with many facial fractures, next step?


A. secure airway B. C-collar

🍓man with facial bones fractures after RTA, in the ER what is the first thing to do?
A. Airway✅ B. Call neurosurgeon

🍓 man after accident and resuscitation in small hospital u need to transfer to another
hospital after stabilization, it is 30 mins far. on xray u see fracture of 2-5 left ribs. no
pneumothorax what will u do?
a. intubate✅ b. call the other hospital to inform the surgeon on call
c. chest tube insertion

🍓A scenario of a woman trapped in house that was burning and lost consciousness, Some
labs were given which I can't recall. How do you manage? intubation🍓

🍓Patient have a car crash , he had an injury in the neck , the is tissue visible , how to
manage him ? A. Intubation. B. Cricothyroectomy. C. O2 mask✅

🌹 Pt came with trauma and from the explanation he have “flail chest” what to do?
A. iv fluid resuscitation ✅ B. Intubation

🌹 Case of flail chest ask about initial thing to do? he was stable
A. IV fluid B. Assistant ventilation
....

🌹40 something old pt came with anterior thigh stab wound 1 cm with active bleeding and
unconscious, most appropriate Mx (I’m not sure if it was written most next appropriate mx or not
but i think it was not written)
A. apply tourniquet B.Blood transfusion/ IV fluid C. intubation ✅✅

🌹patient with trauma to face with fracture mandible resuscitation with?


A.laryngeal mask B. cricothyroidotomy ✅✅ C.Oral endotracheal tube
..
🌹Mva with mandible fracture = Cricothyrotomy

🌹After an accident 18 yrs fracture of mandible and maxilla oropharyngeal tube /
tracheostomyhow we improve breathing? cricothyroidotomy

🌹Absolute contraindications for NG intubation include the following:


1_Severe midface trauma. 2_ Recent nasal surgery.
🌹Chemical product burn ttt ulcer in back = A. wash with water for 30 mints ✅
B.intubation C.dressing

3_Burn on the face , GCS 15 he was alert and speaking. the two striking choosies were=
a. Elective intubation✅ b. ICU admission for 24 hrs
‫ربط = اغلب عمليات الوجه تكون اختيارية‬

Face burns expose patients to a higher respiratory risk, and early prophylactic intubation
before they enter the burn unit might be life-saving✅

Pt had chest trauma nad 3⁄4 rib fracture paradoxical chest


movement? ‫يعني فيه كسر عظم الصدر نص مكسور ونص ال‬
•analgesia
•intubation and ventilation are usually required or high concentration oxygen by mask...
. ...... ..... ......
🍒 Patient after RTA was conscious GCS 15 then suddenly collapsed u see a temporal
fracture what caused his loss of consciousness
a. subdural hematoma b. epidural hematoma✅ c. fracture base of the skull

🌹Patient after RTA was conscious GCS 15 then suddenly collapsed u see a temporal
fracture what caused his loss of consciousness?
A. subdural hematoma B. epidural hematoma C. fracture base of the skull

🍒2nd degrea burn of anterior lower limbs..wt is70 kg ..fluid replacement


A-Normal saline 200ml/hr
B-Ringer lactate 1 boluse then 120ml/hr
C-Normal saline 5 L in the 1st hr then
D-Ringer lactate 2.5 L through the 1st 6hrs then 2.5 L in the next 6 hrs

🌹Young male with bilateral anterior without post lower limbs 2nd degree burn who
weighs 70 kgs Asks about fluid replacement according to parkland formula:
A. 2.5L to be given in the first 8 hrs and 2.5 L in 16 hrs
B. 5L in first 6 and in 16 hrs
‫ نستعمل‬parkland form=
4ml x TBSA (%) x body weight (kg)
‫ الثابت الي هو‬9‫ نضربها بالرقم‬70 ‫ نضربها بالوزن الي هو‬18 = ‫ االثنين مع بعض‬٩ = ‫ الوحدة فيهم‬lower limp‫ال‬
‫ ساعة فتساوي‬١٦ ‫ ساعات والجزء الثاني في‬٨ ‫ نقسمها على اثنين عشان نعطي اول جزء في اول‬5040 ‫ تساوي‬4
‫ ساعات‬٨ ‫ يعني اثنين لتر في اول‬2500

🌹pt with 20% back burn which indicate good response== Urine output 0.9/kg/hr

🌹Burn case black soot over nostrils and mouth 40% carboxyhemoglobin.
(Carbon monoxide toxicity):
A. Hyperbaric oxygen. ‫ نعطيه اوكسجين كثييير حتى يطلع‬co
B. Intubation and ventilation with 100% o2. C. Carbonic anhydrase inhibitors.

1skin
● Dibetic has progressive painful lump on back of neck with multiple openings :
A. cellulitis B. Abscess C. Furuncle D. carbuncle ‫مرض السكر = كرب‬
🌹diabetic patients with unhealed ulcer for 5 years and biopsy showed
pseudoepitheliomatous hyperplasia.
A. Biopsy B. Debridement C. -ve pressure dressing

🌹Patient with liver mass Ct first stage filling periphery late washout centrally :
A-Hepatoma B-hemangioma C-metastatic

🍒-25yr old girl with skin lesions since childhood= -Laser -Excision -Observer✅

🍒Open fracture came 4 days later to ER with signs of infection ( blue, necrosis) what's the
most common organism ?
clos ‫ هو‬necrosis ‫البكتيريا الي تيجي بسبب الجرح المفتوح وعدم اهتمام بنظافته وفيه عالماات‬
A-clostridium✅🍒 B- Staph aureus C- Actinomyces israelii

🌹 case about gas gangrene due to open leg fracture infected by? Clostridium perfringens

🌹Case of melanoma = Excsion with safety margins

🍓Patient with limp sarcoma confirm the diagnosis :


A. incision biopsy✅. B. core biopsy. C. fNA biopsy
Sarcoma > incisional biobsy
Melanoma > excisional biobsy

🌹Skin lesion irregular and dark over the knee


A.bunch biopsy B.Chemo C.Radiation D. excisional biopsy ✅

🌹 Old patient with 3*4 dark elevated skin lesion at the ant of his thigh, what’s the
next appropriate step? A.excision B.Punch biopsy C.Chemotherapy

🌹 girl complains of rash in perianal area with pic provided (typically the same) Dx:
molluscum contagiosum
‫ شكلها عندكم‬9‫ جبت لكم كذا صورة عشان يتركز‬، ‫ شكلها مثل الحبوب الصغيرة‬.
‫ = ربط‬molluscum = ‫ كلمة‬moll ‫ معناها حبيبات صغيرة و‬contag ‫ المول سار ملوث بال‬، ‫معناها ملوثة‬..

. .. . . . . .
🌹ptient fall on his leg with skin loss vascular structure appear and ask 'bout ttt :
Debritment and 2ry skin graft ✅✅ Debritment 'nd prim'ry closure Abx
‫ معناها الجلد ماهيقدر يقفل على هذا الجرح الن جدا عكيق فالزم ناخذ‬vascular appear ‫ في الجروح ؟ اذا‬skin graft ‫متى نلتجا الى‬
. .. . ... ‫ ونغطي فيها الجرح‬skin graft

🌹Run over case, exposed neurovascular, most appropriate mx?


A.something then primary closure B.debridement + vacuum C. debridement + graft ✅✅

🌹patient with post operetive 'abdominal destintion and small and large bowel
obstruction and no fever 'nd a'bdomen not tender :
Hypo Mg Hypo CL Hypo k

: ‫ يعمل‬k = hypokalemia ‫طيب من اعراض انخفاض‬

constipation or abdominal distention, muscle weekness .


. . .. . .
🍓 What's the cause of pain in compartment syndrome?
• Nerve ischemia✅✅ • Muscle enlargement • Bone ischemia

🍓55 pt present to the ER after a car accident, there's a pic of CXR show aortic dissection
and lung contusion , what’s next step :A. chest tube. B. aortic repair and stent✅

🍓 Patient lost sensation over medial side of leg but motor function is intact.
Which nerve is injured?A-Obturator B-Femoral C-Saphenous✅✅ D-Sciatic
🌹lost sensation over = Mid thigh = obturator
🌹lost sensation over = medial side of leg = saphenous

1 Diaphragmatic
🌹Child Diaphragmatic hernia first thing you do? NGT first thing to do after stabilization 🌹
Newborn just delivered having diaphragmatic hernia , what’s the first thing to do: NGT
Diaphragmatic Hernia
Treatment
1_ immediate intubation required at birth: DO NOT bag mask ventilate because air will
enter stomach and further compress lungs.
2_ place large bore orogastric tube to decompress bowel.
3_ initial stabilization and management of pulmonary hypoplasia = hemodynamic support
and surgery when stable.
Clinical Presentation
• respiratory distress, cyanosis
• scaphoid abdomen and barrel-shaped chest
• affected side dull to percussion and breath sounds absent, may hear bowel sounds
instead
• often associated with other anomalies (cardiovascular, CNS, chromosomal abnormalities)
• CXR: bowel loops in thorax (usually left side), displaced mediastinum
.........
Weber in right and bilateral + rinne test?
A-conductive hearing loss B-right schwannoma C- right presbycusis D/ Left sensory ✅
Positive rinne = normal or sensorineural Weber shift to right so it’s either right conductive
or left sensorineural

🍓Colon cancer common metastasis to? A. Liver✅ B. Lung


‫ربط = القولون والكبد = كلهم من اجزاء الجهاز الهضمي مسؤوليين عن االكل‬

🌹Patient complained of Rt groin pain for 2 weeks and exacerbated by exercise,


physical examination showed normal groin and genitalia, next step?
A. CT abdomen B. MRI abdomen C. Us D. Re-evaluate after 2 wks
‫قالك انه يزيد مع الرياضة ف غالبا حصل للعضلة شد‬
overstretching the muscles during sport.t'
‫مايحتاج اشعة‬

🌹Pt diagnosed with small cell lung cance, presented with dehydration, serum osmo low,
urine osmo high. ‫ هذه اعراض الجفاف‬. Ttt:
A. 5% dextrose B. Normal saline C. Hypotonic saline

🌹Male have problem initiating urination , bladder fell un empty after , no dribbling or
urge felling , type of incontinence ?A. Overflow B. Reflex C. Urge D. Sterss

🌹 Bradycardia (40 bpm) during laparoscopic procedure Is due to :


A. Cold gas insufflation B. Rapid expansion of parietal peritoneum C. Increase venous return

🌹60 years old patient come with sudden onset of upper abdominal pain after a
few bouts of vomiting. Examination confirme sick patient with tenderness in
epigastrium and supraclavicular subconscious emphysema . What’s Dx ?
A. esophagitis B. acute gastritis C. perforated peptic ulcer D. boerhaav’s syndrome(is
spontaneous perforation of the esophagus )
.... subconscious emphysema 9‫الن قالكم‬
🌹Pt k/c of CKD undergo apendecyomy K was 6.5 with tened T wave ??
Calcium gluconate

🌹 Pt with dysuria and cloudy urine with bubbling for 2 months, hx of recurrent
left iliac fossa pain for the past 2 years ! Colonoscopy: no diverticulosis or
polyp Cystoscopy: erythema on dome of the bladder. Most likely diagnosis:
(9‫ )كذا مكتوب بالزبط‬disease Diverticular.
B. sq. Cell bladder cancer C. transitional cell bladder cancer

🌹Female with history of regurgitation and heartburn.. all conservative management failed
include PPI ..endoscopy done showed erythema and erosion suggest esophagitis
A. manometry B. 24 h PH monitoring C. Lifestyle modification

🌹45yo female has dysphagia with liquids only retrosternal pain and regurgitation
of food, what is the test of high diagnostic value?
A. UGD. B. . Barium swallow. C. Low esophageal manometry. D. CT with contrast

🌹Patient had a hyperextension trauma, complains of distal phalanx pain and tenderness in
the volar aspect, he also feels tenderness in the palm, what's the dx:
A- Rupture of flexor profundus B- Rupture of flexor superficialis
C- intra-articular fracture of proximal phalanx D- extra-articular fracture of distal phalanx
Memorize them as follow :
P roximal > superficial. D istal > dee

🌹50 male came with 4 time hematochezia , no weight loss , no abdominal pain dx:
A- diverticulosis B- internal piles C- cecum cancer

🌹patient on TPN will increased INR what to give= vit K

🌹Perforated appendicitis post laparotomy, presented with fever, abdominal pain CT


done showed foreign material , reexploration done, gauze was found intraabdominal ،
What to do?
-Issue complaint against assistant
-Call your lawyer and report the incident
-((apologize to the patient and tell her what happened ✅))
……
● Patient with jerky carotid? A. Hypertrophic cardio myopathy
…..
● patient came with melena and they did upper gi endoscopy and found 1cm
bleeding duodenal ulcer and they ask about the duration of ppi ?
A. oral ppi B. iv ppi for 24 hours then convert to oral
C. iv ppi for 72 hours then convert to oral
Answer is C
● Patient post graham patch surgery, what is the most appropriate management?
A. High dose PPI B. IV ppi for 24 hours followed by oral PPI
C. IV ppi for 48 hours followed by oral PPI
…..
6 yrs old, female presented with nausea and recurrent vomiting of every intake , pt
became lethargy with decreased skin turgor and cry with tears.Investigation
revealed Normal investigations apart from :High Na Low glucose
What is the best next management :
A. isotonic saline 20ml/kg B. D10% with 1/2 saline C. ORS
Hypovolemic Hypernatremia First correct the volume with half normal saline then once the
patient euvolemic shift to slow D5W infusion.
….
Old man with slow progression dysphagia, came with vomiting without blood, abd
pain associate with heartburn at eating. Abd ex normal DRE revealed slightly
change stool with blood. What is suspected diagnosis :
A. PUD. B. CA esophagus. C. CA rectum
….
Young female c/o fatigue and jaundice high ALP and high bilirubin Us: no any finding MRCP :
multiple stricture Next step to *confirm* Dx ?
A. Liver biopsy B. Antinuclear antibody C. Repeat US D. Colonoscopy
…..
Case of established diagnosis of biliary sludge of postoperative sleeve pt with jaundice and US
report of biliary sludge. What is the management?
A. Expectant. B. Endoscopic sphincterotomy C. Cholecystectomy
….
What is the Pathophysiology of (case of) bacterial peritonitis due to E. Coli
A. translocation of bacteria B. bowel Perforation
….
young male k/c of ulcerative colitis c/o bloody diarrhea, abd pain and S/s
of intestinal obstruction X-ray shows: megacolon What is appropriate management:
A. CT abdomen B. Ileocolectomy with. Answer is : IV steroids
….
65 male patient present with SOB and generalized fatiguability, On Ex: non tender mass in right
iliac fossa, Hemoglobin: low What the most important step in Mx:
A. colonoscopy ‫اشتباه ب ورم في القولون‬B. Percutaneous biopsy C. Abdominal CT D. Ultrasound
…..
Surgeon at elective laparoscopic cholecystectomy find large mass in
Stomach= A. stop oper B. cholecystectomy only C. chlesrecomy and resection mass
….
Patient underwent esophageal dilation. After 12h patient had symptoms and they did
gastrografin which showed leak. Temperature is 39. BP 100/50*=
A. Stent B. Esophagectomy C. Surgical drainage!@
(esophageal rupture treat by Tube thoracostomy (drainage with a chest tube or operative
drainage alone)

● neck mass LN FBA normal thyroid cell dx A. Follicular ca C. Papillary ca D. Ectopic

● *Sacral ulcer, skin necrosis and exposed subcutaneous fat manage?*
A. Debridement with dressing B. Debridement with primary closure C. Debridement with
secondary closure graft

*Penetrating stab wound. Patient is stable. What is the next step*
A. US. B. CT
….
● Case of cystic fibrosis Wit abd pain , bloating , pale stool Abd scraled or curve
A. Chronic pancreatitis * B. Acute pancreatitis C. Pseudocyst

Patient claudication long case, then after full treatment with thrombolysis, patient is risk for
what while staying in hospital? A. Stroke B. PE C. MI

Scenario Liver mass 3 cm next with splenomegaly
A. AFP B. triphasic CT * C. Biopsy
..
Epilepsy pt posterior dislocation, which joint? A. Subacromial

Tamoxifen for breast ca the came with abnormal uterine bleeding or something
like that?? A. Endometrial ca B. Endometriosis

Patient has crushed by a car came with 10cm deep laceration of his thigh with
continues bleeding what you will do
A. tourniquet B. compress at the site of the injury C. compress the femoral area
….
Same scenario but with out mentioning of the depth and the patient were
Hypotensive A. normal saline B. compress the site of the laceration C.
Meigs syndrome How to confirm the Dx?
A. Tissue histopathology B. Tumor marker C. Mri D. Or Us I think

Old pt after hysterectomy what may cause her to stay more in the hospital? Her
UO is 35ml per hr = A. Decrease oral fluid intake B. Decrease urine output
N.B :35ml/h = more than 800 per days Oligouria < 500m

Patient has hx of rta since 2 weeks came today with sob the chest x ray has
shown haemothorax you have put him on chest tube and want to transfer him
to higher center while he was in the ambulance the pulse oximetry drop
suddenly what to do
A. continue the journey to the higher center B. check the potency of the chest tube
C. triphasic chest intubation
…..
Sigmoid volvulus case (s&s), ttt: A. total colectomy with ileostomy B. sigmoidoscopy detorsion if
stable
….
Hepatitis B and lesion 6 cm what to do next?
A. TACE Transcatheter arterial chemoembolization) B. ExcisionC. systemic chemo therapy

● 24 years old with history of appendectomy 5 years ago present with abdominal
pain , distintion , vomitting for 3 days Ct scan show obstruction signs And
peritonitis , Whats contraindicated in this patient ?
A. Propofol. B. Ketamine. C. Sevoflorane. D. Nitros Oxide
….
🌹 patient with s&s of bowel obstructions, investigations shows air fluid level and
dilated loop and colon collapse , asking about what drug is C/I?
A. Propofol. B. drug started by *sul* I think. C. nitric oxide


🌹A baby boy came for circumcision in the clinic. The urine was coming from the
mid shaft (I think hypospedias) what to do next?
A. open circumcision B. plastibell circumcision C. circumcision with gomco
D. refer to pediatric surgery

🌹What is associated with Biliary colic?
A. Bilirubin B. Amylase C. Alkaline phosphatase. D*cholecystokinin*

🌹 pt. k/c of crohn's after colon surgery developed severe diarrhea what you will give ?
A. mesalamine. B. cholystaimne
….
🌹 Low grade dysplasia of oesophagus
A. Resection B. Panzole C. Ranitidine
….
🌹Morbid obese for consultation. What is the investigation to determine the
appropriate surgical procedure?
A. US B. CT C. UGI endoscopy D. Barium enema
….
🌹45 YO male underwent sleeve gastrectomy 3 weeks ago , came
complaining of severe abdominal pain, What the next management ?
A. endoscopy. B. NPO. C. exploration. D. forget the choice but something irrelevant
🌹Patient post-surgery came for follow up develop seroma near the wound or superficial
can't remember ( no pain, no erythema, no any sign of inflammation) what your
management is: A. percutaneous drainage. B. daily dressing

🌹55 year old with constipation and distention, on examination there is , CXR
pic with coffee bean appearance, where is the site of obstruction?
A. Ceccal B. Sigmoid

🌹Old pt male presents with sudden severe central abdominal pain he is kc of
cardiomyopathy his vital I think all is normal x -ray normal what is Dx ?
A. mesenteric vascular occlusive B. aortic dissection C. pancreases
..
🌹Euvolemic hyponatremia fluid replacement:
A. Normal saline B. Half normal saline C. Hypertonic saline D- Water Restriction

🌹Post colectomy for colon cancer she is Diabetic, received dextrose and
Insulin for 2 days, then she developed confusion and agitation. Lab :
hypoNA hypoK, urine osmolality normal, serum 270. Most likely cause?
A-Water overload. B-Addison disease. C-SIADH

🌹What of the following pass through deep inguinal ring:
A- Round ligament B- illo-inguinal nerve
….
🌹burn patient and resuscitation done, which of the following reflect a good
resuscitation has been achieved?
A-normalization of heart rate B-normalization of blood pressure
C-Urine out of 0.6ml/kg/h D-central venous pressure 12

December1
🌹Pt with hypertriglyceridemia came epigasttic pain radiated to back, hemodynamicaly unstable and other
syx, ct show dilated lobe in leÿt hypockondrium,lap show: lapase and amylase within normal range
A. pancritits (my answer) B. perÿorated ulcer C. lower lobe pneumonia
NB:Why would pancreatitis presents with normal lipase and amylase?
Because enzymes sensitivity can be *reduced by*:
-late presentation. -hypertriglyceridaemia. -chronic alcoholism

🌹Patient is going to undergo lap chole and incidental found AAA size 4.5
WhAt you are going to do?
A- Do surgery follow up by US B- Do surgery follow up by CTA C-Do CTA before surgery
…..
🌹pt with pelvic pain and ct shows low density adrenal mass lipid rich what is the treatment:
A. lap adrenalectomy
….
🌹15 years old pt going to surgery what to do:
A. both assent from pt and consent ÿrom parent.
🌹 surgeon operate on rt kidney and resident think lt kidney:
A. inform surgen
……
🌹 drowsy despeic child victim on chest trauma with trachea shift to left decrease breath sound
hypotension raised JVP what's Dx: A. tension pneumothorax

🌹 post sleeve gastrectomy pt has change bowel habit what's treatment: reassurance

🌹 surgeon inj CBD during surgery and repair what to do: reassure and tell pt
….
🌹 q. agter appendicitis pt reoperated due to missed gause inside pt
abdomen what to do: A. tell and apologize to pt
….
🌹 mid shaft femur # with 30 degree anterior angulation what is treatment:
A. closed reduction and intramedullary ifxation
……
🌹 pt few days post cholecystectomy developed inspiration chest pain, LUQ pain and tenderness
with dull percussion, tem 38 basal crepitation what is the likely Dx:
A. sup-phrenic abscess. B. lower lobe pneumonia.
…..
🌹Pt with acute cholecystitis, when to do cholecystectomy? Immediate lap chole🌹
‫حل جلوري‬
.....
🌹Case of appendicitis.. how to find concealed appendix? tenia coli🌹
....
🌹Ventral hernia.. where to put the mesh?
sublay mesh reapir ✅🌹
..... ...
🌹 Treatment of anal fissure?lateral internal sphinecterotomy🌹
.......
🌹Pancreatic Psudecyst for 3 wk go for observation? observation for 6 w🌹
........
● 65 Y women went to the clinic. Incidentally diagnosed with small femoral hernia. What’s your management?
A. Observation
B. Open with mesh
C. Open without mesh
D. Laparoscopic repair
Answer is B ( if Lap with mesh , better)
‫هذا حل جلوري‬
..... ......
Pt with hx of hashimoto’s thyroiditis for years. Presents with of thyroid mass around 2x3, what is it?
A. Papillary cancer
B. Thyroid lymphoma✅✅
C. Subacute thyroiditis
....
🌹 Testicular atrophy After inguinal hernia repair== pampiniform plexus occlusion🌹
.......
Comorbid 35 male for bariatric surgery which investigation before procedure?
A. barium enema
B. Ct with contrast
C. Upper GI endoscopy
D. abdominal US✅✅
Pre-bariatric surgery > abdominal US or CXR.
🌹Comorbid 35 male for bariatric surgery which investigation before procedure?US🌹
........
🍒Patient post-op with signs of lower lobe infection and collapse = atelectasis= Treat by = incentive spirometry(is a medical
device used to help patients improve the functioning of their lungs.)🍒🍒
.....
Old patient with retroperitoneal hemorrhage with history of hodgkin
lymphoma whats the diagnosis?
A.Lymphosarcoma
B.Liposarcoma
C.Sarcoma✅
.....
4th day post OP (appendicitis) presented with diffuse abd pain, distension , vomiting and sluggish bowel sounds
A. Abd xray
B. Multiple air/fluid level
C. ileus ( paralytic ileus ) ✅✅
D. adhesions and SBO ‫ له اربع ايام فقط طالع من العملية ما امداه يعمل التصاقات‬،
🍒🍒4th day post OP (appendicitis) with sluggish bowel sounds = ileus ( paralytic ileus ) 🍒🍒
..... ......
🌹 Patient involved in RTA was transmitted to nearby limited facilities hospital and the physician decided to refer him to
advanced facilities hospital Patient is unconscious What is the most important thing he should be maintained : intubation🌹
...
🌹Patient with AAA clinical findings. Most appropriate next laboratory investigation?
A. Amylase✅✅🌹
B. Serum
C. Calcium
D. ABG
....
🌹Best modality to diagnose perforated peptic ulcer? Erect Chest x-ray🌹
...... ....
‫ جلوري‬1 ‫ مراجعة اكتوبر ملف‬.
‫بسم هللا الرحمن الرحيم‬
🍒🍒urine incontinence when cough?
stress incontinence tx: kegel exercise🍒🍒
... .....
🍒🍒Pic of uterus ( mass ) fibroid, Q: what’s the procedure? hysteroscope🍒🍒
..........
stable ectopic pregnancy but live far.= admitte her
..........
Pregnant came because his son has infection, What to give her? .influenza
...........
🍒🍒Pregnant has T2DM and HTN on ACEI, her HBA1c is 8, wants to get pregnant, best next step? . ACEI will Cause
malformation, first defer pregnancy to control the blood sugar🍒🍒
......
🌹 Symptoms of endometriosis management? - laparoscopic exploration🌹
.......
🍒Ectopic pregnancy is a defect in which process = Implantation🌹
A. Implantation✅
B. Ovulation
....... .....
🍒 skin pigmentation over back of neck and axillary region in female=
Acanthosis nigrican🍒
......... .....
🌹 contraindication for breastfeeding? cracked nipple🍒🍒
....... ......
Female pregnant and smoking, Best advice?Smoking cessation
.....
🌹PCOS symptoms what to test for?
glucose and lipid profile🌹
.......
🌹Decreased fetal movement?
non-stress test🌹
..... ....
🌹 hyphae in urine microscopy? Candida🌹
‫ربط = هيفا من كندا‬
....
🌹Bleeding diagnosed with Ovulatory dysfunction treatment?OCP🌹
.... ....
🌹17 years old medically free brought to Gynecology clinic by her mother
with history of no menstruation. On examination there was low hairline, high BP and short stature. Both mother and father
were having short stature at her age. What is the most likely diagnosis?
Answer is: Turner🌹
. ... . .
🌹Young girl with symptoms of primary dysmenorrhea treatment? NSAID🌹
.....
A women with disabling symptoms of menstruation that affect daily activity= SSRIs
.................
🌹Vaccine in pregnancy=Influenza, Dtap.🌹
.... ......
🌹 A women with symptoms of dysmenorrhea, she took NSAID but needs
stronger treatment, her symptoms not significantly affecting daily activity?OCP 2nd line
....
General Surgery
🍒elderly patients around 60's having asymptomatic inguinal hernia, discovered incidentally during a routine check up, what to
do? Observe bez Asymptomatic . 🍒
. . . . . . .. . .
🍒hirschsprung disease management?
Surgical resection 🍒
Hirschsprung's disease occurs when nerve cells in the colon don't form completely. Nerves in the colon control the muscle
contractions that move food through the bowels , not work therefor this muscle will be very weak and could not pass stool ,
espichaly at birth , not pass mecnium .
Hirschsprung's disease is a very serious condition. But if found quickly, it can almost always be cured by surgery. usually do one
of two types of surgery:
Pull-through procedure: This surgery simply cuts out the part of the large intestine with the missing nerve
. .. . . . .
🍒case about large calcified echinoccocus But all choices was medications name = its case of Hydatid cyst = Alpendizole (If
rhere is option of surgery it will be more right ) ✅ 🍒
Treatment echinoccocus cyst ... Chemotherapy ( like Alpendizole) , cyst puncture, and PAIR (percutaneous aspiration, injection
of chemicals and reaspiration) have been used to replace surgery as effective treatments for cystic echinococcosis. However,
surgery remains the most effective treatment to remove the cyst espichaly if it was large and can lead to a complete cure.
.......
🍒ttt for long therapy prevent of bleeding of esophageal varices ? b blocker (propranolol) 💐
....
💐 tt of features of severe DVT ? LMWH + Warfarrin 💐
......
🍒post surgery case and developed features of DVT Asking about ttt.? LMWH + Warfarin ( if thers is Enoxaparin , Bez this drug
is low molecular weight heparin. 💐💐
. .. . .
‫ عشان نعرف كيف نعطيهم‬،‫ نتكلم شوية عن الوارفرين والهيبارين‬...
Heparin and warfarin are 2 types of anticoagulants that are used to treat DVT. Heparin is usually prescribed first because it
works immediately to prevent further clotting. After this initial treatment, you may also need to take warfarin to prevent
another blood clot forming.
Heparin:
Heparin is available in 2 different forms:
# standard (unfractioned) heparin
# low molecular weight heparin (LMWH)
dosage must be carefully monitored and adjusted if necessary. You may need to stay in hospital for 5 to 10 days and have
frequent blood tests to ensure you receive the right dose.
LMWH  contains small molecules, which means its it's easier to use and causes fewer side effects. and you won't have to stay
in hospital and be monitored.
Warfarin
need to take it after initial treatment with heparin to prevent further blood clots occurring. Your doctor may recommend that
you take warfarin for 3 to 6 months. In some cases, Warfarin isn't recommended for pregnant women who are given heparin
injections for the full length of treatment.
....
🍒 -eledrly man k/c of ESRD developed features of DVT Asking about ttt.?
UFH🍒
‫ وعادي لو استخدمنا معهما‬LMWH ‫💐 كل االبحاث تقول انه امن‬
. . .. .
Close monitoring of anticoagulation is recommended when argatroban or high doses of unfractionated heparin are
administered in patients with severe chronic renal impairment. Low-molecular weight heparins, danaparoid sodium, hirudins,
and bivalirudin all safe.
.... ......
💐free air under diaphragm?Laparotomy💐
...... .......
Varicose veins for cosmetic want to remove? Sclerotherapy
.......... ........
💐Varicose vein feels pressure want to remove? Endovenous laser ✅✅💐
....
💐Hernia ligation is done for which type of hernia? Internal💐
......
💐A boy with signs of obstruction post surgery what best investigation to be done? X.ray for obstruction intestin💐
.....
Post surgery slugish bowel sounds ? Ileus
........
💐💐 Mass in distal esophagus ?!
Adenocarcinoma of esophagus
SCC > middle and upper esophagus
Adenocarcinoma > lower 💐
...
💐canceo hilum of lung?SCC(Not adeno)💐
..........
💐Old age patient complain of abdominal pain and tenderness And axr showed multiple air-fluid levels Ct with contrast show
cut off dye at terminal ileum showing mass and target sign
A. small bowel cancer
B. late chron
Cutoff sign might be seen in : acut pancreatitis ✅💐
 signs in conventional radiographic imaging of the abdomen in acute pancreatitis is the so-called colon-cut-off-
sign.This sign refers to the abrupt termination of gas filling of the colon at the level of the left flexure.
The more distal part of the colon usually shows a markedly reduced or a lack of gas filling. This bowel gas distribution, which
feigns a constriction of the colon at the left flexure, has been observed within the context of acute pancreatitis for over 50
years
......
💐*thyroidectomy for pt has valve regurgitation Ask about ABx as prophylaxis ? no need prophylactic 💐
.............
💐Abcess in lesser sac = Percutaneous drainage 💐
...........
Old pt smoker diabetic Sx of claudication ^left leg ^ by Ex found pulsatile mass or sub inguinal canal teder non reducible ?
A. ileocecal nerve
B. Psoas abscess
C. pseudoaneurysm ✅✅
(BEZ pulsatile )
D. femoral hernia
💐Old pt smoker diabetic Sx of claudication ^left leg ^ by Ex found pulsatile mass or sub inguinal canal teder non reducible =
pseudoaneurysm💐
........... ........ .......
💐Traumatic pt thoracic aorta and splenic laceration what Mx ?
Laparotomy and thoracotomy 💐💐
. . . . . . .. . . . . . . .
💐Farmer dx as hydatid liver cyst Mx?
A. Percutnus Drainage
B . Alpindizol ✅
Hydatid cyst of the liver must be treated surgically. Albendazole 10 mg/kg/day for 3–6 weeks before surgery should be given to
sterilise the cyst.💐
.......
💐💐Mx of pneumothorax increase JVP?
A_ Thoracostomy
B_ needle > then chest tube ✅
Treatment is immediate needle decompression by inserting a large-bore (eg, 14- or 16-gauge) needle into the 2nd intercostal
space in the midclavicular line. Air will usually gush out. 💐💐
.........
💐Peri anal leasions Qualiforme ‫ ?? شي زي كذا‬chondylomata acumenta 💐
......
🌹Post appendectomy the wound with purulent discharge next step x?
A. Dressing !
B. Or exploration ✅
How to manage surgical site infection?
First evaluate surgical wound by *inspection*
*If there’s signs of infection* (purulent discharge, erythema, tenderness, then wound exploration
*if superficial >* clean, drain pus, dressing and you may give abx
*If deep collection suspected >* order CT
*If small collection less than 4cm >* abx
*If large 4cm and more* > percutaneous drainage + abx
*If signs of peritonitis >* Laparotomy 🌹
‫👏 من جلوري تيم‬
.....
🌹Patient have femoral fracture + pleural effusion after MVA, they did chest tube and stabilised the femoral want to transfer
him to another hospital for surgery, on the way he had sudden tachypnea and shortness of breath ?
A_ Check femoral bleeding
B_ check chest tube ✅ ‫حل جلوري‬
......

Surgery
Post appendectomy case and there is pain and in abdomen X-ray there was
mass what is u management
A. Observe
B. Antibiotics
C. Surgery
‫ من كتاب‬tornot.
🍒if localized abscess (palpable mass or large phlegmon on imaging and often pain >4-5 d), consider radiologic drainage +
antibiotics x 14 d ± interval appendectomy in 6 wk. (controversial)
......
🍓Patient with tingling in ring and little fingers with positive elevation test ?
A. thoracic outlet obstruction ✅
B. carpaltunnel ×
C. ulnar thrombophlebitis ×
🍒What are the signs and symptoms of thoracic outlet syndrome?
  The classic, most common symptoms are pain, numbness, and tingling that radiates below the shoulder down towards the
hand and usually into the pinky and ring finger.
🍒Numbness and tingling in the ring finger and little finger are common symptoms of ulnar nerve entrapment (Cubital Tunnel
Syndrome)
.......
🍓 Diabetic pseudo eplthelium :
A. Amputate
B. surgical debridement ✅
.....
🍓 POST ERCP alr in abdomen, chest, neck which organ Injured
A. Stomach
B. Duodenum ✅✅
C. Esophagus
D. Trachea
.....
🍓Best test to rule out cervical spine ?
A. MRI cervical ✅✅
B. AP X-ray of cervical
C. Ct (without cervical)
D. clinical exam
......
🍓Breast abscess with skin redness only but no dimpling mx ?
A. l&A ✅
B. Aspiration
C. Abx
treatment: initially broad-spectrum antibiotics and I&D✅
..
about a patient who had symptoms of bowel obstruction and asking about
best investigation ?
No X-ray
A. CT ✅
B. Us
C. Enema I think
........
Risk factor of breast cancer
A. age ✅
B. late menopause
C. early menarche
Risk Factors
• gender (99% female)
• age (80% >40 yr old)
nulliparity, first pregnancy >30 yr, menarche <12 yr, menopause >55 yr
...
Chronic constipation treatment :
A. Sena
B. debulking agent
......
🍓Elderly Chronic constipation + lower GI bleeding , initial investigation
A. Sigmoidoscopy
B. Colonoscopy ✅
.......
🍓Gi endoscopy revealed ( interstitial type metaplasia in distal esophagus ) =barretts
.......
🍓Long scenario of patient with diffuse thyroid enlargement On US they
found 1'2 cm nodule on the right lobe and 3'4 cm of the left lobe , What is
the next step?
A. FNA of both nodules ✅
B. FNA of the larger nodule
C. total thyroidectomy
D. biopsy
...
🍓medullary thyroid carcinoma = total thyroidectomy
.........
🍓abdominal pain radiating to back.symptoms of pancreatitis but showing x ray with air under the diaphragm, diagnosis?
A. duodenal perforation ✅
B. Acute pancreatitis
.......
🍓Pt do ercp for cholylithiasis operation was coplicated than pt develop
emphysema ,Abdomen ,chest and neck which organ injured?
A. Duodenum ✅
B. Gastric
Duodenum is the most common cause of subcutaneous emphysema post ERCP
.........
🍓 Patient had fresh blood bleeding per rectum, examination revealed
collection of thrombosed blood in the rectum, patient has nasogastric tube
with green discharge.Colonoscopy no pathology. What is the appropriate
investigation to know the site of bleeding?
A. Upper GIT endoscopy ✅✅
B. Can't remember
C. Gastrografin enema
D. Technetium Tc-99m sodium pertechnetate (Tc sestamibi is used for myocardial perfusion imaging, which shows how well the
blood flows through the heart.)
.......
🍓Old patient after repeated vomiting developed epigastric pain and
subcutaneous emphysema, diagnosis?
A. Gastritis.
B. Esophagitis.
C. Perforated peptic ulcer.
D. Boerhaave syndrome. ✅
Effort rupture of the esophagus, or Boerhaave syndrome, is a spontaneous perforation of the esophagus that results from a
sudden increase in intraesophageal pressure combined with negative intrathoracic pressure (eg, severe straining or vomiting).
......
🍓Pt with Dysphagia to liquid e regurgitation of food how to diagnose?
A. UGI endoscopy
B. Barium swallow
C. Lower esophageal myotomy ✅
Achaiasia mostly
.......
🍓Most common surgical cause of abdominal pain in pediatrics?
A. Appendicitis ✅
B. Cholecystitis
....
🍓Patient came for checkup after open appendectomy for perforated
appendicitis, patient has seroma that is discharged freely from the wound,
what is next?
A. Diagnostic laparoscopy
B. Exploratory laparotomy
C. Regular wound dressings ✅✅
....
🍓Patient has medullary thyroid cancer on the left lobe , right lobe is normal,
management?
A. Total thyroidectomy. ✅
B. Subtotal thyroidectomy.
C. Hemi-Thyroidectomy.
D. Tumor inoculation.
......
🍓patient presented with Jaundice and fever and there is CBD dilation
A. cholecystitis
B. cholangitis ✅
C. gallstone
D. pancreatitis
...
🍓Risk factor of breast cancer
A. DM
B. late menarche
C. early menopause
Answer is A by exclusion
......
🍓The most significant risk factor for breast cancer :
A. age ✅
B. early menarche
C. late menopause
D. nulliparity until 30 age
....
🍓Pregnant , developed rt side movable mass not attached to skin management :
A. mammogram
B. bilateral mammogram
C. FNA
D. reassure and reassess after delivery ✅
Answer is D or US at least.
.....
🍓what's Most common cause of retroperitoneal sarcoma:
A. retroperitoneal bleeding
B. Compression of nearby structure ✅
C. Invading of nearby structure
.....
🍓MVA Seat belt sign symptoms
A. duodenal perforation
B. Jejunal ✅
....
🍓Patient have trauma in right chest and have pneumothorax and do chest
tube after time have 2 liter of blood come what's next step ? Thoracotomy
Massive Hemothorax is
>1,500 cc blood loss in chest cavity + managmannt by =
1_ Restore blood volume
2_ Chest tube
• Thoracotomy if:
• >1,500 cc total blood loss
• ≥200 cc/h continued drainage
....
🍓case of mild cholecystitis ask about treatment = laparoscopic cholecystectomy within 72HRs
.....
🍓pOSt op complication at day 5
A. DVT
B. UTI ✅
.........
‫نتكلم شوية عن‬:
post op complicantion :
🍏Post-Operative Fever🍏
🍒fever does not necessarily imply infection particularly in the first 24-48 h post-operative .
🍒timing of fever may help identify cause:
🍒1️⃣_ fever after 1hours after surgery (immediate):
🔹️inflammatory reaction in response to trauma from surgery; unlikely to be infectious.
🔹️reaction to blood products received during surgery.
🔹️malignant hyperthermi.
🍒2️⃣_ Fever after 1-2 day (acute)
🍓atelectasis (most common cause of fever on POD 1_2 day.)
🍓early wound infection (especially Clostridium, Group A Streptococcus – feel for crepitus .
🍓aspiration pneumonitis
3️⃣_ Fever after 3-7 day (subacute)
= likely infectious = ( UTI , surgical site infection, IV site/line infection).
4️⃣_ 8 day after (delayed)= intra-abdominal abscess, DVT/PE (can be anytime post-operative, most commonly POD #8-10).

.........
🌹10 yr old with dm type 1 known 1 month ago when to do eye screening?
A. Emediate
B. after 1 year (Type 2)
C. 5yrs ( after first diagnosis ) ✅🌹
.......
💐 Picture of echinococcus very large size( occupies half of liver), they say in scenario diagnosis is echinococcus what to do next?
Surgery 💐
….
Surgery
🌹during Lapchole, found a gastric mass what should be done ?
A. stop procedure
B. continue Lap choleucystits ✅✅
C. remove mass
D. Lapchole and remove mass
Never do anything without consent except for life-saving, limb-saving 🌹
. . . . . ..
🌹After trauma GCS 8intracranial pressure 15mmhg arterial pressure 65 mmhg .cerebral perfusion pressure in mmhg? 50🌹
Cerebral perfusion pressure (CPP) =MAP-ICP
.. . . . . . .. . .
🌹RTA 2nd rib fracture 4rib what to do increase respiratory rate pr what initial go do: Acess ventalation ✅
.. . . . . .
🌹Complaint of loss appetite 1 month epigastric pain mass epigastric,
atrophy muscle face +loss of wt diagnosis
A.Stromal carcinoma ✅ (Gastrointestinal stromal tumor )
B. Liver metastasis C.Adeno.
......
🌹1 is day post operative cholecystectomy develop decrease air sound in lower lung what to do == Intensive spirometry
...... ........ .......
Right sided abdominal pain o/e right side abdominal tenderness with positive Grey Turner’s sign = acute pancreatitis ✅✅
‫ = ربط‬panc = ‫ بان كيك 🧁= تارن‬turn= ‫ تعال كل بان كيك = جلس يجري = جري‬grey ‫🧁 حتى يلحق‬
.....
🌹Girl underwent sleeve 17 days ago, presenting with persistent vomiting and mood changes according to family ?Abd us N Ct
N Whats next?
A. Colonoscopy B.Psychiatry referral
C.Reassurance ✅✅🌹
..........
🌹Smoking incubation period in lung cancer = 10 years✅🌹
..........
🌹Patient with spiking fever and anterior bugginess in DRE after colon surgery. Next. A. IV Ab B.Ct
C.Abdominal wash out
D.Ultrasound drainage ✅🌹
....
💐 H.pylori .. found to have Multiple Ulcers and Gastric MALT ..treatment?
A. Chemo B. Radio
C.triple therapy for H Pylori ✅
D. surgical removal
....
🌹Pneumothorax = needle aspiration 🌹
......
🌹Saudis percentage smokers = 21%🌹.....
......
🌹surgery to move Undescended testes into the scrotum = orchidopexy🌹
‫ ربط = رشوة‬orchi = ‫ = ارشي فلوس‬desecend ‫ شيء نازل لالسفل النه عمل سيء جدا‬...
.........
Cardiac tamponade management
= Pericardiocentesis
.....
🌹 Lung mass screening =Screen by low dose CT scan for adults from 55 to 80 who smoke 30 packs per year or quit within 15
year 🌹
............
🌹Mirizzi syndrome :
gallstone in the cystic duct causing compression of the common hepatic duct💐
......
💐A 66 years old male was hyperthyroid for last 3 months. After proper evaluation and necessary lab tests, he was prescribed
radioiodine to destroy the hyperactive thyroid gland. He is now evaluated again in the outdoor department for the efficacy of
the treatment and was found to be hypothyroid now. What should be the best next step in the management of this patient?
A. Start levothyroxine at full replacement dose.
B. Go for evaluation for malignancy.
C.Start levothyroxine at half the normal dose. ✅
D. Perform radionuclide scan
...... ..
🌹 A young female patient with clear bilateral breast discharge. She is otherwise normal. What will you do = prolactin level🌹
..........
🌹 How to monitor patient condition after radioiodine ablation? TSH🌹
A. clinical picture B.T3+T4 C.TSH ✅
............
A diabetic patient presents with unhealed foot ulcer for months. What should you do
A. Antibiotics B. Refer to Diabetic clinic
C. Wound swab
D.Debridement and tissue biopsy ✅
.....
🌹 20 years old with a skin tag/lesion of 1-2 cm in forearm, no changes since birth, (no further descriptions or colors of the
lesion). Management? Reassure
.........
Color of discharge from duct papilloma? Red 🌹
..........
🌹Trauma Patient when knee examination, the tibia was forward anterior to femur, which ligaments affected?
A.Anterior cruciate ✅🌹
B. Posterior cruciate
C. Medi/lateral
D. Leminiscal tear
‫ من اسمها‬anterior ‫ مع‬anterior.
. . .. . . .
Long case of pt post thyroidectomy have lost sensation below right ear and upper neck and mandible and under the lower lip.
They asked what nerve is injured? is :Great auricular nerve
.......
‫مصدر االسالة من جلوري تيم فقط جزاهم هللا خيرا‬
Surgery 21 , 22, 23, 24 octop
🍒Urethral injuries?Suprapupic cath
......
🍒sign of obstruction erect x ray show multiple air fluid level initial step ?
examination of the groin for any hernia.
.......
🍒 latral and inferior to pupic tubercle ?
femoral hernia
........
🍒pt bilateral rinne test positive and weber lateralized to the right ?
A. conductive hearing loss on rt side
B. conductive hearing loss on left side
C. sensorineural hearing loss on the left side✅
+ve rinne test -> SNHL or normal
-ve rinne test -> CHL
‫ ربط = الصوت رنين مرة يزعج االذن = حساس‬sensory , ‫كثير‬ ‫ فلما نسمع الصوت رنين نعرف انه االذن راح تتحس‬.
.....
🍒pt come with fever earache perforated tympanic membrane and release of pus through external auditory canal ?
A. acute otitis media✅✅(fever +pus)
B. serous otitis media
C. chronic otitis media
........

🌹 Hypothenar atrophy but sensation intact? Ulnar injury

🌹weak of dorsoflection of hand , which nerve ? Radial 🌹

Patient diagnosed with CKD now he is coming in outpatient clinic and you decided to start him on
hemodialysis, his crt is 10, what best method of access to start hemodialysis?
A. Arteriovenous graft‫ جرافت مشاكلها كثيير‬B. Arteriovenous fistula

🌹55 man hypertensive and has renal insufficiency. He is evaluated for AV fistula , no history of trauma or
AV malformations in his non dominant hand , what type of fistula should be offered for him first=A-
Brachiocephalic B-radiocephalic ✅✅

🍒testiculer pain with an absent cremasteric reflex. Best step to do ?


A. US of scrotum
B. Surgical consultation✅
C. give painkiller
D. give antibiotic
.........
🍒 post appendectomy day 8 present with fever and pus coming out of the wound what to give ?
A. Iv antibiotics B. Open drainage✅
C. Ct abdomen D. Laparoscopic drainage
......
🍒Bethesda IV score tt?hemithyroidectomy
.....
🍒BiRADS 4 for breast=?Core biopsy
.......
🍒Cervical lymph node swelling ,cells is cervical follicular cell What is treatment?
A. surgical referral✅
B. reassurance
........
🍒55 years old full screening of mammogram normal , when next mammogram? 2 year
......
🍒 breast oval, smooth mass what’s the diagnosis ? Fibroadenoma ( Not breast cyst)
........
🍒scrotum swelling +ve Transillumination ?
. hydrocele
‫ ربط = الضوء‬transllumi ‫ يمر عبر الماء‬hydro
‫ لما تشوفوا كلمة‬trans + ‫ بدون تفكير اختاروا‬hydrocele .
.......
🍒scrotum pain for 5 hours came to ER -ve light reflex what do?
A_ surgical consult✅ B_testis U/S
TESTICULAR TORSION
1_absent cremasteric reflex.
2_ Testis elevated into the inguinal canal + swelling ( horizantal)
3_ Immediate surgery to salvage testis;
surgical Orchiopexy typically also describes the surgery used to resolve testicular torsion.
.........
🍒Anal fissure surgery after medical tt failed ?? lateral internal sphincterotomy
..........
🍒patient with hx operation “I think in rectum or anus”developed fever and can’t pass stole, anal redness, examination was
difficult because cannot enter finger into
anus diagnosis??
A. anal stenosis B. anal abscess✅
If fever -> abscess .
.......
🍒women has after meal abdominal pain , palpitation relieved by vomiting what investigation ?
A. Barium meal , B. endoscopy✅
......
🍒Case of asymptomatic gallbladder polyp, management?
A. follow up after 6 months.✅
B. the other options were surgical
.........
🍒 Scenario of Toxic megacolon. Colon dilation on imaging 15 cm. Next ttt?
A. high dose steroid✅ B. Surgical resection
‫ ليه ستيرويد ؟ الن معروف انه يخفف االلتهاب قي اي مكان بالجسم وسبب انه حصل‬megacolon ‫مرض‬ ‫ هو‬ulceraitve coloitis ‫وهذا‬
‫ فالهدف ليس جراحة اهم شيء نخفف‬، ‫ فلما نعطي ستيرويد راح يخفف هذا االلتهاب‬، ‫المرض معناها حصل غي القولون التهاب شديد‬
‫ االلتهاب االول‬.
Toxic megacolon = manifestation of fulminant colitis = extreme dilatation of colon (>6.5 cm) + loss of normal haustral pattern
= Treatment= NPO, NGT + spectrum antibiotics + steroids in IBD, metronidazole for C. difcile)+ surgery indications = 50%
improve on medical management = worsening or persisting toxicity or dilation afer 48-72 h + severe hemorrhage, perforation
= surgery.

.........
🍒Elderly, progressive jaundice, abdominal discomfort and RUQ pain. On imaging showed intrahepatic duct dilatation with
shrunken gallbladder. The diagnosis?
A. mirizzi syndrome B. Klatskin tumor ✅
C. Periampullary tumor D. gall bladder cancer
‫ ربط = كالتي سكين = خالص جلدي سار ذايب‬shrunk
....
🍒 Old Pt with sudden abdominal pain, long senario. Examination showed empty rectum. Y shaped colon on image. The most
appropriate management?
A. Fleet emena
B. Sigmoidectomy with end colostomy
C. CT with contrast
D. Colonoscopy
Colonic duplication -> best CT
....
🍒Pt presented with signs of bowel obstruction (vomiting, abdominal distention). Abdominal X Ray revealed air fluid level. Dx?
A. obstructed✅
B. Irreducible
C. Incarcerated
D. Strangulated
....
🍒 Pt was having reducible inguinal hernia then presented typically as the above scenario, asking about complication
this hernia can do?
A. irreducibility
B. Obstruction
C. Incarceration✅ ‫حل جلوري‬
D. Strangulation
‫ كلهم صح بس لو نمشي باالرتيب اول مضاعافات يسير ء يسير‬incar
......
🍒 Pt 6 hours post thyroidectomy presented with neck swelling and had difficulty breathing, no other complains + next step in
management?
A. tracheostomy
B. IV calcium gluconate
C. Epinephrine inj
D. Suture wound site exposure/exploration in the ward✅✅
🌹Hematoma post thyroidectomy Symptoms may include an area of firmness and swelling on the front or side of the neck
(usually beneath the incision), neck pain, and symptoms of airway obstruction such as shortness of breath, lightheadedness, or
stridor (a high-pitched wheezing sound that's usually most noticeable with inspiration than exhalation)=.immediate surgery to
remove the hematoma and address any areas of bleeding.
........
Same and typical scenario but 5 hours post thyroidectomy neck swelling, but pt has stridor and SOB. Vitals within
normal, only RR is there = 24, no O2 Saturation given. Treatment?
A. tracheostomy
B. Percutaneous drainage
C. Wound exposure/exploration✅✅
If a neck hematoma is compromising the patient's airway, open the surgical incision at the bedside to release the collection of
blood, and immediately transfer the patient to the operating room.
.......
🌹After surgery. The incision was sutured. Then scary happened at the same site of the surgical scar. It was removed, but it
grew again and again and grew even larger. What’s the most likely diagnosis?
A. Keloid Scar✅ B. Warts C. Melanoma
.........
🌹Pt after accident has an open wound in the neck with destabilized tissue what to do next ?
A. Oxygen mask ✅ B. Tracheotomy
C. Cricothyroidotomy
..........
🌹 7 years post cholecystectomy ( us show dilated CBD) what the cause
A. Retrain stone✅ B. Bile duct injury
......
🌹absolute constipation with free rectum, result:( dilated loops at upper right quadrant) what the possible cause?
A. sigmoid volvulus ✅B. loop obstruction
‫ = ربط = سيجمويد = سجود = السجود شيء كويس‬right .
.....
🌹Crohn patient after ileocecal resection After one month he starts to have watery diarrhea 5 times a day. Physical Exam :
unremarkable ,Lab : unremarkable ,Ct normal ,What is the most appropriate treatment ? A. mesalamine B. prednisone
C. cholestyramine‫✅ لعالج االسهال‬
..........
🌹knife injury before 3 days now came with progressive erythema in the dorsum of the hand == A. Cellulitis B. Gas gangrene
C. Necrotizing fasciitis✅
.......
🌹long scenario then ask about most common small bowel cancer
A. Lymphoma B. Carcinoid ✅C. Schwan...
‫ = كسر‬9‫ = كارسينويد‬small .
....
🌹How to prevent atelectasis after operation: = incentive spirometry = is a medical device used to help patients improve the
functioning of their lungs.
‫ اتليكتازيا = اقلقتيني = خالص انسى القلق = انسينتيف = انسى‬.
.....
🌹most striking indication of vascular injury : A-change in color of the skin✅
B-massive fractures
........
🌹 Patient done surgery 7 days ago Had abnormal bowel sounds abdominal pain=
A. paralytic ileus✅B. volvulus C. adhesion
‫ لو قال ايام بعد العملية نفكر ب‬، ‫ عدد االيام مهمة لو قال سنةات بعد العملية نفكر بااللتصاقات‬paralytic ‫ يسمونه شلل االمعاء‬.
.........
Retinoblastoma diagnosis: A. MRI✅B. CT
......
🌹stab wound in the right upper quadrant. No mentioned any vital sign
A. Ct scan✅ B. Observations
If stable CT, unstable laparotomy.
........
🌹thigh injury unconscious, active bleeding
A. Iv ringer lactate✅ B. Intubation
........
Most important sign of inhalation burn?
A. Tachycardiab B. Hypoxia
C. Change in voice (hoarseness ) ✅
‫ ربط = فيه حريقة = يصارخ بصوت عالي‬.
.....
🌹 tension pneumothorax management
A. Needle ✅ B. Chest tube. C. Conservative
Tension pneumothorax Treat with needle thoracentesis, followed by insertion of a chest tube.
.......
🌹 abdominal aortic aneurysm  case after surgical repair came with low blood presser, high vascular resistance & normal PCWP
type of shock ? Hemorrhage ✅
B. Septic C. Anaphylactic D. Obstructive
...........
🌹Female Patient with DVT what to ask in the hx: OCP use
.......
🌹Patient on ocp presented with RUQ pain on investigation 5cm hemangioma tt?
A. Excision✅ B. stop ocp
.......
🌹 Pt underwent cholecystectomy after that develop ruq pain , us show dilated cbd and small collection in gall bladder grove?
remaining stones in the CBD
.......
🌹 25 years old , chemical burn , how to remove burning clothes from skin ?
A. water irrigationb ✅B. powder sweep
A if liquid chemicals, B if dry chemicals
.........
🌹Pt e trauma and crushing of his leg, e pain and weak puls what is the most appropriate investigation:
A. CTA ✅ B. Angiography
.......
🌹Patient with DM and HTN presented with ingrown toe nail, what is the thing you must do before the operation:
A. Check peripheral pulses ✅B. Check the other toe nails C. Take swab culture
D. Antibiotics
.. ..
🌹Pt came with open fracture of the leg, there is devitalized tissue and dirt at the site of the fracture what is the best initial
step ? A. surgical debridement B. External fixation C. Antibiotics✅
......
🌹Pt after few days of cholecystectomy, she develop chills and rigor, temperatures, HR and RR and WBC count was given ?
A. bacteremia
B. SRIS (high wbc + high temp, high RR , tachy , low blood presser without confirm infection , after confirm infection called sepsis
C. Sepsis ( high wbc + high temp + confirm infection ) ✅
D. Septic shock ( persistent hypotenstion with end organ damage )
.....
🌹Patient came complaining of swelling in inguinal area ...etc, you examine the patient but examination were negative what is
the
best next step ? A. US ✅ B. MRI C. CT
......
🌹You are primary health care physician in hospital with no facilities, pt came after RTA with fractured femur and
pneumothorax , you stabilize the pt and insert chest tube, in your way to a big
hospital for further management the pt developed tachycardia and tachypnea, what should you do ?
A. IV fluid B. Rapid sequence intubation
C. Check chest tube placement and patency ✅
tachycardia and tachypnea= Tenstion pnumothorax .
.........
🌹Patient diagnosed with thigh sarcoma how you will stage it ?
A. bone scan B. MRI abdomen C. X-ray chest D. CT chest ✅
....
🌹 Patient with bowel obstruction post appendectomy which of the following medication is contraindicated in this case ?
A. profolol B. Ketamine C. Nitric oxide✅
‫ اوكسايد = أوعى تستخدمه مع اي شيء فيه انسداد‬:)
.......
🌹 Diabetic pt came with ulcer to the right lateral mallules , there is discoloration around the wound, puls present, healthy
looking ulcer bed next investigation ?
A. MRA B. Arterial duplex C. Venous duplex✅
.........
Hemothorax case what to see in the xray?
A. lung collapsed in the right side✅
B. consolidation in the right side
.......
🌹15 year old presented to the clinic with neck mass, painless midline mass that moves with swallowing. diagnosis?
A. thyroglossal cyst✅ B. hematoma
C. lymphadenopathy
......
🌹 Which orgnism cause adhesion post operative ? A. E.coli B. Chlymdia ✅C. Staph
‫ ربط = التصاقات = بعد ما المريض يطلع من العملية االهل يلتصقوا و‬chlym ‫ كالم كثير واسالة عن كيف المريض طلع‬.
.......
🌹A man came to ER complaints of soot in the respiratory tract , signs of inhalation injury. most appropriate action ?
A. Face mask B. nasal Cannula
C. Intubation✅
......
🌹40-year old man has history of varicose veins for 20 years , now complaining of right cord-like swelling and pain for 48 hours.
appropriate management:?
A. NSAIDS✅ cause this is the superficial thrombophlebitis
B. Heparin C. Mechanical
.....
🌹A lady complains of breast mass , biopsy done and shows : phylloid tumor 5*5
appropriate management:?
A. Radiotherapy B. Chemotherapy
C. Wide local excision ✅
D. Simple Mastectomy ( only indicated if you can’t take the safety margin ).
‫ ربط = فيل = كبير وضخم‬wide local
......
🌹Trauma with blue discoloration and gas within the wound what type of bacteria?
A. Clostridium perfringen
......
🌹Sigmoid voluls admitted and in pain since 24 hr, X-ray showed Y shape, wbc 20,000. What is the management ?
A. Sigmod colectomy and anastomoses
B. Colonoscopy c. CT✅
.......
🌹22 years old tall with dysnea and chest pain x ray showed pneumothorax , what is the appropriate initial management step?
A. thoracocentesis ✅B. chest tube C. observation
Needle aspiration (thoracocentesis) is now an established initial intervention in selected patients with primary
spontaneous pneumothorax
.(fail put chest tube)
.....
🌹Pregnant with right side pain, after ruling out all OBGYN causes, what can it be?
A. Appendicitis ✅B. Cholecystitis
.......
🌹Female complaining of varicose veins in her legs, there was no pain associated with it but patient wants to treat it for
cosmetic reasons, what do you want to investigate this case:
A. No need for investigation in this case
B. Venous Doppler US✅
C. Magnetic resonance venography
D. CT
.......
🌹1 cm adenocarcinoma from the anus best management?
A. abdomino-perineal resection=surgery in which the anus, rectum, and sigmoid colon are removed. This procedure is most
often used to treat cancers located very low in the rectum or in the anus.
B. LAR = LAR is a common treatment for rectal cancer when the cancer is located well above the anus
C. Local exsition ✅ ‫الن واحد سم‬
........
🌹 Patient post thyroid surgery, SOB what to do? open wound bedside✅
....
🌹Patient has Achalasia cardia 5 days post esophageal dilatation presented with neck pain and emphysema in the neck, chest
and abdomen. They did labs found increased WBCs and patient had 40 degrees fever, other vital signs were normal. They did
gastrografin and found leak in the esophagus. How would you manage the patient:
A. Esophageal Stent
B. Surgical drainage and repair✅
C. Conservative
D. Abx and follow up after 1 week
........
🌹 Progressive increase brest mass in 2 yrs size 15*15 cm.. management
A. Chemo
B. Wide local excision
C. Simple mastectomy✅
.....
🌹Ventral hernia? Subly
....
🌹Patient complaining of right lower quadrant pain for 3 days presented to ER and was managed conservatively for 1 week
then was discharged, On follow up US (or CT I don't recall) they found a mass in appendix area but no pus or abscess
accumulation. How would you manage:?
A. - Colonoscopy after 6 weeks
B. - Open Appendectomy after 12 weeks
C. - Laparoscopic Appendectomy after 12 weeks✅
D. - Reassure the patient
........
🌹 70 years old with sacral ulcer, fat tissue was exposed, what is the management?
A. Debridement & primary closer
B. Debridement and graft✅
C. Debridement and VAC
....... ...........
🌹 40 years old with abdominal pain in the RIF for the last 3 day, he tells you he was hit with a wood 4 days ago in the same
area. Any movement of his leg would aggravate the pain, on inspection there
is a small opeing in the skin and purulent fluids are coming out of it. next best step? (Enterqutanues fistula distribution)
A. CT✅ B. Surgical drain
C. Culture the discharge
.......
🌹 Physical exam : fixed unilateral breast mass retracted nipple Mammogram: speculated mass suggested malignancy
Most appropriate next step in management ?
A. Modified radical mastectomy
B. Ct chest and bone scan✅
.......
🌹hip effusion : best next step
A. Joint fluid aspiration✅ B. X-ray
......
🌹Vein thrombosis of spleen= Splenectomy
......
🌹 Most common shoulder dislocation
= posterior dislocation
........
🌹Bilateral varicosities in both leg , with symptoms ( but no ulcer or pain ) just uncomfortably next step?
A. V duppler ✅B. A duppler
C. Ct angio D. No need for investigation
.....
🌹 Old age Hx of IHD , after cholecystectomy had chest pain you
can confirm dx by
A. ECG✅ B. Echo C. Chest x ray
......
🌹Post ERCP , epigastric pain N/V
Lap high WBC , high total bilirubin , CBD 9mm
A. Re operate now with ERCP
B. Observe
C.Ct ✅

The most common complication ERCP is pancreatitis which is due to irritation of the pancreas from the dye used to take
pictures,
specific complications of ERCP, which include pancreatitis, duodenal perforation, duodenal hemorrhage, infection, stent
migration. CT is performed if patients have severe abdominal pain, elevated white blood cell count, and fever after ERCP.
.....
🌹 Man with fracture he had fever next step?
A. Antibiotic✅ B. Surgical debridement
C. Internal fx D. External fx
.........
🌹After surgery you want to discharge the pt : urine output: 35 ml , temp.: 37.4 . Normal Hb and hematocrit but pt still in iv
fluid and didn’t mention if he can tolerate oral feeding then ask about what thing let you not discharge the pt;?
A. Hb and hematocrit level
B. unable to tolerate oral
C. fever D. urine output✅
......
🌹40 years old women presented with 4 h of severe right leg pain , femoral pulse palpable and distul not , diminished sensory
and alter motor , medically free , but ECG show A fib = investigation?
A. Ct ango✅ B. Vascular us
C. Conventional angiography
D. Embolectomy with no amage
......
🌹 Pt presented with RUQ pain And jaundice, pt look unwell and abdominal tender witn no gurding , WBC 15 HIgh total
bilirubin ,High alkaline phos Us show ( multiple stone with no pericystic fluid and dilated CBD with 1 *cm* Doesn’t mention fever
in scenario
A. Ascending cholangitis
B. Acute pancreatitis
C. Acute cholecystitis✅
D. Choloducholethiasis
......
🌹72 year old male present with mild intermediate urine flow , Rectal and urinalysis and PSA normal what is the investigation to
diagnosis ?
A. repeat PSA after time ✅B. cystoscope
.........
🌹Patient have seizure and develop flat shoulder type of dislocation?
subacrompoin posterior
‫ = ربط‬acrompoin ‫ اركع وخلي ظهرك‬post ‫ مستقيم في الركوع‬flat
..........
🌹82 years old male presents with urine retention and low back pain +
high Alkaline phosphatase High PSA
A. prostatitis B. prostatic cancer✅
C. bladder cancer D. BPH
.......
🌹Knife injury in hand .. after 2 day swelling and redness? Dx
A. Cellulitis ✅B. Carbuncle
C. clostridium perfringens
.......
🌹Breast lump with bloody discharge what the investigation i think it’s duct ectasia
core biopsy
......
🌹After ERCP pt develop some complication i think nausea and vomiting and may be collecting of fluid but they ask what will
be beneficial decrease mortality?
A. Administration of IV Abx✅
B. repeat ERCP
.......
🌹Loss of sensation in leg= (saphenous nerve)🌹
.....
🌹Organ Enter inguinal canal
= (round ligaments)
.......
🌹most common organ perforated in ERCP
Duodenum
...
🌹 70 male in CCU for an MI, he developed cholycystitis what to do?
A. percutaneous cholecystostomy ✅=  local anesthesia consisting in the placement of a catheter in the gallbladder lumen with
the purpose of decompressing the gallbladder ‫الن كبير بالعمر فصعب نعمل له جراحة‬
B. ERCP C. lab chole
....
🌹 case of appendicular carcinoid tumor 0.5 cm . What your Rx :
A.colectomy B. CT C. No further investigation✅
...
🌹Pic of ecchinococcus cyst ( 2 cyst ) asking for drainage
A. - albendazole B.open surgical deroofing
C. - percutaneous drainage🌹✅
D. - laparoscopic pericystectomy
Echinococcus = Percutaneous aspiration + perioperative albendazole (Torrnot book).
......
🌹40 year old female underwent rouxny bypass surgery, then she developed
left shoulder pain, fever and rigors. Vitals BB: 90/60 , tachycardia and fever,
what is next step in management?
A. CT ✅ B.US C.exploratory laparoscopy
D. endoscopy
...
🌹 surgeon was doing resection of colon cancer then the patient developed
bleeding in pelvis and the surgen packed it, patient become hypotensive
and vascular surgeon what would he do?
A. on table angiogram
B. infrarenal clamping✅
C. celiac artery clamping
D. remove packing
....... ...
🌹Patient post MVA in coma and has absent gag reflex, what type of feeding
will you give him?
A. -gastrostomy B. -jejenustomy
C. -TPN D. -NGT✅
..........
🌹baby with diaphramatic hernia what will you do after stabilization?
A. -surgery. B. -NGT.✅✅
.....
🌹patient after head trauma developed central DI, what will you give him?
A. -vasopressin✅ B.fluid
........
🌹kid obese had trauma then hip pain,
A. x-ray✅ A. -SCFE B. -Legg-Calve-Perthes
....
🌹RTA patient tachycardic pale has tender abdomen with diffuse enlargement. What type of shock?
A.cardiovenic.B.Hypovolemic c-neruogenic.
...
🌹Case of diaphragmatic hernia ( they mention the diagnosis) the baby was
initial respiratory support what the most important next step : Larg NGT
. . ......
🌹3 weeks Post pancreatitis presented with pseudocyst increasing in size and
symptomatic (bloating and palpable mass) ttt?
A. Percutaneous drainage ‫اذا كان السيست ملتهبة وفيها انفكشن نلتجا لهذه الطريقة‬
B. surgical excision
C. internal drainage✅
Pancreatic pseudocyst
pseudocyst visible on ultrasound (US) or CT
Treatment =
1_ possibly self-resolving ( if below 4_6 week after pancritits and Asymptomatic no nedd to intervention its resolve by self )
2_ drainage ( if persistent cyst more than 5 weeks or less if there is symptomatic or complicantion happen like rupture ,
infection ..., there is 3 option of intervention = ( open surgical, endoscopic, or percutaneous) .
......
🌹 Patient head injury post MVA now resucitated but need to transfer to
trauma
A. centre intubate ✅
B. call oncall surgeon in other hospital
.....
🌹 Patient jaundice raised ALP direct bilirubin eoevated wcc elevated no mention of fevere.
A. Choledocholithiasis B. cholingitis✅
.....
🌹Phyloid tumor expanding over 4 months now is 4x4
A. excision✅ B. Observe C. Excision and radiotherapy
.......
🌹CT 70% lesion what to do =
A. percutaneous biopsy
B. Colonoscopy C. Upper GI scope
if multiple lesion >colonoscopy
If single lesion >biopsy
........
🌹 Post cholecystectomy came with pain & abscess in the mid wound, next initial
step & the cause?
A. Wound explore ✅B. Dressing
C. AB
‫ عالج‬9‫اول شيء فحص بعدين‬
..........
🌹Old patient fall decelop CNS symptoms:
subdural haematoma✅
........
🌹 Amoebic liver abscess= metronidazole
........
🌹Pt on folly catheter develop cloudy urine how to prevent: Assess need daily for the catheter
...........
🌹 before removing ingrown nail:
check peripheral pulses
....
Elderly Pt present with abdominal pulsation, investigation ?
A. CT Angio B. duplex US✅
......
🌹 After inguinal hernia repair there is mass -ve cough pulse ?
A. Hematoma✅ B. sarcoma
Answer is: hematoma or seroma
....
🌹 Male adult with perianal swelling and discharge, then ruptured ,Dx:
A. Anal fissure✅ B. hemorrhoid
C. anal abscess D. fistula
.......
🌹management of 3 degree hemorrhoid:
hemorrhoidectomy
....
🌹pt with thigh spider nevi, ask fok cosmatic management.= Sclerotherapy
......
🌹High grade esophageal displasia:
surgical resection🌹
.....
2s
Patient with DM and HTN came with unhealed ulcer for 3 month on the dorsal of
first foot, biopsy taken showed "pseudoepithelial hyperplasia" what's next?
A. Reassurance B. keep pressure dressing
C. depriment of the ulcer✅
D. Amputation of toe
‫ربط = سودوا = كذب نيغى نشيله ونقص الي يكذب‬
.....
🌹Diabetic patient with lateral malleolus ulcer, intact pulse, next?
A. CTA B. venous duplex✅✅
.....
🌹Patient with thigh spider nevi, ask for cosmetic management = sclerotheray
.....
🌹Patient was hit by a truck. Presents now with raised JVP and muffled heart sound:
BP 80/60 HR 110 PCWP borderline low
What type of shock does he have?
A. Hemorrhagic B. Cardiogenic✅✅
C. Anaphylactic D. Septic
......
22 year old male presents with appendicitis, appendocular, 15×10 abcess extends from right iliac fossa to abdominal wall.
management?
A. Percutaneous drain✅ B. Open drain
C. Laparoscopic appendectomy with drain
D. Open appendectomy with abcess drain
‫ افضل شيء لل‬absess ‫ بكل‬9‫انه نعمل له دراين من برة الن فيه انفكشن ماينقع افتح الزائدة من جوه او بالمنظار كذا راح ينتشر الصديد‬
‫ اي حاجة فيها‬... ‫ اسبوع‬١٢ ‫ وناجل العملية ازالة الزائدة ل‬، ‫ البطن وهذا جدا خطر‬absess or infection ‫افضل شيء نعمل دراين من برة‬
‫ عن طريق فتحة بالجلد‬.
.......
🌹 20 year old female presents with RLQ breast mass. It was small in size but increse to 10×8 CM now. It is smooth round not
attached to skin. The skin is shiny bellow it due to compression by mass weight. There is no inflammation or irritation. What is
the most likely diagnosis?
A. Mastitis B. Phyllodes✅
C. Galactocele D. Fibroadenoma
.......
🌹 Elderly female with smal and asymptotic femoral hernia:
A. observation B. open mesh repair✅( bez he is old better do open ) C. lap repair
.....
🌹Pain mechanism in compartment?
A. Nerve hypoxia ✅B. muscle engorgement
.....
🌹After surgery what effect high patch vocal cord?
A. Inferior laryngeal B. Superior laryngeal✅
C. Recurrent laryngeal
‫ من اسمها هاي يعني فوق‬superior.
.........
🌹Typical picture of lymphedema (non pitting limb edema), appropriate test to order?
A. CT angiography B. Vascular US
C. Lymphantoscnigraphy✅
......
🌹 A case of acute limb insufficiency due to atrial fibrillation embolism, what’s the
best next step?‫نبدء بالعالج فورا‬
A. Vascular US B. Heparin✅
C. Morphine D. CT angiography
.......
🍊Hemorrhoids=
(Painless bright red blood) + and its very painful with thrombosed hemorrhoid
= must do (sigmoidoscopy on old patient to roul out colon cancer. )
🍊Anal fissures= tear of anal canal
very painful bright red bleeding especially afer bowel movement
‫ من اسمها‬tear ‫ يعني مؤلم جدا‬.
🍊Anorectal abscesses =
throbbing rectal pain; fever, tenderness on digital examination
🍊 Anal fstula= its fstula from rectum to perianal skin secondary to IBD like crohn or abscess formation
intermittent pain and constant purulent discharge (like stool due to this open ) from perianal opening+ can palpable cord-
like tract.
........
🌹Perianal painful and tender swelling without fever?
A. Anal fissure B. Perianal abscess
C. Prolapsed piles D. Perianal hematoma thrombosis cause sever pain ✅
.........
🌹 Male adult with perianal swelling and discharge, then ruptured ,Dx:
A. Anal fissure B. hemorrhoid
C. anal abscess✅ D. fistula
.......
🌹 Diabetic Pt complaining of anal swelling with continues discharge, he noticed that
before 2 months there was a small swelling and ruptured. What is the Dx ?
A. Abscess
B. Anal fistula✅
# About 50% of patients with an anal abscess will develop a complication called a fistula. ‫فهو يقول انه قبل شهرين كان فيه سويلنج‬
‫ فممكن تطور وحصل‬fistula
# fistula can do Swelling around your anus.
#fistula came with previos hx of absess drain.
#anal fistula is an abnormal narrow tunnel-like passageway, which is the remnant of an old anal abscess after it has drained. It
connects the mid portion of the anal canal (at the anal gland) to the surface of the skin. After an anal abscess has drained (either
spontaneously or when lanced by a physician), an anal fistula will develop at least half of the time. Sometimes the opening of the
fistula at t
.....
🌹20 or 25 year old male come with painful anal swelling on examination 2*2 cm tender swelling No fever Normal CBC, Dx? A.
Abscess B. Hematoma✅ C. prolapsed pile
.......
🌹 Perianal painful and tender swelling with fever?
A. Anal fissure
B. Perianal abscess✅
C. Prolapsed piles
D. Perianal hematoma
.........
🌹 case of repeated hematemesis +
subcutaneas emphysema
A. mallory-wiess syndrome
B. Perforated peptic ulcer
C. Boerhaave syndrome✅
........
🌹Case of appendicitis, what is CI ?
A. propofol B. Diazepam C. NO✅
....
🌹Pt. With ischemic claudication, What is the possible cause of death due to hospital
stay? A. PE B. MI ✅ C. Pneumonia
........
🌹Pt. Fall from a hight, presented with bleeding from ear, otoscopy confirms
perforated tympanic membrane:
A. base if skull fracture✅ B. Subdural
C. Epidural D. Subarachnoid
.........
🌹X-ray showing huge colon around Y shaped shadow with intestinal obstruction
symptoms. What’s step?
A. Double contrast CT
B. Sigmoidectomy with colorectal anastomosis ✅
Answer is: first sigmoidoscopic detorsion then B
.....
Pseudocyst increasing in size (exact size not given). What’s the best management?
A. percutaneous drainage( if cyst become infection)
B. Internal drainage✅
C. Pseudocyst resection
......
🌹Parathyroid adenoma elective resection indication?
A. Age >50 B. Osteoporosis ✅
C. Normal renal function
.......
🌹Female presented with left lower limb swelling. She has NO history of trauma,
surgery or prolonged travel in the last 6 months. On examination her pulses are intact but there’s non pitting edema and
thick skin with dark pigmentation.
the appropriate management?
A. Anticoagulants B. Heparin
C. Lymph drainage by massaging and compression bandaging✅
......
🌹Best modality for clearing cervical spine injury after stabilizing the patient?
A. CT scan✅ B. MRI of the cervical spine
C. AP and lateral x-ray of the cervical spine
......
🌹Elderly patient presented with abdominal obstruction symptoms (No risk factors of ischemia in the Hx). Imaging shows
pneumobilia. diagnosis?
A. gallstone ileus✅
B. Mesenteric ischemia
The classic radiologic sign of gallstone ileus is the Rigler triad (also called Rigler sign): pneumobilia, intestinal obstruction and an
ectopic gallstone.
‫ = ربط = نيوموبيال = نميمة‬ileus = ‫ = تخلي القلب مثل الحجارة‬9‫ اياك والنميمة‬stone .
....
🌹70 kg, bilateral lower limb 2nd degree burn. How would you resuscitate?
‫‪ :‬كيف نحسبها ؟ بالمعادلة هذه‬
‫‪4 x weight in kg x %TBSA burn‬‬
‫‪Give 1/2 of that volume in the first 8 hours. Give other 1/2 in next 16 hours.‬‬
‫‪ ...‬طيب الوزن هنا ‪ ، ٧٠‬نضربها في ‪4‬‬
‫؟ ‪ TBSA‬طيب ايش هو‬
‫كل ذراع بالجسم له نسبة معينة‪ ... 9‬ركزوا بالصورة‬
‫تساوي ‪ . 4‬يعني لو قال بالسيناريوا اليدين انحرقت نضرب ‪ 4‬في ‪ , 2‬لو قال فقط يد وحدة يعني اربعة ونضربها بالمعادلة الي ‪ arm‬كل يد‬
‫‪ .‬فوق‬
‫‪ .‬الراس ‪ :‬الن اهم شيء بالجسم تساوي ‪9‬‬
‫اليد ‪ 4 :‬لكل يد‬
‫الرجل ‪ :‬الن كبير واكبر من اليد نعطيه الضعف = ‪٩‬‬
‫‪ .‬البطن والظهر الن اكبر اجزاء الجسم وفيها االحشاء 😂 نعطيه اكبر عدد وهو ‪18‬‬
19 ‫ يعني االثنين مع بعض تساوي‬، ‫ بالمية‬٩ ‫ كل رجل وحدة تساوي‬.. ‫طيب هنا السيناريوا قال انها الرجلين انحرقت‬
19 × 4 × 70 = 5320
‫ ساعة‬١٦ ‫ ساعات و الجزء الثاني على مدار‬8 ‫ ونعطي الجزء االول في اول‬2 ‫ طيب نقسم الناتج على‬.
‫ ساعات‬٨ ‫ هذا الرقم في اول‬2660 = 2 ‫ قسمناه على‬5320 ‫ يعني الناتج‬.
Give 1/2 of that volume in the first 8 hours. Give other 1/2 in next 16 hours.
🌹70 kg, bilateral lower limb 2nd degree burn. How would you resuscitate?
A. Ringers lactate 200 mL/kg per hour for 24hrs
C. Ringers lactate 5L in the first 8 hrs and 5L in the following 16 hours
D. Ringers lactate 2.5L in the first 8 hrs and 2.5L in the following 16 hours ✅✅
.........
🌹Patient presented to derma clinic. She has a skin lesion 1x2 cm (no details
provided) on her left forearm since she was a kid. What would you do?
A. Follow up✅
B. Laser
C. Excision
..........
🌹Patient known to have motion sickness. (Vomits when traveling by plane).
medication is suitable for this case?
A. Ginger B. Ondansetron
C. Granisetron D. Diphenhydramine✅
‫ ربط = دايفين = دايخين‬. :)
.......
🌹 Family breast cancer core biopsy done and show, Atypical ductal hyperplasia
A. Wide Excision✅ B. simple mastectomy
C. Follow D. Mammogram
is generally treated with surgery to remove the abnormal cells and to make sure no in situ or invasive cancer also is present in
the area.
.......
🌹Smoker and dm with claudication + bottock pain + glutius muscle atrophy + weak femoral pulse ttt?
A. Vasodilator B. iliofemoral bypass✅
C. other bypass
.......
🌹Obese man with GERD and wants to have weight reduction surgery. Endoscopy
showed grade 3 hiatal hernia and GERD. Which surgery is suitable for him?
A. roux en y gastric bypass✅
B. Sleeve gastrectomy
C. gastric banding
‫ = ربط‬y = ‫ يشبه ممر ال‬esophages
.......
🌹Which can help us in choosing the appropriate type of bariatric surgery for the patient?
A. barium swallow B. endoscopy✅
C. CT scan D. Ultrasound
Upper gastrointestinal = endoscopy is performed to rule out intrinsic upper GI disease because after gastric bypass surgery, the
ability to nonsurgically visualize the distal stomach and the duodenum could be a challenge.
.......
🌹pudendal nerve block in sacrococcygeal lig near ischial spine which part is still
not blocked :
A. rectum ✅B. valva C. perineal body
D. urogenital diaphragm
......
🌹Direct q about hemorrhoids tx
Which type treated by sclerotherapy
Answer is Internal grade 1 and 2
......
🌹 Female presented with dorsum of hand progressive redness and fever after knife
prick 3 days ago= Cellulitis
.......
🌹20 yo female came with RUQ pain and jaundice, Total bilirubin high Alt and AST very high What is the mos prognostic :
A. ALT B. Bilirubin c. PT ✅
......
🌹Patient with 10x15 echinococcus liver abscess, what is your management?
A. Metronidazole
B. Percutaneous drainage
C. Percutaneous aspiration
Answer is : Antibiotic + Drain .
.......
🌹A surgeon during surgery, got needle stick injury, the patient is known case of
HCV, what is the percent if transmission?
A. 0.3% B. 3% C. 30%
HCV = 3 ‫ثالث حروف‬
.....
🌹Man present with pelvic pain and incidentally in ct found rt adrenal mass
asymptotic what dx:
A. Adenocarcinoma
B. Non functioning adenoma
....
Pt diagnosed as achalasia after barium she develop leaking of barium with no sign
of peritonitis what’s the mx :
A. endoscopic esophagectomy
B. Esophagus balloon
C. endoscopic stent ✅
....
🌹Young Female patient came with difficulty swallowing and regeneration of liquid and solid Which is the diagnostic test :
A. Barium study B. Upper GI endoscopy
C. Lower esophageal manometry ✅
achalasia
Failure of smooth muscle relaxation at LES
Increased LES pressure
Patients have intermittent dysphagia for solids and liquids, but no heartburn because the lower esophageal sphincter stays
tightly closed and does not allow acid reflux. Barium swallow reveals a dilated esophagus with distal
“"bird’s beak". The diagnosis often is confirmed with esophageal manometry. Treat with calcium channel blockers, pneumatic
balloon dilatation, or botulism toxin injection. Surgery (myotomy) is a last resort. Patients have an increased risk for esophageal
carcinoma.
.....
🌹kid get stabbed at Rt lower chest in US there is fluid accumulation pt stable what’s
the Management:
A. Thoracic Tube B. exploratory
C.. laparotomy D. CT ✅
.......
🌹Mid thirties female came with skin hyperpigmentation on anterior thigh raised :
A. Excision✅ B. Punch biopsy C. Chemo
D. Radio
.......
🌹25 year old patient presents with on off bloody vomiting for the last couple of
weeks. He has no history of chronic liver disease and doesn't drink alcohol. diagnosis?
A. Erosive gastritis B. PUD✅
C. Esophageal varices
D. Mallory Weiss tear
......
🌹 RTA pstient, present with open central neck open wound and tissue out, on
examination: He is conscious and alert, next appropriate management?
A. O2 mask B. Tracheostomy
C. Cricothroidectomy D. Intubation✅
.........
🌹10 weeks lady un the surgical clinic with recurrent right upper quadrant abdominal
pain for 5 weeks when to do lap chole?
A. Immediate B. 2nd trimester✅
.......
🌹28 year old lady, complain of progressive abdominal pain after 3 weeks of gastric sleeve, next step ?
Upper GI endoscope (not sure)
there were no US or CT in the choices.
........
🌹Case about abdominal collection 24x16 after acute appendicitis, what is your
management?Imaging guided drainage
.......
🌹Case of tender mass 1x1cm near to the anal verge, WBCs = 8, what is thediagnosis?
A. Perianal abscess (thrombing pain)
B. Anal fissure
C. Thrombosed biles✅
.........
🌹Patient after hernia repair, developed pain in the medial thighs, management?
A. Nerve block B. NSAIDs
Answer is B then A
......
🌹RTA patient with thorasic aorta injury and splenic laceration in a tertiary center,
patient is hypotensive and unstable, what to do?
A. Laparatomy✅ B. Thoracotomy
C. Refere to vascular surgen in other hospital
......
🌹Most affected site by crohn's disease?
Ilium.
.....
🌹Old lady complain of pain after carbal tunnel syndrome surgery?
Needs physiotherapy
........
🌹 Young man with bilateral inguinal hernia, management? Lap with mesh
........
🌹63 year old lady during annual examibation found to have small asymptomatic femoral hernia management? Open with
mesh
........
🌹Best investigation for AAA? US
.....
🌹Case of a patient has typical wilson disease Kayser-fleishcer ring + low ceruloplasmin What do you want to give him?
penicillamine
‫ دواء يمسك‬copper ‫ الي هو تراكم بالجسم وحول العين ويطلعه برة الجسم‬.
.......
🌹Hiv patient , has fever malaise .
Positive anti-toxicplasmosis IGM antibodies What want to give him?
A. doxycyclin + clindamycine
B. ceftriaxone + ..
C. pyrimthamin+ sulpha✅
........
🌹nurse prick finger of HBV patient and she already vaccinated what to give her?
IVIG
....
🌹patient with TB what the precautions?
Airborne.
....
🌹 patient whith sick euthyroid syndrome

What is sick euthyroid syndrome?


Any patient with any illness may have temporary derangements in thyroid function tests that resemble hypothyroidism. TSH
ranges normal to mildly elevated, and serum T4 ranges from normal to mildly decreased
‫ نفس عالمات‬hypothyroid ‫ بالضبط لكنه مؤقت بعد مايروح المرض يرجع وظائف الغدة للطبيعي‬.
Clinical circumstances and physical findings are the best guides to whether the patient has true hypothyroidism. In patients
with sick euthyroid syndrome, simply treat the underlying illness. If the
diagnosis is in doubt, repeat the thyroid tests after the patient recovers (preferred) or give an empirical dose of levothyroxine
(if the patient does not respond to treatment of the underlying illness).
.....
🌹patient dm and heavy smoker with 100 claudication what should he do to reduce
the claudication?
A. smoking cessation ✅✅‫هذا اهم شي‬
B. sport C. antiplatelet
.....
🌹Scenario of patient heavy smoker with SOB, there is no cough ,decreased
FEV1/FVC, decreased DLCO :
A. Bronchial Asthma B. Emphysema✅
C. Chronic bronchitis D. Lung fibrosis.
......
🌹Scenario of PE with ECG pic. And ask about a highly diagnostic value :
A. D-dimer B. Spiral CT✅
..... ....
🌹 air under diaphragm dx: perforated duodenal.
........
patient with AAA what best investigation of choice ?
A. US ✅ B. CTA
.......
🌹patient with hashimoto what the expected ca ? lymphoma.
.......
🌹patient RTA with leg pain and tense skin with fracture and compartment
syndrome? external fixation and fasciotomy
........
3 weeks baby with scrotal swelling, non tender none erythematous, and negative
trnsilluminate:
ATesticular torsion B. Hydrocele
C. Orchitis D. Inguinal hernia✅
......
🌹 Scenario of gunshot, chest tube inserted and blood loss more than 2 L :
Thoracotomy
.....
🌹 Scenario of Epigastric pain, endoscopy show mass in the stomach, on
histopathology 5 cm Gastrointestinal stromal tumor (GIST) :
A. Partial gastrectomy B. Total gastrectomy
C. wide local excision✅
........
🌹62 years old male with pale, chronic
2nd-grade internal piles, hemoglobin 8 and
(they didn't mention any red flags about the cause of anemia ) :
A. Cecal Cancer B. Sigmoid Cancer
C. Rectal Cancer ✅D. Chronic piles
‫ اي مرة بحياتكم تشوفون مريض ب‬pills ‫ وكبير بالعمر وعنده‬bleeding ‫وجود‬ ‫ الاااازم تستبعدون‬rectal cancer
.......
🌹baby with necrotizing enterocolitis, least associated with NEC?
A. male gender B. full term baby✅
‫ الن هذا غالبا يجي مع‬preterm .
......
🌹Small lesion (7mm) found in asyptomatic, non smoker, young man,
what to do:look for previous CXR
....
🌹 Young woman stab wound in right side mid-axillry of abdomin, the wound is not
too deep (in the abdominal wall) and she is doing fine, vitally stable, what to do:
A. Observation B. CT of abdomen✅
C. US abdomen
......
🌹 Man k/c of GERD, tried all the conservative measures of treatment including PPIs, but with no improvement, what to do:
A. Con’t on PPIs B. Life style modification
C. Nissen’s fundoplication✅= laparoscopic procedure performed for patients with gastroesophageal reflux disease
‫ = ربط‬GERD = ‫ = االكل نايس‬nice = nisse
.........
🌹Pt came for ct angio due to claudication but after 2 days came with protrusion
below inguinal not reducible or expansile , It is plustial:
A. pseudoaneurysm B. spigelian hernia
C. Saphena varix✅
saphenous varix is a dilation of the saphenous vein at its junction with the femoral vein in the groin. Saphena varix is a common
surgical problem. Patient can present with a groin swelling due to saphena varix.
.........
🌹Pt use of non steroidal suddenly came with abdominal pain :
A. chest x ray ✅ B. abdominal x ray
‫ قصدهم انه حصل‬perforate peptic ulcer .
.......
🌹Pt after 7 day of cholecystectomy came with dilatation of cbd 9mm with
gallbladder fossa collection of fluid :
A. Retained stone✅ B. Injury to cbd
.......
🌹Pt after trauma came with decrease of pulse and numbness:
A. Fasciotomy ✅B. Ct angio
.....
🌹Pt have before 2 weeks mi now came with sudden pain in leg and decarees pluse :
A. Arterial thrombotic insufficiency
B. Atrial embolic insufficiency
.......
🌹phylloid breast ca no LN next ?
A. pet scan B. contrast chest ct✅
C. wide local excision.
.......
🌹Woman with a history of Hashimoto thyroiditis, what is the most likely cancer may she develops?
A. Thyroid lymphoma✅
B. Papillary carcinoma
C. Medullary cancer
.......
🌹Medullary cancer in one lobe, what is the treatment
A. Total thyroidectomy ✅
B. Partial thyroidectomy
C. Hemithyroidectomy
.......
🌹Benign phylloid tumor of 3 cm, what is the treatment?
A. Wide local excision ✅
B. Simple mastectomy
.......
🌹A case of bleeding from the nipple. On mammogram, a 3 cm lump was found?
What is the management?
A. FNA
B. Core biopsy
C. Excisional biopsy✅
.....
🌹A case of acute limb insufficiency due to atrial fibrillation embolism, pulse present
in femoral artery but absent below, sensation lost, and with altered motor
function. what’s the best next step?
A. Vascular US B. Heparin ✅
C. Morphine D. CT angiography
......
🌹A female patient with varicose veins wants to treat her condition for cosmetic
reasons, what is the best option?
A. Endovenous thermal ablation (laser and radiofrequency)
B. Sclerotherapy✅
C. Open surgery with removal of a vein
.......
🌹Typical picture of lymphedema (non pitting limb edema), what’s the most
appropriate test to order?
A. CT angiography
B. Vascular US
C. Lymphantoscnigraphy✅
.....
5 Nov 2019
Surg3
🌹hip effusion, best next step
A. Joint fluid aspiration✅ B. X-ray
.......
🌹Sarcoma in thigh staging
A. bone scan B. ct chest✅
C. mri abdomen D. xray leg
.....
🌹underwent tonsillectomy and the procedure take long time due to bleeding and patient recover will after the operation but
concern why it took long time :
A. Reassure and tell the pt what happen✅
B. tell other doctor to tell pt
....
🌹Man while playing football the ball hits his chest after that he developed SOB , chest pain on x-ray the trachea was shifted ,
what is the Dx ?
A. tension pneumothorax✅
B. cardiomyopathy
....
RTA pt came with mandibular and maxillary fracture?
A. nasal intubation B. cricothyrotomy✅
C. oral intubation
If adult >B If child>tracheostomy
........
🌹Risk of breast cancer ?
A. age✅ B. early menarche
C. late menopause
.....
🌹 Most common shoulder dislocation (posterior dislocation)
.......
🌹 treatment of anal fissure and anal fistula with pus discharge from the fistula >if associated with access you should drain first
while if only fistula you do
fistulotomy
........
🌹spider nevi TTT needs cosmetic treatment and best investigation> sclerotherapy and US for investigation
......
🌹Loss of sensation in leg (saphenous nerve)
......
Biared 5 of breast
A. core biopsy✅B. excisional Biopsy
........
🌹 After ERCP pt develop some complication i think nausea and vomiting and may be collecting of fluid but they ask what of the
following will be beneficial to decrease mortality?
A. Administration of IV fluid✅
B. repeat ERCP
.......
*Ventral hernia? Subly
....
Medullary thyroid carcinoma, how to manage? A. Total thyroidectomy✅
B. Partial C. Radiation
....
🌹Pt came from a recent trip, recalls frequent falling, xray normal , the dx?
A. Concussion
B. Subdural hematoma✅
.......
🌹Pt admitted to icu. Best way to feed him :
A. NGT✅ B. Gastrotomy C. Gastrostomy
.......
🌹Cpt stroke and absent gag reflex pt admitted to icu . Best initial way to feed him :
A. NGT✅ B. Gastrotomy
C. Gastrostomy D. Central line
......
🌹Pt with appendicitis + fecalith + abscess 10*15 cm ask about the Rx ?
A. open drainage B. lap drainage
C. open appendectomy with drainage
Percutaneous drainage✅
.........
🍄Old women with history of bilateral varicosities for 6 years. Now complaining of pain and heaviness in the lower limbs
Intact pulses in both legs
What is the best initial investigation?
A. CT angio B. Conventional CT
C. Venous duplex✅D. Venography
.........
🍄Patient present to the ER of a hospital lacking surgical facilities with tension
pneumothorax and femoral fracture. The resident on duty place a chest tubes to
stabilize the patient then move him to a trauma centre. In the way to the
ambulance the patient developed sudden sob, tachycardia, tachypnea
best initial step in management ?
A. Proceed in transforming the patient
B. Rapid response intubation
C. Check the position and patency of the chest tube ✅
D. Stop the transforming
.......
🍄Pt who is a smoker came with spontaneous pneumothorax, what is the best way to avoid recurrence?
A. Stop smoking✅
B. No travel by airplane for 12 months
.....
🍄The patient did cholecystectomy
post-surgery day 5 I think he developed
abdominal pain & distention.By examination: tender distended abdomen. US: shows ascites ( I can't recall) How do you
manage?
A. percutaneous drainage✅
B. ERCP C. Exploratory laparotomy
D. diagnostic laparoscopic
If more than 4 cm> percutaneous drainage
Less than 4 cm>abx and observe
......
🍄Pt post appendectomy presented with 2x2 cm collection confirmed by ct on
retrocecal, no fever no n/v, how to manage?
A. Percutaneous drain
B. Conservative treatment✅ C. Open drain
......
🍄Phylloid tumor 2 questions
A. One >10
B. One <10
‫من كتاب الجراحة‬....Phyllodes tumor.....
Treatment of phyllodes tumours, whether malignant or berhgn, is wide excision. If the lesion is large, mastectomy may be
needed to ensure complete removal.
........
🍄Direct and total bilirubin are high Amylase high Liver enzymes I think were within normal range (I don’t remember exactly)
Patient is jaundiced and has picture of gallbladder disease. Asking about diagnosis:
A. - Acute chole B. - pancreatitis
C. - cholangitis ✅D. - hepatitis
.....
🍄known case of Varicose veins came with swilling , pluses intact ،test to do?
A. no need for test
B. venas ultrasonography✅
......
🍄Gallbladder polyp 0.6 mm , how to manage? follow up in six months
...
🍄Patient with metastatic bronchogenic tumor with recurrent pleural effusion with
multiple pleural tap in the past: What is the beat for management:
A. Thoracocentesis B. Chest tube
C. Chemical pleurodesis✅
‫ ربط = ورم منتشر‬metast = ‫ = يحتاج عالج كيماوي‬chemical
......
🌹History of RTA Fx in pictures, tt ?
A. debridement +external fixation + nail
.....
🌹minimum duration of PPI post endoscopy = 72hrs.
‫ = ربط‬ppi ‫ ثالث حروف = ثالث ايام‬.
......
🌹Trauma with blue discoloration and gas within the wound what type of bacteria?
A. Clostridium perfringen
......
🌹After appendectomy done for pt the histology result find 0.5 tumer in the
appendix, what is your mangment?
A. - nothing to do ✅B. - endoscopy
.....
🌹 23 yo male, after accident has tibial fracture and manged by closed reduction and cast .. after that he developed toe swelling
and pain Whats the earliest alarming sign?
A. - pain ✅B. - foot drop
C. - diminished pulse
D. - loss of sensation btw toes
......
🍄ER pt with head trauma stable need to be transferred to 40km hospital.Report
shows : Widening of mediastinum no pneumothorax: Pt stable, What to do next:
A. Intubation ✅B. Chest tube
C. 2 unrelated
......
🍄Patient with progressive unilateral non-pitting lower limb edema with skin
discoloration into darker color. Pulse intact. (Detailes toward lymphatic
problem)What is the BEST imaging modality?
A. Lymphoscintigraphy✅
B. CT angio C. Duplex US D. MRI
.....
🍄Same Q but what is the NEXT step? (Lymphoscintigraphy wasn't in the choices
this time) A. US
...
🍄 Same Q asking about Mx= lymphatic massage
......
🌹 Sarcoma ?excisional biopsy
.....
🍄Angiodysplasia Mx>argon cautery
..
🌹Varicose vein cosmetics tx= sclerotherapy
......
🍄 Patient with suspicious breast lump, BIRAD 4. What is the appropriate action?
A. Needle core biopsy B. Excisional biopsy
.......
🌹 Patient with humoral and ulnar fracture and fixed, presented after a while with
difficulty in extending the wrist and fingers. What is injured nerve?
A. Ulnar due to fracture B. Median
C. Radial due to humoral groove fracture✅
......
🌹It was clearly written echinococcus case but CT with 2 cysts not1 treatment?
A. Albendszole ✅
B. Open something deroofing
.
echinococcus cyst = In asymptomatic patients, small calcified cysts may requim no treatment.
Patients can be treated successfully with albendazole or mebendazole but this may be prolonged. Large symptomatic cysts are
best managed by complete excision,
together with the parasites contained within. A laparoscopic approach is possible for simple accessible cysts. Princeple surgery
book
......
🍒Indication of surgery in vascular malfunction in leg = Pain.
.........
🍒Hemangioma growing on the thigh what is an indication for surgery?
A. pain✅
B. cosmetic
..
🍒MVA with facial trauma and evidence of brain injury.
A. secure airway B. stabilize neck✅
C. iv fluid
....
🌹 Varices with upper GI bleeding, hemoglobin was a little bit low, first line of
management:
A. Blood transfusion B. Colloid fluid
C. Normal saline✅
..
🌹A smoker with history of GERD, Upper GI shows high dysplasia in the lower
esophagus, what it the recommended treatment?
A. PPI B. Follow up in 6 months
C. Treatment of H. pylori
D. Esophageal resection✅
....
🌹A patient starts to have dysphagia to liquids, what is the investigation of choice:
A. Upper GI endoscopy✅
B. Barium swallow
C. CT scan
D. X-ray
.......
🍒Gastrointestinal Stromal Tumour of 5 cm, what is the management?
A. Endoscopic ablation
B. Wide local excision✅
C. Partial gastrectomy
D. Total gastrectomy
......
🌹Duodenal perforation? Best treatment?
A. Omental patch ✅B. Resection
......
Pt with prosthetic valve and should do hernia repair prophylaxis:
A. Amoxicillin B. No need✅
......
🌹 Patient intubated in ICU, most likely ulcer he will develop?
A. Stress gastritis ✅B. Duodenal ulcer
. . ...
🌹A morbid obese who should do a bariatric surgery, what investigation you are
going to do before surgery:
A. Abdominal US✅
B. Abdominal CT scan
C. Barium swallow
D. Upper GI endoscopy
...
Asymptomatic unilateral femoral hernia in a woman, what is the best choice of
management:
A. Observation
B. Open repair with mesh
C. Laparoscopic surgery
Answer is B
. ....
🌹Strangulated hernia in lateral and bellow pubes tubercule?
A. Femoral hernia✅B. Indirect hernia
C. Direct hernia D. Obturator hernia
Answer is A
........
🌹 Recurrent biliary colic in gestation week 10, management?
A. Immediate cholecystectomy
B. Cholecystectomy in 2nd trimester✅
C. Cholecystectomy in 3rd trimester
D. Cholecystectomy after delivery
.......
🌹 Child with Grade 3 spleen injury and 7 cm hematoma? Best treatment?
A. Observation B. Spleen preserving surgery
C. Splenectomy
........
🌹 Nausea, vomiting, and left lower quadrant pain with WBC of 19,000 and
thumbprinting sign on imaging?
A. Ischemic colitis B. Diverticulitis✅
C. Ulcerative colitis D. Crohn disease
.......
🌹Appendicular abscess larger than 2 cm, management?
A. Antibiotics
B. Percutaneous drainage✅
C. Appendectomy with drainage
......
Angiodysplasia, best treatment?
A. Embolisation B. Laser ablation✅
C. Resection
.....
A case of intussusception, best diagnostic test:
A. Abdominal US B. X- ray C. CT scan
D. Barium enema✅
......
🌹A 4-day old patient with persistent vomiting (did not mention if it is bilious or not!), what is the diagnosis:
A. Duodenal atresia ✅B. Pyloric stenosis
C. Hirschsprung disease D. NEC
......
🌹A case of Congenital diaphragmatic hernia, after the infant is stabilized, what you should do?
A. Gastric decompression✅
B. Immediate surgical repair
......
🌹 Chest trauma of with elevated JVP, normal bilateral entry, and hypotension:
A. Cardiac tamponade✅
B. Pulmonary contusion
C. Pneumothorax
..
🌹 Wound infection management after exploration?
A. Open drainage B. Antibiotics✅
C. Laparotomy
.....
🌹Classical triad of Asc.cholangitis, what is the appropriate next step:
A. Us B. Ct C. MRCP D. Ercp✅
......
🍒 Abdominal X Ray shows thump-print sign: Ischemic colitis
....
🌹 lung infiltration in Sickler pt with several symptoms and asking about dx :
Acute chest syndrome ✅
.......
🌹 Elderly hg 7 on ex rt iliac fossa mass:
A. Colonoscopy
....
Neck injury zone one: CT
.....
🌹 In mid thigh (soft smooth mass negative transluminal):
A. lipoma B. seabeacyst✅
Answer is B but incomplete
....
🌹Elderly 65 years old came with reducible inguinal hernia ttt ?
A. open repair with mesh ✅
B. laparoscopy repair
......
🌹pt has (echinococcus ) hydatid cyst , most appropriate ttt ? (( no size I swear))
A. metronidazole
B. surgical excision percutaneous drainage
C. deroofing surgery
D. Albendazole therapy✅
Answer is D in general
.....
🌹pt with crohn's disease has perianal pain and purulent discharge , WBC high , best next step :- diagnosis is fistula.
A. empirical broad abx
B. swap culture of perianal
C. surgery
D.fistuloctomy ✅
.........
🌹pancreatic calcified and fat in stool .. what to do ?
A. tell him to decrease fat intake✅
B. give lipase 30000 with each meal
.......
🌹Old patient >50 years old. Bloody from rectum and weight loss with abdominal pain . Physical is normal. Best Next step in
management?
A. Sigmoidectomy B. Colonoscopy✅
C. CT abdomen
.......
🌹Female 57 year old presented with severe left lower abdominal pain, Vomiting, and abdominal distention for 6 hours , WBC
high , diagnosis:
A. mesenteric ischemiaB. Perforated viscus
C. diverticulitis✅ D. peptic ulcer
.......
🌹Scrotum red but not tender , horizontal lie: A. surgical exploration ✅B. ABx
C. observe D. US
........
🌹Eldely, weight loss , vague epigastric pain, loss of appetite, Most appropriate
investigation :
A. Gastroduodenoscopy✅B. Abdominal CT
.......
🌹Case of achalasia after esophageal dilation procedure, complicated with rupture management:
A. esophageal stent B. Esophagectomy✅
C. surgical drainage
.......
🌹Case of esophageal high grade dysplasia
A. refer for possible surgical resection
... .
🍒67 years old male bedridden in hospital for many years because of recurrent strokes is found now to have sacral ulcers. On
examination ulcer was involving the sacral and gluteal region with necrosis of the skin and subcutaneous tissues. What is the
best management plan?
A. Debridement with dressing
B. Debridement with primary closure
C. Debridement with skin graft✅
.....
🍒child pass stool after 73 h as yellow and delayed Dx as intrassuption , What is the most important appropriate management :-
A. resection B. Steroid
C. Decompression by Enema✅
........
🍒patient underwent to the surgery after that he is bleeding from different area From the dressing , and skin of incision , what is
the most likely cause :-
A. thrombocytopenia
B. Von will brand factor
C. transfusion reaction✅
.....
Patient with painful mass diagnosed as strangulated hernia, at the surgery the neck of the hernia was found to be inferior and
lateral to the pubic tubercle, what the type of this hernia?
A. femoral hernia✅
B. Direct inguinal hernia
.....
🍒Child with inguinal hernia without family ( abundant) and Doctor see him and decide to do surgery. What to do?
A. Do surgery without consent
B. talk to hospital committee✅
.....
🍒Pt with swelling and chronic cough you prescribed medication 6 month ago and he did take it and come now he said if you
don't do the surgery I will Complaint against you. What to do?
A. Do surgery B. refuse to do surgery
C. talk to hospital committee✅
.......
🍒Lobectomy and 9mm papillary carcinoma left distant from the removed lobe, no lymph or vascular invasion
A. observe/follow up B. Radioactive✅
C. Total thyroidectomy
......
🍒Trauma patient has basal skull fracture that includes the jugular foramen what will the patient have :
A. Hoarseness of voice✅
B. Paralysis of the muscles of mastication
.........
🍒 pt present with massive pleural effusion causing progressive SOB, caused by
malignancy
A. chest tube with under water drain
B. thoracentesis
Answer is chemical pleurodosis
.......
🌹RTA pt thoracic aorta injury and splenic laceration and hemodynamic unstable :
A. Thoracotomy B. laparotomy✅
C. aortic clamp in 2 arteries
.....
🌹Trauma pt , sign ... air entry in right , low gcs what is most urgent action
A. rt thoracostomy✅
B. Intubation
........
🌹Head injury no GCS in the question, went to primary health care with limited facility and nearest trauma center is 40km you
did imaging no pneumothorax and 2 broken ribs, in the meantime, what to do?
A. call surgeon in trauma centre
B. intubation now ✅C. insert chest tube
D. Pericardiocentesis
.....
🌹Pelvic trauma pt and blood in urethra, imaging show retroperitoneal urethral injury, what to do?
A. folycath B. suprapubic cath✅
C. cystoscope
......
🌹Anal mass couliflower 2 cm away from anal verge , wight loss and change bowel habit for 2 month ?
A. anal cancer ✅B. condylomata
C. rectal cancer
.......
🌹Pt did rigid sigmoidoscopy and biopsy and found Adenocarcinoma what next?
A. colonoscopy ✅B. pelvic MRI
.......
🌹Breast mass mobile painless and pregnant in 32 yrs what to do after examination ?
A. Bilateral US✅ B. Bilateral Mammo
C. Reassure and exam after delivery
.....
🌹Thyroid nodule and FNA, show follicular cells (FLUS)..what to do?
A. hemithyroidectomy
B. excisional of the nodule
Answer is core needle biopsy
P.s: you can't differentiate between follicular benign and malignant so you need to take so you
need to take more biopsy
......
🌹Elderly female have limb pain with walking increased with walking down hill , decreased with walking up hills , pain get so
severe to stop her from walking :
A. venous insufficiency
B. lumber stenosis✅
......
🌹Case of ruptured spontaneous pneumothorax (tall thin pt after landing from plane sudden chest pain and dyspnea ) mx:
A. chest tube B. Blebectomy
C. Needle thoracentesis✅
..........
🌹Case of post acute biliary pancreatis after resolution of symptoms and there is still small hepatic stones but no symptoms
and asking about tho next step
A-ct abdomen B-ERCP ✅
........
pt develope breast mass ( give labs results) , then ask about type of plural effusion?
A- exudate due to malignancy ✅✅
B- transdate due to malignancy
C- exudate due to CHF
.......
🌹What crletrla to identify perl-opertalve mortality?
A-American society of anesthiolgists (ASA)
........
🌹Pregnant 36 yrs with hx of mobile breast mass about 3 cm what to do?
A-Bilateral US B-Bilateral Mammo
C-Reassure and re-evaluate after pregnancy ✅
......
🌹pt has difficulty in breastfeeding due to unilateral nipple Inversion on examination we even the nipple and find transverse
slitted nipple what us the diagnosis?
A-Breast ca B-Duct ectasia ✅
.......
🌹pt did emergent lap chole came after 2 weeks vague abdominal pain imaging showed large sub hepatic fluid collection what
you will do?
A-Ct guided drainage ✅
B-Operative drainage
C-Laparoscopic drainage
D-ERCP with biliary stent
.....
🍒pt came to do lap chole but they discovered that he had 4.5 AAA so they refer him the vascular surgery what they will
recommend the best?
A-Lap chole then us follow up ✅
B-Lap chole then ct angio follow up
C-Ct angio then do the surgery
D-Conventional angio then do the surgery
.....
🍒young pt has thyroid lump moves with swallowing after all laps she had single 2-3 cm hot thyroid nodule the TFT revealed
hyperthyroidism but she didn't had any symptoms of hyperthyroidism best initial ?
A-Rt thyroid lobectomy
B-Radioactive ablation
C-Anti thyroid medication ✅
D-Total thyroidectomy
.If there is scan more accurate ...
........
🍒Post appendiclctomy with seroma and no signs of Inflamation in the wond what to do / A-order ct B-daily dressing ✅
.......
🍒Case of compartment and treatment
-extend fixattiin abd faciotomy
.......
🍒Patient can't do dorsal extension for hand and forearm =Radial nerve injury
Drop hand
.......
Case of gall bladder thickness, pericystic fluid, no gall stone Treatment?
A-Us guided aspirations cholecystectomy✅
B-Emergency cholecystectomy
......
🍒When to do emergency cholecystectomy in Acalculas ?
Gallbladder necrosis+perforation ✅
'Ttratment :
-broad-spectrum antibiotics+ cholecystectomy
-If patient unstable cholecystostomy
........
🍒symptoms of appendicitis then they give xray of a rounded white opacity measure 1 .5 cm in right side just above the pelvis
most likely a stone. What is your next step?
A-appendectomy✅B-CT C-refer to urologist
.........
🍒Air under diaphragm = perforated peptic ulcer =Pancratic Infarction in ICU What will change in his physiology
A. Decrease resistance to insulin
B. Hypoglycaemia✅C. Increase lipolysis
.....
🍒Fast growing breast mass it showed to be Malignant phylloid tumor wat to do ?
A-wide local excision B- pet scan
C -hormonal receptor D-ct chest ✅
.....
🍒 Most common site for basal cell carcinoma?
A- face✅B- lower lip C- upper lip
........
🍒Male with I day abdominal pain , dlstenslon ,constpatlon ,diagnosis of Small bowel obstructing, what is manegment:
A-Small bowel restraction ✅
B-Small bowel C-Brim enema
..........
MVA high speed patient conscious , wearing seatbelt (CXR : normal) what is best next step :
A-abdomen US
B-abdomen CT ✅
......
Case of hernia. patient came with swelling, redness and tenderness on touch, negative cough impulse. Type of hernia?
A. Strangulated hernia
B. Incarcerated hernia ✅
Toxic appearance, leukocytosis, instability, gangrene, any feature of ischemia in imaging = strangulated
.....
Patient had Thyroid or neck surgery, came now with hoarseness. Nerve injured?
A. Recurrent laryngeal nerve ✅
'Superior laryngeal loss of high pitched sound--p Hoarseness
......
MVA sign of tension pneumothorax , GCS 8: best initial step = needle decompression.
........
🍄injury to the head exactly to the left jugular foramen= -left vocal cord paralysis
.......
🍄patient S+S of what is most appropriate diagnostic test: A-US ✅ B- CT
....
🍄Pelvic truma pt and blood in urethra, imaging show retropretonial urethral injury what to do?
A- folycath B- suprapubic cath ✅
C- cystocope
.......
🍄Gs surgeon did LAR and massive bleeding he called vascular surgeon, what is first thing to do?
A- pelvic packing ✅
B- immediate bedside angie
C- clamping something
.......
🍄Breast mass mobile painless and pregnant in 32 yrs what to do after examination ?
A-Bilateral US ✅ B-Bilateral Mamma
C-Reassure and exam after dilevry
........
🍄Female 57 year old presented with severe left lower abdominal pain, Vomiting, and abdominal distention for 6 hours most
likely diagnosis
A-mesentric ischemia B-Perforated viscus
C-diverticulitis ✅D-peptic ulcer
.......
🍄"simple pneumothorax" as I remember around I cm, ask about mgt :
A- Observation✅ B- thoracotomy
C- needle decompression D- chest tube
‫ ال‬simple pnumo ‫مايحتاج نعمل له شيء بس اوبزيرف‬.
.....
🍄RTA pt thoracic aorta injury and splenic laceration and hemodynamic unstable :
A- Thoracotomy B- laprotmy ✅
C- aortic clamp ...In 2 arteries..
.......
🍄Female asymptomatic, us no CBD stone , normal biliary tree, but gallbladder polyp 0.2 cm what to do next?
A- reassure
B- follow up imaging after ✅
c- order LFT D- surgery consultation
....
Pt did rigid sigmoidscopy and biopsy and found Adenocarcinoma what next?
A- colonoscopy ✅B- pelvic MRI
......
🍄Thyroid nodule and fna, show follucalr cells (FLUS)..what to do?
A- hemithroidectomy
B- excisonal of the nodule ✅
.....
🍄Anal mass couliflower 2 cm away from anal verge , wight loss and change bowel habit for 2 month ?
A- anal cancer ✅B- condaylomata
C- rectal cancer
......
🍄Case of ruptured spontaneous pneumothorax (tall thin pt after landing from plane sudden chest pain and dyspnea ) mx:
A- chest tube✅ B- blebectomy
C- thoracentesis D- observe
.......
🍄Elderly female have limb pain with walking Increased with walking down hill , decreased with walking up hills , pain got so
severe to stop her from walking :
A- venous insuffecincy
B- lumber stenosis ✅
.....
🍄Pt with cranial n ocular manifestations and lower limb neurological manafistaions and ask about investigation?
A- brain and spinal cord MRI ✅
B- thoracic spine CT
.......
🍄Man came with abdominal pain and after investigation showed that he have hydatid cyst in liver 6x5
A-surgery ✅B-mebendazole
......
🍄anal fissure : Lateral internal sphincterotomy
-celiac disease ; subtotal atrophy
(Villus atrophy ,Crypt cell hyperplasia, Lymphocytic infiltration)
....
🍒Pt with cranial n ocular manifestations and lower limb neurological manafistaions and ask about investigation = MRI
......
Suspected appendicitis with faecolith and abscess 2x2, managment
A-open drainge B-ABX
C-open appendictomy with drainge✅
D-lap appendictomy with drainge
‫اي شيء فيه انفكشن ال يحبذ نستعمل معهم المنظار‬
......
Phyllodes tumor of the breast ?
A-wide local resection ✅B-Conservative
...
Pt showed esophageal cancer located at the mid esophageal tract:
A-Adenocarcinoma
B-Squamous cell carcinoma ✅
......
Elderly present with weight loss and appednicular mass
What is the most appropriate invistigatlon?
A-Abdominal CT B-Colonoscopy
.......
🍒Eldely, weight loss , vague epigastrlc pain, loss of appetite Most appropriate Invistigatlon :
A-Gastrodeunoscopy ✅B-Abdominal CT
.........
Pt going to do lap-surgry and x-ray shows collapsed colon which of the following contraindicated
A-propofol B-ketamin C-sevoflurane
D-nitric oxide ✅
...........
management of subacuto thyroiditis?
A-PTU B-Iodine C-prednisone D.Nsaids
How is subacute thyroiditis treated?
Ÿ Over-the-counter nonsteroidal anti-inflammatory drugs (NSAIDs). Medications like aspirin and ibuprofen work by reducing
inflammation. ...
Ÿ Corticosteroids. Corticosteroids are used when NSAIDs aren't enough to reduce swelling. ...
Ÿ Beta-blockers.
...
Clinical features of appendicitis and ask about what appropriate Investigation?
A-CT✅ B-US C-MRI
If next step ) US
If best or most accurate ) CT
........
Emery pt k/c/o DM not insulin dependent c/o swelling in posterior aspect of the neck with multiple openings ooslng pus for 2
days =
A.access B. Cellulitis
C.furuncle D.Carbuncle ✅
......
Pt with cellulitis on the eye. What to do next? CT
.......
🍒Transmural lesion =
A-Colon B-Sigmoid C-Rectum D-llium ✅
........
Finger hyperextending pt can't move top of finger with swelling and pain =
Flexor Digitorum profundus
Flext VS extend
.....
Pt with thyroid nodule one is cold the other was hot what is the initial management
A-Hemithyroidectomy
B-Total thyroidectomy
C-Radioiodine ablation ✅
.......
Old patient planed for lapchole was on IV fluid because of poor oral Intake, 2 days post op she developed sob, on PE bilateral
basal crepitatlons , how ro prevent such a complication
A- daily reassessment of fluid level ✅
B- give furosemide
.......
Patient under went surgery for varicose veins. 2end day she has paresthisa in Iner aspect of thigh, examination shows Loss of
light touch and temperature in the limb ( did not specify if medial thighs or all limb) mussle power normal what is affected nerve
:
A-Femoral B-Sciatic C-Sphenous ✅
...
femoral hernia = below and lateral to the pubic tubercle
......
🍒gastrografin leak +ve ask about Mx = esophageal stent 🍒
.......
🍒Pt did rhinoplasty, he always complaining of mandibular and axillary deformity in face. He went to 2 plastic surgeons but
both said he is normal. Dx?
A- Body dysmorphic disorder ✅✅
B- Hypochondriasis
........
🍒Hernia case, male with chronic cough for 6 months, came to surgery OPD, threatening the doctor he will do a complaint
about you if you didn't do the surgery. What to do?
A- Inform administration
B- Refuse to do surgery C- Do surgery
D. Referral to another doctor. Any argument between patient and dr ) referral to another. ✅
..........
🍒RTA, crushed limb, skin and SC tissue damaged. Nerves and vessels exposed. Best management?
A- Debridement and skin grafting ✅
B- Debridement and vacuum
C- Unrelated other 2 choices
N.B: if there is flap better.
..........
🍒Child 10 years with inguinal hernia?
herniotomy up to 12 years, mesh repair for boys more than 12 years. ✅✅
.......
Asymptomatic patient had imaging for lipoma in the neck and incidentally it
shows 2 masses in thyroid 1 in each lobe, one of them was larger than the other
what to do? Fine needle biopsy for both.
....
patient lower parathyroid glands removed because of primary parathyroidism then
presented 4 months later with bone and muscle pain and fatigue, parathyroid was
elevated, ca elevated, phosphate nomial what she has:
A. new adenoma B. recurrent C. cancer
D. previous lesions where not fully removed ✅
........
🍒Patient with breast lump and family history of breast cancer on examination, hard lump on the upper lateral quadrant no
lymphadenopathy best initial investigation:
A. Core needle biopsy
B. Needle aspiration
C. Bilateral mammography ✅
.........
25 y Old come on RTA unresponsiveness ,severe RD on auscultation there hyperresonant rt.lung and GCS is 8 what is the first
line to do
A. needle decompression ✅
B. endotracheal tube insertion
C. Thoracotomy
....
🍒Male infant presenting with a 2-day hx of unilateral scrotal redness and
tenderness. Transillumination test is negative, and on examination skin is thick
and red and shiny.Imaging shows a thick and swollen cord.
A. Epididymo-orchitis ✅
B. Incarcerated hernia C. Testicular torsion
D. Torsion of testicular appendage
.....
A man was shot in the chest and the bullet went through lateral part of sternum
and exited from under the scapula. Chest tube was inserted that relieved mild
hemothorax. Was stable and suddenly became hypotensive and heart sounds
were muffled. (Cor pulmonale?) what is the next best option?
A. immediate thoracotomy
B. Thoracocentesis
C. IV fluids D. pericardiocentesis ✅
......
🍒A man was in a high speed MVA and received a head injury; was resuscitated
immediately. Due to lack of resources in local hosp needs to be transferred to a
more specialized center 40 km away. How to transport him? A. IV fluids B. intubate✅
....
Female with headache which is progressive increasing on CT there is 11'13
macroadenoma what is the next investigation
A. MRI brain ‫خالص عملنا سي تي‬B. hormones of anterior pituitary ✅
......
🍒Patient with solitary nodule on the right lobe TSH:0.05 ( decreased ) T4:normal
next step ? FNA
........
atrial fibrillation
..
4s
🍒mammogram showed a lesion I think it was 2x2 or something and suspected
malignant and they asked what is the next step:
A. core biopsy✅ B. excision biopsy
C. mastectomy D. chemoradiation
.......
Patient with multinodular goitre after surgery he has severe drop in serum calcium level refractory to oral and Intravenous
calcium replacement what l your plan of management:
A. Give him PTH injections
B. Measure serum magnesium
C. Give KCL
........
🍄Post cholecystectomy 10'15 cm appropriate ttt?
A. US guide drainage✅ B. Antibiotic
....
🍄25% Total body surface area(TBSA) burn case what would be present ?
A. Reduce urine output
......
🍄Pt develop spontaneous pneumothorax resolved after two weeks without
intervention what to advise him to not occur again
A. Not to go on an airplane for one years
B. Not to play football for one year
C. Avoid travel I week
.......
🍄Pt post thyroidectomy hypocalcemia on IV calcium and oral and still hypocalcemia
A. check MG level ✅
B. potassium something C. give pth
....
🍄Old man with increase amylase enzyme about 300 Xray Shows dilated and
distended loop but large bowel normal
A. Acute pancreatitis
B. perforated peptic ulcer
C. bowel obstruction ✅
D. ischemic mesentery
......
🍄Bilateral varicose vein causing heaviness, how to treat?
A. Sclerotherapy B. embol
Endovascular ablation
...........
🍄Thyroid Nodule, normal FSH and T4. US: Benign looking, bilateral LN involvement?
A. Observe/Follow up B. FNA ✅
.......
🍄Lobectomy and 9mm papillary carcinoma left distant from the removed lobe, no lymph or vascular invasion:
A. observe/follow up ✅B. Radioactive
C. Total thyroidectomy
.............
🌹Intussusccption:=Shock is common complication 🌹
......
🍄Patient with history of Bilateral claudications when walking for 200 m came to ER with sudden severe pain in left leg no
popliteal pulse, other leg no distal pulse or diminished, what is the best next management?
A. Heparin B. CT angio C. Catheterization
.......
🍄Penetrating neck trauma anterior to ear at the angle of mandible pt is conscious
and alert , the next step ?
A. CT Angio✅ B. Surgical exploration
..........
🍄Time of IV PPI after endoscopic treatment of UGI bleeding
A. 24 h B. 36 h C. 48 h D. 72 h ✅
.........
🍄Breastfeedinq women present with nipple inversion and transverse slittinq ?
A. Duct ectasia
.....
🍄Female (age under 40 thirty something) usually have bilateral breast tenderness
before menses , with multiple mass ' Mamo show bilateral hyper dense tissue , US show bilateral multiple cyst component
and 2 solid mass in rt , Dx?
A. fibrocystic breast ✅ B. fibroadenoma
C. cancer
......
🍄37 yo with FHx of Breast Cancer complain of breast mass o/e irregular and nipple retraction what ta do next ?
A. Breast MRI B. Bilateral US C.mammogram✅
.......
🍄Female after carpal tunnel complain of complex regional pain , she have been
reviewed by Orthopedic doctor and pain doctor , what it appropriate Mx?
B. physiotherapy✅ C. triptan
.....
🍄case of elective choly and the surgeon find large stomach mass What to do ?
A. Choly ✅ B. Stop surgery
C. Mass resection
D. Mass resection and lap coholy
......
Patient admitted for Rt Adrenalectomy because of Cushing what to give ?
A. Post op fludrocortisone
B. Pre op Corticosteroids ✅
....
🍄did open hernia repair 3 years ago and now he has a hernia what to do ?
A. Lap with mesh
.......
🍄in Compartment syndrome what's the cause of pain ?
A. Hypoxia to the nerve ✅
B. bone ischemia
C. Muscle engorgement
......
🍄35 Mother can not breast feed his baby due to nipple Inversion With examination, you tried to return it to normal,
however...the nipple still in transverse way ..what is the diagnosis?
A. Duct ectasia ✅B. breast cancer
.......
🍄 alarming sign in compartment syndrome: Pain
...
🍄Pt with spider vein in her leg came want cosmetic therapy
A. do sclero B. do venous duplex
......
🍄Old age male with slight retention, Examination and investigation is normal except middle prostate enlargement, what is
next?
A. Renal function test annually
B. PSA ✅C. Cystoscopy
..........
🍄RTA pt high velocity in small hospital needs to go other hospital before transfer
find wide midstaing chest and "no pneumothorax " he is hypotensive what you will do before transfer?
A-chesttube B-intubate
....
🍄Bi lateral Great saphenous varicose vein with sx = endovenous ablation✅
.......
Abdominal pain nausea vomiting jaundice , ct show air in the biliary tree , diagnosis?
A. Pancreatitis B. Cholangitis
C. Cholecystitis D. Gallstones ileus ✅
......
patient k/c of hyper triglyceridemia presents with 4h history of sever epigastric
pain radiate to back with N&V (x ray description was written) shows air in (right or left I forgot ) upper quadrant intestinal loop
Labs show , High AST & ALT
Upper normal amylase 150 (Nz151 )
Upper normal lipase , LDH was given
What's the diagnosis?
A. Mesenteric ischemia
B. Acute pancreatitis ✅
.........
🍄Hyperthyroidism + 2 mass with different sizes
A. Scan
B. fine need for the bigger one
C. fine need for both ✅
according to prof.surgeon ) FNA both (I have no evidence) Mohtadi.
......
🍄Case of cholangitis treatment?
A. ERCP ✅B. cholecystectomy
C. CBD exploration
No abx in options
Answeris:A
.......
🍄supracondylar fracture in a child w/ no distal pulse mx?
A. Early k wire B. surgical exploration
........
🍄pt went for percutaneous cholecystectomy then develop upper gi bleeding what to do ?
A. Endo B. ct abd C. angie ✅
.....
pt has something with gallbladder develop sins of peritonitis and asked about the
source of the organism ?
A. Hematogenous , B. perforation
C. Translocation ✅
....
🍄Post-Colectomy for colon cancer she is Diabetic, received dextrose and Insulin for 2 days, then she developed confusion and
agitation. Lab hypoNA hypoK, urine osmolality normal, serum osmo: 270. Most likely cause?
A.Water overload ✅B.Adrenal crises C.SIADH
.......
Hard signs of vascular injury
A weak pulses B audible bruit ✅
C blue color tissue
....
🍄40 yo Patient came with severe epigastric pain after meal, and minimal burning sensation, what is most important to reach
diagnosis?
A.Endoscopy B.Barium swallow
C.US abdomen D. H.pylori AG ✅✅
.......
🍄Elderly sudden abd pain and distontion, absolute constipation, x ray air fluid level , goes to RUQ, Dx?
A.Sigmoid volvulus✅ B.Colon cancer
C.Rectal cancer
........
🍄6 days with abd distention not passing stool, empty rectum, gush of stool with finger withdrawal. Dx?
A.Biliary atresia B.Hirschsprung ✅
C.Midgut volvulus
.......
🍄Female 64 yo , came with severe epigastric pain, radiating to the back associated with vomiting, 6 hours ago, kc of multiple
gallstones, upon examination: diffuse abdominal distention, and epigastric tenderness, erect chest x ray : left pleural effusion,
ALP, B total, amylase all normal what is the most app inv to reach diagnosis?
A. Abd US B.Abd CT✅ C.Barium enema
D.Barium swallow
.......
🍄Pt has (echinococcus ) hydatid cyst , most appropriate ttt ? (( no size i swear))
A- metronidazole
B-surgical excision percutaneous drainage
C- deroofing surgery
D- Albendazole therapy ✅
......... .
Pancreatic calcified and fat in stool what to do ?
A. tell him to decrease fat intake ✅
B. give lipase 30000 with each meal
.........
🍄Old patient )50 years old. Bloody from rectum and weight loss with abdominal pain . Physical is normal. Best Next stop in
management?
- Sigmoidectomy - Colonoscopy ✅
- CT abdomen
........
🍄Female 57 year old presented with severe left lower abdominal pain, Vomiting, and abdominal distention for 6 hours , WBC
high , likely diagnosis
A-mesenteric ischemia B-Perforated viscus
C-diverticulitis ✅ D-peptic ulcer
. ......
Pregnant at 10 weeks of gestation with Recurrent Biliary colic for 6 weeks when do
cholecystectomy ?
A- immediate B-2nd trimesters ✅
C- After delivery D- 3rd trimester
.....
Scrotum red but not tender , horizontal lie ?
A- surgical exploration B- ABx C- observe
D- US ✅
Note: questionable dx since it is non-tender and no mention of absent cremasteric reflex is
being made. Most likely testicular torsion (comes with tenderness).
....
🍄Eldely, weight loss , vague epigastric pain, loss of appetite Most appropriate investigation :
A-Gastroduodenoscopy ✅
B-Abdominal CT
........
🍄Case of achalasia after esophageal dilation procedure, complicated with rupture management:
A- esophageal stent ✅
B- Esophagectomy
.......
🍄Case of esophageal high grade dysplasia = Refer for possible surgical resection
........
🍄67 years old male bedridden in hospital for many years because of recurrent strokes is found how to have sacral ulcers. On
examination ulcer was involving the sacral and gluteal region with necrosis of the skin and subcutaneous tissues. What is the
best management plan?
A- Debridement with dressing
B- Debridement with primary closure
C- Debridement with skin graft ✅
.......
Child pass stool after 73 h as yellow and delayed Dx as intussusception , What is the
most important appropriate management :-
A- resection B- Steroid
C- Decompression by Enema ✅
.........
Patient with painful mass diagnosed as strangulated hernia, at the surgery the neck of the hernia was found to be inferior and
lateral to the pubic tuberclo, what the type of this hernia? A- femoral hernia ✅
......
Child with inguinal hernia without family ( abundant) and Doctor see him and decide to do surgery. What to do?
A- Do surgery without consent
B- talk to hospital committee ✅
......
🍄Pt with swelling and chronic cough you prescribed medication 6 month ago and he did take it and come now he said if you
don't do the surgery I will Complaint against you. What to do?
A- Do surgery B- refuse to do surgery ✅
C- talk to hospital committee
.........
🍄Lobectomy and 9mm papillary carcinoma left distant from the removed lobe, no lymph or vascularinvasion ' observe/follow
up Radioactive =
Total thyroidectomy
.......
Small lesion (7mm) found in asymptomatic, non smoker, young man, what to do:
A. Biopsy of lesion
B. Repeat CT after 6 months
C. compare with old x-ray ✅
.........
🍄Farmer in 405 came with RUQ pain and fever, vomiting, eam:' RUQ tenderness, CT:
hydatid cyst 7x6cm, most app management?
A.metronidazole B.Steroid
C.Percutaneous cath aspiration.
D.Surgical excision. ✅
.......
🍄3w post sleeve came with progressive pain, what's the most appropriate next step in management?
A.Endoscopy B.Diagnostic laparoscopy
C.EXP laparotomy D.NPO ✅
.......
🍄Suspected Sarcoma, What's the most appropriate diagnostic biopsy?
A.incisional. B.excisional. C.core needle.
.....
🍄Pt KC of crohns on infliximab and azithromycin, came with perianal purulent discharge no pain, no tenderness no fever,only
induration and discharge, what's the most app next step in management?
A.increase infliximab dose and decrease frequency.
B. broad spectrum Abx. C.pelvic MRI ✅
.....
5 Nov 2019

5 Nov 2019

4 Nov 2019

You might also like